You are on page 1of 74

Labor Law 1

DMA SHIPPING PHILS INC V CABILLAR


452 SCRA 551
CALLEJO, SR; February 28, 2005
NATURE
Petition for Review
FACTS
- Henry Cabillar was hired by Monsoon, through DMA Shipping,
as Chief Officer of the M/V Eagle Moon.
- After three (3) months, Cabillar wrote the manager of
Monsoon, requesting for an early repatriation and for his
reliever grounded on the failure of DMA Shipping to give the
promised additional allowance. Monsoon approved an increase
in Cabillars wage and the latter withdrew his request for
repatriation.
- While the vessel was docked in India, the gantry crane
operators refused to work and demanded for an increase in
their allowance. The master of the M/V Eagle Moon instructed
Cabillar to talk to the crew members under his immediate
supervision to convince them not to proceed with the intended
strike and have the matter discussed with the management
when the vessel returns to Singapore.
- Instead of talking to the crew members, Cabillar himself joined
the strike. Monsoon expressed its displeasure on Cabillar for
joining the strike. Nevertheless, Monsoon agreed to the
demands of the striking crew members to avert any further
losses.
- When the vessel arrived at Singapore, officers of Monsoon
informed Cabillar that he has been separated from his
employment because of the incident in Calcutta.
- Cabillar filed a complaint with the POEA against DMA and
Monsoon seeking payment for the unexpired portion of his
contract.
- The Executive Labor Arbiter rendered a decision in favor of
Cabillar declaring his dismissal as illegal. The NLRC and the
Court of Appeals affirmed. Hence this petition.
ISSUES
1.
WON the respondent was dismissed by the petitioner
Monsoon and
2. If so, WON his dismissal was for a valid cause
3. WON the respondent is entitled to backwages, damages and
attorneys fees
HELD
1. Petitioner was dismissed.
Ratio WON the respondent was dismissed or that he resigned
as chief officer of the vessel is a question of fact. The labor
arbiter ruled that the respondent was dismissed. The NLRC and
the Court of Appeals affirmed. Petitioners failed to make a clear
showing
that the findings were are arbitrary and bereft of any rational
basis.
Reasoning
- The entry in the logbook of the vessel shows that the ship
captain, for and in behalf of the petitioners, dismissed the
respondent for joining the strike.
- The petitioners failed to adduce documentary evidence to
prove their allegation that (1) they and the respondent agreed
that in consideration for the respondents resignation, they
would give him a very good rating (2) they defrayed his plane
fare back to the Philippines (3) they paid for his hotel bills in
Singapore.
2. Respondent was dismissed for just cause.
Ratio Under their employment contracts, the crew and officers
of the vessel bound themselves to follow certain procedures for
their grievances.
Reasoning
- The crew and the respondent refused to follow the procedure
and stop the strike.

A2010

Disini

- 37 -

- They may have a valid grievance against the petitioners but


they are bound to follow the procedures set forth in their
contracts of employment to address said grievances.
3. Petitioners are to pay indemnity.
Ratio The petitioners themselves violated their contracts of
employment with the respondent and the crew because the
captain of the vessel failed to comply with the disciplinary
procedures.
Reasoning
- The respondent was not furnished with any written notice of
any charges against him.
- There was no formal investigation of the charges.
- Respondent was not furnished with a copy of the written
notice of the penalty imposed on him.
- For such violation, petitioners are liable for moral damages or
for indemnity of P30,000, if the respondent fails to prove such
moral damages.1 In this case, the respondent failed to prove
such moral damages.
Disposition AFFIRMED with MODIFICATION. Petitioners are
ordered to pay P30,000
by way of indemnity. The awards for other damages and
attorneys fees are deleted.

UY V BUENO
484 SCRA 628
PUNO; March 14, 2006
NATURE
Petition for review on certiorari of a decision of the CA
FACTS
- Amalia Bueno was the Manager of Countrywide Rural Bank of
La Carota, Inc. (bank hereafter) Marbel Branch. She was
verbally and summarily dismissed by Atty. Andrea Uy, interim
President and Corporate Secretary of the bank, during a
depositors' meeting.
- Bueno filed a case for illegal dismissal and prayed for
reinstatement with full backwages and damages.
ISSUE
WON Uy is an officer of the bank, making her soldarily liable
with the corporation for illegal dismissal
HELD
NO
- The minutes of the depositors' meeting clearly showed that Uy
was a mere depositor of the bank. She was only elected as
officer of the Interim Board of Directors craeted by the
association of depositors with the sole task of rehabilitating the
bank (which is under receivership).
- There is no evidence that the association of depositors that
elected the interim board was recognized by BSP. Hence, it had
no legal authority to act for the bank.
- The act of dismissing Bueno by Uy cannot be deemed as an
act as an officer of the bank. Consequently, it cannot be held
that there existed an employer-employee relationship between
Uy and Bueno.
- The requirement of employer-employee relationship is
jurisdictional for the provisions of the Labor Code on Postemployment to apply. Since such relationship was not
established, the labor arbiter never acquired jurisdiction over
Uy.
Disposition CA decision finding Uy solidarily liable with the
bank reversed

CABRERA V NLRC (NATIONAL SERVICE


CORPORATION, VILLAMOR)
198 SCRA 573
1

Agabon v. NLRC, G.R. No. 158693, 442 SCRA 573, Nov. 17, 2004.

Labor Law 1
CRUZ; June 27, 1991
NATURE
Appeal from the decision of the NLRC dismissing the complaint
for illegal dismissal by the petitioners on the ground that it is
without jurisdiction.
FACTS
- Dismissed by the National Service Corporation, the petitioners
complained to the Ministry of Labor and Employment on
September 17, 1980. After considering the position papers of
the parties, the Labor Arbiter ordered the petitioners'
reinstatement without loss of seniority rights and the payment
to them of two years back wages and other benefits. 3 The
decision was appealed to and affirmed by the First Division of
the NLRC on December 9, 1985, and in due time, the petitioners
moved for the issuance of a writ of execution. This was opposed
by NASECO on the ground that it had not been furnished with a
copy of the decision, but the opposition was rejected and the
petition was granted. Reconsideration of the order having been
denied, the NASECO appealed to the NLRC, which, through its
Third Division this time, declared itself without jurisdiction and
dismissed the case on August 18, 1987. 4 Citing the NHA case,
the public respondent held that the NASECO was not covered by
the Labor Code but by Civil Service rules and regulations, being
a government-owned or controlled corporation.
ISSUE
WON the National Service Corporation is covered by the Labor
Code
HELD
YES
- The decision in National Housing Corporation v. Juco was
already overturned by the decision in National Service
Corporation v. NLRC. The NLRC erred in dismissing the
petitioners' complaint for lack of jurisdiction because the rule
now is that only government-owned or controlled corporations
with original charters come under the Civil Service. The NASECO
having been organized under the Corporation Law and not by
virtue of a special legislative charter, its relations with its
personnel are governed by the Labor Code and come under the
jurisdiction of the National Labor Relations Commission.

GAMUGAMO V PNOC SHIPPING AND TRANSPORT


CORP
381 SCRA742
DAVIDE JR; May 7, 2002
NATURE
Special Civil Action in the Supreme Court. Certiorari
FACTS
- On January 23,1963, petitioner Cayo Gamogamo was
employed with the Department of Health as Dental Aide
(wherein he was also promoted to the position of Dentist 1). He
remained employed at the DOH for 14 years until he resigned
on November 2, 1977.
- On November 9, 1977, petitioner was hired as a company
dentist by Luzon Stevedoring Corporation (LUSTEVECO).
Subsequently, respondent PNOC Shipping and Transport Corp
acquired and took over the shipping business of LUSTEVECO.
Petitioner was among those who opted to be absorbed by the
respondent. He continued to work as a company dentist.
- ON June 10,1993, President Fidel V. Ramos issued a
memorandum approving the privatization of PNOC subsidiaries,
including respondent pursuant to the provisions of Section III (B)
of the Guidelines and Regulations to implement E.O. No. 37.
Respondent implemented a Manpower Reduction Program to
govern employees whose respective positions have been
classified as redundant (respondent decreased its operations

A2010

- 38 -

Disini

and downsized its organization due to lay up and sale of its


vessels.
- Sometime in 1995, petitioner requested to be included in the
next retrenchment schedule. However, his request was turned
down because: 1.) he was holding a permanent position 2.) he
was already due for mandatory retirement in April 1995 under
his retirement plan. Eventually petitioner retired after serving
respondent for 17 years and 4 mos. He received a retirement
pay which is equivalent to one month pay for every year of
service and other benefits (P512,524.15)
- On August 30,1995, respondents president died and was
replaced by Nemesio Prudente who implemented significant
cost-saving measures. He ordered that a study on the costeffect of the retrenchment of employees be conducted (upon
motion of 2 other employees, Dr. Rogelio Buena (company
doctor) and Mrs. Luz C. Reyes (telephone operator) who were
holding
permanent/non-redundant
positions.
These
2
employees were retrenched and paid a 2-month separation pay
for every year of service under Respondents Manpower
Reduction Program.
- In view of the action taken by respondent in the retrenchment
of the said 2 employees, petitioner filed a complaint at the
NLRC for the full payment of his retirement benefits. Petitioner
argued that his service with the DOH should have been included
in the computation of his years of service. Hence, with an
accumulated service of 32 years he should have been paid a 2month pay for every year of service per the retirement plan
(and thus should have received at least P1,833,920)
- The Labor Arbiter dismissed petitioners complaint. On appeal
however, the NLRC reversed the decision of the Labor Arbiter
(considering the 14 years of his service to DOH)
- Respondent filed with the CA a special civil action for
certiorari. CA set aside the judgment of the NLRC. Hence,
petitioner filed this petition alleging that 1.) his years of service
with the DOH must be considered as creditable service for the
purpose of computing his retirement pay 2.) he was
discriminated against in the application of the Manpower
Reduction Program.
ISSUE
WON, for the purpose of computing an employees retirement
pay, prior service rendered in a government agency can be
tacked in and added to the creditable service later acquired in a
government-owned and controlled corporation without original
charter.
HELD
NO
- Petition denied and the appealed decision in CA is affirmed (in
favor of respondent)
Ratio The Court cannot uphold petitioners contention that his
14 years of service with the DOH should be considered because
his last 2 employers were government-owned and controlled
corporations and fall under the Civil Service Law. Article IX (B),
Section 2 paragraph 1 of the 1987 Constitution states: The civil
service embraces all branches, subdivisions, instrumentalities,
and agencies of the Government, including government owned
or controlled corporations with original charters. While
respondent and LUSTEVECO are government-owned and
controlled corporations, they have no original charters; hence,
they are not under the Civil Service Law
Reasoning
- totalization of service credits is only resorted to when the
retiree does not qualify for benefits in either of both systems. In
this case, petitioner is qualified to receive benefits granted by
the GSIS, if such right has not yet been exercised.
- It may also be pointed out that upon his receipt of the amount
of P512,524.15 from respondent as retirement benefit pursuant
to its retirement scheme, petitioner signed and delivered to
respondent a release and undertaking wherein he waives all
actions, causes of actions, debts, dues, monies and accounts in
connection with his employment with respondent.
This

Labor Law 1
quitclaim releases respondent from any other obligation in favor
of petitioner.

LIGHT RAILWAY TRANSIT AUTHORITY V VENUS


485 SCRA 301
PUNO; March 24, 2006
FACTS
- consolidated petitions of Light Rail Transit Authority (LRTA)
and Metro Transit Organization, Inc. (METRO), seeking the
reversal of the Decision of the Court of Appeals directing them
to reinstate private respondent workers to their former
positions without loss of seniority and other rights and
privileges, and ordering them to jointly and severally pay the
latter their full back wages, benefits, and moral damages. The
LRTA and METRO were also ordered to jointly and severally pay
attorneys fees equivalent to ten percent (10%) of the total
money judgment.
- Petitioner LRTA is a government-owned and controlled
corporation created by Executive Order No. 603, Series of 1980,
as amended, to construct and maintain a light rail transit
system and provide the commuting public with an efficient,
economical, dependable and safe transportation. Petitioner
METRO, formerly Meralco Transit Organization, Inc., was a
qualified transportation corporation duly organized in
accordance with the provisions of the Corporation Code,
registered with the Securities and Exchange Commission, and
existing under Philippine laws. petitioner LRTA, after a bidding
process, entered into a ten (10)-year Agreement for the
Management and Operation of the Metro Manila Light Rail
Transit System from June 8, 1984 until June 8, 1994 with
petitioner METRO.
- The Agreement provided, among others, that
4. METRO shall be free to employ such employees and
officers as it shall deem necessary in order to carry out the
requirements of [the] Agreement. Such employees and
officers shall be the employees of METRO and not of the
AUTHORITY [LRTA]. METRO shall prepare a compensation
schedule and the corresponding salaries and fringe benefits
of [its] personnel in consultation with the AUTHORITY [LRTA]
[par. 3.05];
- On July 25, 2000, the Union filed a Notice of Strike with the
National Conciliation and Mediation Board National Capital
Region against petitioner METRO on account of a deadlock in
the collective bargaining negotiation. On the same day, the
Union struck. They completely paralyzed the operations of the
entire light rail transit system. As the strike adversely affected
the mobility of the commuting public, then Secretary of Labor
Bienvenido E. Laguesma issued on that same day an
assumption of jurisdiction order [3] directing all the striking
employees to return to work immediately upon receipt of this
Order and for the Company to accept them back under the
same terms and conditions of employment prevailing prior to
the strike
- Despite the issuance, posting, and publication of the
assumption of jurisdiction and return to work order, the Union
officers and members failed to return to work. Thus, effective
July 27, 2000, private respondents, were considered dismissed
from employment
- Workers filed a complaint for illegal dismissal. On October 1,
2001, Labor Arbiter Luis D. Flores rendered a consolidated
judgment in favor of the private respondent workers
- On May 29, 2002, on appeal, the NLRC found that the striking
workers failed to heed the return to work order and reversed
and set aside the decision of the labor arbiter. The suit against
LRTA was dismissed since LRTA is a government-owned and
controlled corporation created by virtue of Executive Order No.
603 with an original charter and it ha[d] no participation
whatsoever
with
the
termination
of
complainants
employment. In fine, the cases against the LRTA and METRO

A2010

- 39 -

Disini

were dismissed, respectively, for lack of jurisdiction and for lack


of merit.
- On a petition for certiorari however, the Court of Appeals
reversed the NLRC and reinstated the Decision rendered by the
Labor Arbiter. Public respondent appellate court declared the
workers dismissal as illegal, pierced the veil of separate
corporate personality and held the LRTA and METRO as jointly
liable for back wages.
ISSUE
WON LRTA should be held liable for the illegal dismissal of
employees
HELD
NO
- petitioner LRTA argues that it has no employer-employee
relationship with private respondent workers as they were hired
by petitioner METRO alone pursuant to its ten (10)-year
Agreement for the Management and Operation of the Metro
Manila Light Rail Transit System with petitioner METRO. Piercing
the corporate veil of METRO was unwarranted, as there was no
competent and convincing evidence of any wrongful, fraudulent
or unlawful act on the part of METRO, and, more so, on the part
of LRTA.
- Petitioner LRTA further contends that it is a governmentowned and controlled corporation with an original charter,
Executive Order No. 603, Series of 1980, as amended, and thus
under the exclusive jurisdiction only of the Civil Service
Commission, not the NLRC.
- We agree with petitioner LRTA. Section 2 (1), Article IX B,
1987 Constitution, expressly provides that [t]he civil service
embraces all branches, subdivisions, instrumentalities, and
agencies of the Government, including government-owned or
controlled corporations with original charters. Corporations
with original charters are those which have been created by
special law and not through the general corporation law.
- In Philippine National Oil Company Energy
Development Corporation v. Hon. Leogrado- under the
present state of the law, the test in determining whether a
government-owned or controlled corporation is subject to the
Civil Service Law is the manner of its creation such that
government corporations created by special charter are subject
to its provisions while those incorporated under the general
Corporation Law are not within its coverage.
- There should be no dispute then that employment in petitioner
LRTA should be governed only by civil service rules, and not the
Labor Code and beyond the reach of the Department of Labor
and Employment, since petitioner LRTA is a government-owned
and controlled corporation with an original charter, Executive
Order No. 603, Series of 1980
- In contrast, petitioner METRO is covered by the Labor Code
despite its later acquisition by petitioner LRTA. In Lumanta v.
National Labor Relations Commission, this Court ruled that
labor law claims against government-owned and controlled
corporations without original charter fall within the jurisdiction
of the Department of Labor and Employment and not the Civil
Service Commission
- We therefore hold that the employees of petitioner METRO
cannot be considered as employees of petitioner LRTA. The
employees hired by METRO are covered by the Labor Code and
are under the jurisdiction of the Department of Labor and
Employment, whereas the employees of petitioner LRTA, a
government-owned and controlled corporation with original
charter, are covered by civil service rules. Herein private
respondent workers cannot have the best of two worlds, e.g., be
considered government employees of petitioner LRTA, yet
allowed to strike as private employees under our labor laws.

EBRO III V NLRC (INTERNATIONAL CATHOLIC


MIGRATION COMMISSION)

Labor Law 1
261 SCRA 399
MENDOZA; September 4, 1996
NATURE
Petition for review on certiorari to set aside the order dated
October 13, 1992 and the resolution dated March 3, 1993 of the
National Labor Relations Commission.
FACTS
Private
respondent
International
Catholic
Migration
Commission (ICMC) is a non-profit agency engaged in
international humanitarian and voluntary work. It is duly
registered with the United National Economic and Social Council
(ECOSOC) and enjoys Consultative Status, Category II. It was
one of the agencies accredited by the Philippine Government to
operate the refugee processing center at Sabang, Morong,
Bataan.
- On June 24, 1985, private respondent ICMC employed
petitioner Jose G. Ebro III to teach "English as a Second
Language and Cultural Orientation Training Program" at the
refugee processing center. The employment contract provided
in pertinent part:
Salary: Your monthly salary for the first 6 months
probationary period is P3,155.00 inclusive of cost of living
allowance. Upon being made regular after successful
completion of the six (6) months probationary period your
monthly salary will be adjusted to P3,445.00 inclusive of cost
of living allowance
If either party wishes to terminate employment, a notice of
two (2) weeks should be given in writing to the party.
- After six months, ICMC notified petitioner that effective
December 21, 1985, the latter's services were terminated for
his failure to meet the requirements of "1. classroom
performance . . . up to the standards set in the Guide for
Instruction; 2. regular attendance in the mandated teacher
training, and in the schedule team meetings, one-on-one
conferences with the supervisor, etc.; and 3. compliance with
ICMC and PRPC policies and procedures."
- On February 4, 1986, petitioner filed a complaint for illegal
dismissal, unfair labor practice, underpayment of wages,
accrued leave pay, 14th month pay, damages, attorney's fees,
and expenses of litigation. Petitioner alleged that there was no
objective evaluation of his performance to warrant his dismissal
and that he should have been considered a regular employee
from the start because ICMC failed to acquaint him with the
standards under which he must qualify as such. He prayed for
reinstatement with backwages; P3,155.00 for probationary and
P3,445.00 for regular salary adjustments; value of lodging or
dormitory privileges; cost of insurance coverage for group life,
medical, death, dismemberment and disability benefits; moral,
and exemplary, and nominal damages plus interest on the
above claims with attorney's fees.
- Answering the complaint, ICMC claimed that petitioner failed
to quality for regular employment because he showed no
interest in improving his professional performance both in and
out of the classroom after he had been periodically evaluated;
that petitioner was paid his salary up to December 31, 1985,
two weeks pay in lieu of notice, and 14th month pay pro-rata;
and that his accrued leave balance already been converted to
cash.
- After the parties had formally offered their evidence, private
respondents submitted their memorandum on July 31, 1989 in
which, among other things, they invoked ICMC's diplomatic
immunity on the basis of the Memorandum of Agreement
signed on July 15, 1988 between the Philippines government
and ICMC.
- The Labor Arbiter held that ICMC's legal immunity under the
Memorandum could not be given retroactive effect since "[that
would] deprive complainant's property right without due
process and impair the obligation of contract of employment."
He also expressed doubt on the ground that it was provided for
by agreement and not through an act of Congress. Accordingly,

A2010

- 40 -

Disini

the Labor Arbiter ordered ICMC to reinstate petitioner as


regular teacher without loss of seniority rights and to pay him
one year backwages, other benefits, and ten percent attorney's
fees for a total sum of P70,944.85.
- Both parties appealed to the NLRC. On August 13, 1990,
petitioner moved to dismiss private respondent's appeal
because of the latter's failure to post a cash/surety bond. In its
order of October 13, 1992, however, the NLRC ordered the case
dismissed on the ground that, under the Memorandum of
Agreement between the Philippine government and ICMC, the
latter was immune from suit.
ISSUE
WON the Memorandum of Agreement executed on July 15, 1988
granted ICMC immunity from suit
HELD
YES
Ratio The grant of immunity from local jurisdiction to ICMC . . .
is clearly necessitated by their international character and
respective purposes. The objective is to avoid the danger of
partiality and interference by the host country in their internal
workings. The exercise of jurisdiction by the Department of
Labor in these instances would defeat the very purpose of
immunity, which is to shield the affairs of international
organizations, in accordance with international practice, from
political pressure or control by the host country to the prejudice
of member State of the organization, and to ensure the
unhampered performance of their functions. (International
Catholic Migration Commission v. Calleja)
Reasoning
- The grant of immunity to ICMC is in virtue of the Convention
on the Privileges and Immunities of Specialized Agencies of the
United Nations, adopted by the UN General Assembly on
November 21, 1947, and concurred in by the Philippine Senate
on May 17, 1949. This Convention has the force and effect of
law, considering that under the Constitution, the Philippines
adopts the generally accepted principles of international law as
part of the law of the land.
- The scope of immunity of the ICMC contained in the
Convention on the Privileges and Immunities of the Specialized
Agencies of the United Nations is instructive. Art. III, 4 of the
Convention provides for immunity from "every form of legal
process." Thus, even if private respondents had been served
summons and subpoenas prior to the execution of the
Memorandum, they, as officers of ICMC, can claim immunity
under the same in order to prevent enforcement of an adverse
judgment, since a writ of execution is "a legal process" within
the meaning of Article III, 4.
- Art III 4 of the Convention on the Privileges and Immunities of
the Specialized Agencies of the United Nations requires that the
waiver of the privilege must be express. There was no such
waiver of immunity in this case. Nor can ICMC be estopped from
claiming diplomatic immunity since estoppel does not operate
to confer jurisdiction to a tribunal that has none over a cause of
action.
- Finally, neither can it be said that recognition of ICMC's
immunity from suit deprives petitioner of due process. As
pointed out in ICMC v. Calleja, petitioner is not exactly without
remedy for whatever violation of rights it may have suffered for
the following reason:
Section 31 of the Convention on the Privileges and Immunities
of the Specialized Agencies of the United Nations provides that
"each specialized agency shall make provision for appropriate
modes of settlement of: (a) disputes arising out of contracts or
other disputes of private character to which the specialized
agency is a party." Moreover, pursuant to Article IV of the
Memorandum of Agreement between ICMC and the Philippine
Government, whenever there is any abuse of privilege by ICMC,
the Government is free to withdraw the privileges and
immunities accorded.
Disposition Petition is DISMISSED for lack of merit.

Labor Law 1
NATIONAL MINES AND ALLIED WORKERS UNION V
SAN ILDEFONSO COLLEGE
CHIANG KAI SHEK COLLEGE V CA (NLRC,
CALAYLAY, AQUINO, GACUTAN, BELO)
437 SCRA 171
DAVIDE, JR; August 24, 2004
FACTS
- In 1992, Ms. Diana Belo, a teacher of Chiang Kai Shek College
since 1977, applied for a leave of absence for the school year
1992-1993. Upon submitting her application, she was informed
of the school policy that if she takes a leave of absence, she is
not assured of a teaching load upon her return. She was
likewise informed that only teachers in active service may enjoy
the privilege and benefits provided by the school, such as free
tuition for the teachers children.
- Ms. Belo, nonetheless, took her leave of absence. In May 1993,
she attempted to return to CKSC and signified her readiness to
teach for the coming school year. However, she was not allowed
to return. Hence, she filed a complaint for illegal dismissal,
among others, against CKSC.
- The Labor Arbiter dismissed the complaint but the NLRC
disagreed. The Court of Appeals upheld the NLRCs ruling.
Hence, this petition.
ISSUE
WON private respondent was constructively dismissed
HELD
YES
- Under the Manual of Regulations for Private Schools, for a
private school teacher to acquire a permanent status of
employment and, therefore, be entitled to a security of
tenure, the following requisites must concur: (a) the
teacher is a full-time teacher; (b) the teacher must have
rendered three consecutive years of service; and (c)
such service must have been satisfactory. Since Ms. Belo
has measured up to these standards, she therefore enjoys
security of tenure.
- Constructive dismissal is defined as a cessation from work
because continued employment is rendered impossible,
unreasonable, or unlikely; when there is a demotion in rank or a
diminution in pay or both; or when a clear discrimination,
insensibility, or disdain by an employer becomes unbearable to
the employee.
- Ms. Belo was constructively dismissed when the petitioners, in
implementing their policies, effectively barred her from
teaching for the school year 1993-1994. The three policies are
(1) the non-assurance of a teaching load to a teacher who took
a leave of absence; (2) the hiring of non-permanent teachers in
April to whom teaching loads were already assigned when Ms.
Belo signified in May 1993 her intention to teach; and (3) the
non-applicability to children of teachers on leave of the free
tuition fee benefits extended to children of teachers in service.
- Ms. Belo was definitely singled out in the implementation of a
future policy (i.e., the policy that employees not in service are
not entitled to any benefit extended by the school). The
petitioners did not take heed of the principle enshrined in our
labor laws that policies should be adequately known to the
employees and uniformly implemented to the body of
employees as a whole and not in isolation.
- The continued employment of Ms. Belo was also rendered
unlikely by the insistence of the petitioners in implementing the
alleged policy that a teacher who goes on leave for one year is
not assured of a teaching load. While this alleged policy was
mentioned in Mr. Chiens letter of 9 June 1992, it was not
included in the schools written statement of policies dated 13
March 1992. Hence, it was then a non-existent policy. When a
non-existent policy is implemented and, in this case, only to Ms.
Belo, it constitutes a clear case of discrimination.

A2010

- 41 -

Disini

- Petitioners invocation of the third policy that of giving


teaching assignments to probationary teachers in April to
justify their refusal to provide Ms. Belo a teaching load is a lame
excuse that rings of untruth and dishonesty. Patently clear is
the illegal manner by which the petitioners eased out Ms. Belo
from the teaching corps.
- Likewise, we do not find merit in petitioners assertion that the
Court of Appeals should not have passed upon the illegality of
the school policy of non-assurance of a teaching load, since the
alleged illegality was never raised as an issue before the
respondent court or in the forums below. As pointed out by the
private respondent, that policy was part of the defense invoked
by the petitioners in the Arbiter level, in the NLRC, and in the
respondent court to the charge of illegal dismissal; and, hence,
it must necessarily be passed upon and scrutinized. Besides,
that policy is intimately intertwined with the main issue of
whether Ms. Belo was illegally dismissed.
- This case is an exception to the general rule that the factual
findings and conclusions of the Labor Arbiter are accorded
weight and respect on appeal, and even finality. For one thing,
the findings of the NLRC and the Labor Arbiter are contrary to
each other; hence, the reviewing court may delve into the
records and examine for itself the questioned findings.
Disposition The Petition is DENIED.

AUSTRIA V NLRC (CENTRAL PHIL. UNION MISSION


CORP. OF THE 7TH-DAY ADVENTIST)
312 SCRA 410
KAPUNAN; August 16, 1999
FACTS
- Pastor Dionisio Austria worked with the Central Philippine
Union Mission Corporation of the Seventh Day Adventists (SDA)
for 28 years from 1963 to 1991. He began his work with the
SDA on 15 July 1963 as a literature evangelist, selling literature
of the SDA over the island of Negros. From then on, he worked
his way up the ladder and got promoted several times. In
January, 1968, he became the Assistant Publishing Director in
the West Visayan Mission. In July, 1972, he was elevated to the
position of Pastor covering the island of Panay, and the
provinces of Romblon and Guimaras. He held the same position
up to 1988. Finally, in 1989, he was promoted as District Pastor
of the Negros Mission of the SDA and was assigned at Sagay,
Balintawak and Toboso, Negros Occidental, with 12 churches
under his jurisdiction. In January, 1991, he was transferred to
Bacolod City. He held the position of district pastor until his
services were terminated on 31 October 1991.
- On various occasions from August up to October, 1991,
Eufronio Ibesate, the treasurer of the Negros Mission asked him
to admit accountability and responsibility for the church tithes
and offerings collected by his wife, Thelma Austria, in his
district which amounted to P15,078.10, and to remit the same
to the Negros Mission. Petitioner reasoned out that he should
not be made accountable for the unremitted collections since it
was Pastor Gideon Buhat and Ibesate who authorized his wife to
collect the tithes and offerings since he was very sick to do the
collecting at that time.
- On 16 October 1991, petitioner went to the office of Pastor
Buhat, the president of the Negros Mission. During said call,
petitioner tried to persuade Pastor Buhat to convene the
Executive Committee for the purpose of settling the dispute
between him and Pastor David Rodrigo. The dispute between
David Rodrigo and petitioner arose from an incident in which
petitioner assisted his friend, Danny Diamada, to collect from
Pastor Rodrigo the unpaid balance for the repair of the latter's
motor vehicle which he failed to pay to Diamada. Due to the
assistance of petitioner in collecting Pastor Rodrigo's debt, the
latter harbored ill-feelings against petitioner. When news
reached petitioner that Pastor Rodrigo was about to file a
complaint against him with the Negros Mission, he immediately
proceeded to the office of Pastor Buhat and asked the latter to

Labor Law 1
convene the Executive Committee. Pastor Buhat denied the
request of petitioner since some committee members were out
of town and there was no quorum. Thereafter, the two
exchanged heated arguments. Petitioner then left the office of
Pastor Buhat. While on his way out, petitioner overheard Pastor
Buhat saying "Pastor daw inisog na ina iya (Pastor you are
talking tough)." Irked by such remark, petitioner returned to the
office of Pastor Buhat, and tried to overturn the latter's table,
though unsuccessfully, since it was heavy. Thereafter,
petitioner banged the attache case of Pastor Buhat on the table,
scattered the books in his office, and threw the phone.
Fortunately, Pastors Yonillo Leopoldo and Claudio Montao were
around and they pacified both.
- On 17 October 1991, petitioner received a letter inviting him
and his wife to attend the Executive Committee meeting. From
October 21 to 22, the fact-finding committee conducted an
investigation. Petitioner immediately wrote Pastor Rueben
Moralde, president of the SDA and chairman of the fact-finding
committee, requesting that certain members of the fact-finding
committee be excluded in the investigation and resolution of
the case. Out of the 6 members requested to inhibit themselves
from the investigation and decision-making, only 2 were
actually excluded: Pastor Buhat and Pastor Rodrigo.
Subsequently, petitioner received a letter of dismissal citing
misappropriation of denominational funds, willful breach of
trust, serious misconduct, gross and habitual neglect of duties,
and commission of an offense against the person of employer's
duly authorized representative, as grounds for the termination
of his services.
ISSUES
1. WON the Labor Arbiter/NLRC has jurisdiction to try and
decide the complaint filed by petitioner against the SDA
2. WON the termination of the services of petitioner is an
ecclesiastical affair, and, as such, involves the separation of
church and state
3. WON such termination is valid
HELD
1. YES and 2. NO [Resolved jointly since they are related]
Ratio An ecclesiastical affair is one that concerns doctrine,
creed or form or worship of the church, or the adoption and
enforcement within a religious association of needful laws and
regulations for the government of the membership, and the
power of excluding from such associations those deemed
unworthy of membership.
Reasoning
- Based on this definition, an ecclesiastical affair involves the
relationship between the church and its members and relate to
matters of faith, religious doctrines, worship and governance of
the congregation. To be concrete, examples of this so-called
ecclesiastical affairs to which the State cannot meddle are
proceedings for excommunication, ordinations of religious
ministers, administration of sacraments and other activities
which attached religious significance. The case at bar does not
even remotely concern any of the above cited examples. While
the matter at hand relates to the church and its religious
minister it does not ipso facto give the case a religious
significance. Simply stated, what is involved here is the
relationship of the church as an employer and the minister as
an employee. It is purely secular and has no relation
whatsoever with the practice of faith, worship or doctrines of
the church. In this case, petitioner was not excommunicated or
expelled from the membership of the SDA but was terminated
from employment.
- Aside from these, SDA admitted in a certification issued by its
officer, Ibesate, that petitioner has been its employee for 28
years. SDA even registered petitioner with the SSS as its
employee. The worker's records of petitioner have been
submitted by private respondents as part of their exhibits. From
all of these it is clear that when the SDA terminated the
services of petitioner, it was merely exercising its management

A2010

- 42 -

Disini

prerogative to fire an employee which it believes to be unfit for


the job. As such, the State, through the Labor Arbiter and the
NLRC, has the right to take cognizance of the case.
- Finally, private respondents are estopped from raising the
issue of lack of jurisdiction for the first time on appeal. The
active participation of a party coupled with his failure to object
to the jurisdiction of the court or quasi-judicial body is
tantamount to an invocation of that jurisdiction and a
willingness to abide by the resolution of the case and will bar
said party from later on impugning the court or body's
jurisdiction.
3. NO.
Reasoning
- The issue being the legality of petitioner's dismissal, the same
must be measured against the requisites for a valid dismissal,
namely: (a) the employee must be afforded due process, i.e., he
must be given an opportunity to be heard and to defend
himself, and; (b) the dismissal must be for a valid cause as
provided in Article 282 of the Labor Code. Without the
concurrence of these twin requirements, the termination would,
in the eyes of the law, be illegal.
As to Due Process
- Article 277(b) of the Labor Code further require the employer
to furnish the employee with 2 written notices, to wit: (a) a
written notice served on the employee specifying the ground or
grounds for termination, and giving to said employee
reasonable opportunity within which to explain his side, and, (b)
a written notice of termination served on the employee
indicating that upon due consideration of all the circumstances,
grounds have been established to justify his termination.
- The first notice, which may be considered as the proper
charge, serves to apprise the employee of the particular acts or
omissions for which his dismissal is sought. The second notice
on the other hand seeks to inform the employee of the
employer's decision to dismiss him. This decision, however,
must come only after the employee is given a reasonable
period from receipt of the first notice within which to answer the
charge and ample opportunity to be heard and defend himself
with the assistance of a representative, if he so desires. Noncompliance therewith is fatal because these requirements are
conditions sine quo non before dismissal may be validly
effected.
- SDA failed to substantially comply with the above
requirements. With regard to the first notice, the letter dated 17
October 1991, which notified petitioner and his wife to attend
the meeting on 21 October 1991, cannot be construed as the
written charge required by law. A perusal of the said letter
reveals that it never categorically stated the particular acts or
omissions on which his impending termination was grounded. In
fact, the letter never even mentioned that he would be subject
to investigation. The letter merely mentioned that he and his
wife were invited to a meeting wherein what would be
discussed were the alleged unremitted church tithes and the
events that transpired on 16 October 1991. For this reason, it
cannot be said that petitioner was given enough opportunity to
properly prepare for his defense. While admittedly, SDA
complied with the second requirement, the notice of
termination, this does not cure the initial defect of lack of the
proper written charge required by law.
As to Just Cause
- Private respondents allege that they have lost their confidence
in petitioner for his failure, despite demands, to remit the tithes
and offerings which were collected in his district. Settled is the
rule that under Article 282 (c) of the Labor Code, the breach of
trust must be willful. A breach is willful if it is done intentionally,
knowingly and purposely, without justifiable excuse, as
distinguished from an act done carelessly, thoughtlessly,
heedlessly or inadvertently. It must rest on substantial grounds
and not on the employer's arbitrariness, whims, caprices or
suspicion, otherwise, the employee would eternally remain at
the mercy of the employer. It should be genuine and not
simulated. This ground has never been intended to afford an

Labor Law 1
occasion for abuse, because of its subjective nature. The
records show that there were only 6 instances when petitioner
personally collected and received from the church treasurers
the tithes, collections, and donations for the church. The
testimony of Naomi Geniebla, the Negros Mission Church
Auditor and a witness for private respondents, show that Pastor
Austria was able to remit all his collections to the treasurer of
the Negros Mission. Private respondents try to pin on petitioner
the alleged non-remittance of the tithes collected by his wife. In
the absence of conspiracy and collusion, which private
respondents failed to demonstrate, between petitioner and his
wife, he cannot be made accountable for the alleged infraction
committed by his wife. After all, they still have separate and
distinct personalities. Thus, the allegation of breach of trust has
no leg to stand on.
- Misconduct has been defined as improper or wrong conduct. It
is the transgression of some established and definite rule of
action, a forbidden act, a dereliction of duty, willful in character,
and implies wrongful intent and not mere error in judgment. For
misconduct to be considered serious it must be of such grave
and aggravated character and not merely trivial or unimportant.
Based on this standard, we believe that the act of petitioner in
banging the attache case on the table, throwing the telephone
and scattering the books in the office of Pastor Buhat, although
improper, cannot be considered as grave enough to be
considered as serious misconduct. After all, as correctly
observed by the Labor Arbiter, though petitioner committed
damage to property, he did not physically assault Pastor Buhat
or any other pastor present during the incident of 16 October
1991. In fact, the alleged offense committed upon the person of
the employer's representatives was never really established or
proven by private respondents. Hence, there is no basis for the
allegation that petitioner's act constituted serious misconduct
or that the same was an offense against the person of the
employer's duly authorized representative.
- The final ground alleged by private respondents, gross and
habitual neglect of duties, does not requires an exhaustive
discussion. All private respondents had were allegations but not
proof. Aside from merely citing the said ground, private
respondents failed to prove culpability. In fact, the evidence on
record shows otherwise. Petitioner's rise from the ranks proves
that he was actually a hard-worker. Private respondents'
evidence, which consisted of petitioner's Worker's Reports,
revealed how petitioner travelled to different churches to attend
to the faithful under his care. Indeed, he labored hard for the
SDA, but, in return, he was rewarded with a dismissal from the
service for a non-existent cause.
Disposition Finding of the Labor Arbiter that petitioner was
terminated from service without just or lawful cause is
SUSTAINED. Petitioner is entitled to reinstatement without loss
of seniority right and the payment of full back-wages without
any deduction corresponding to the period from his illegal
dismissal up to the actual reinstatement. Challenged Resolution
of NLRC is NULLIFIED and SET ASIDE.

PEARANDA V BAGANGA PLYWOOD CORP


[PAGE 1]
CBTC EMPLOYEES UNION V CLAVE
141 SCRA 9
DE LA FUENTE; January 7, 1986
NATURE
Petition for certiorari seeking to annul and set aside the
decision of the respondent Presidential Executive Assistant
FACTS
- Commercial Bank and Trust Company Employees' Union
lodged a complaint with the Department of Labor, against
Comtrust Bank for non-payment of the holiday pay benefits

A2010

- 43 -

Disini

provided for under Art 95 of the Labor Code in relation to Rule


X, Book III of the Rules and Regulations Implementing the Labor
Code.
- Failing to arrive at an amicable settlement at conciliation level,
the parties opted to submit their dispute for voluntary
arbitration. The issue presented was: "Whether the permanent
employees of the Bank within the collective bargaining unit paid
on a monthly basis are entitled to holiday pay effective
November 1, 1974, pursuant to Article 95 (now Article 94) of
the Labor Code, as amended and Rule X (now Rule IV), Book III
of the Rules and Regulations Implementing the Labor Code. "
- In addition, the disputants signed a Submission Agreement
stipulating as final, unappealable and executory the decision of
the Arbitrator, including subsequent issuances for clarificatory
and/or relief purposes, notwithstanding Article 262 of the Labor
Code.
- In the course of the hearing, the Arbitrator apprised the
parties of an interpretative bulletin on "holiday pay" about to be
issued by the Department of Labor.
- The Union filed a Manifestation stating that in the event that
said Interpretative Bulletin regarding holiday pay would be
adverse to the present claim union respectfully reserves the
right to take such action as may be appropriate to protect its
interests, a question of law being involved. An Interpretative
Bulletin which was inexistent at the time the said commitment
was made and which may be contrary to the law itself should
not bar the right of the union to claim for its holiday pay
benefits
- Voluntary Arbitrator stated that, there is more reason to
believe that, if the Bank has never made any deduction from its
monthly-paid employees for unworked Saturdays, Sundays,
legal and special holidays, it is because there is really nothing
to deduct properly since the monthly, salary never really
included pay for such unworked days-and which give credence
to the conclusion that the divisor '250' is the proper one to use
in computing the equivalent daily rate of the monthly-paid
employees; that both the decree itself and the Rules
mentioned enumerated the excepted workers. It is a basic rule
of statutory construction that putting an exception limits or
modifies the enumeration or meaning made in the law. It is thus
easy to see that a mere reading of the Decree and of the Rules
would show that the monthly-paid employees of the Bank are
not expressly included in the enumeration of the exception.
- Voluntary Arbitrator directed the bank to pay its monthly paid
employees their legal holiday pay.
- The next day, the Department of Labor released Policy
Instructions No. 9 which clarifies controversies on the
entitlement of monthly paid employees. The new determining
rule is this: If the monthly paid employee is receiving not less
than P 240, the maximum monthly minimum wage, and his
monthly pay is uniform from January to December, he is
presumed to be already paid the ten (10) paid legal holidays.
However, if deductions are made from his monthly salary on
account of holidays in months where they occur, then he is still
entitled to the ten (10) paid legal holidays.
- Bank appealed to NLRC but appeal was dismissed because it
was filed way beyond the ten-day period for perfecting an
appeal and because it contravened the agreement that the
award shall be final and unappealable.
- Acting Secretary of Labor reversed NLRC decision and ruled
that the appeal was filed on time and that a review of the case
was inevitable as the money claim exceeded P100,000.00.
- Presidential Executive Assistant affirmed DOJ ruling, relying
heavily on the Manifestation and Policy Instructions No. 9.
Petitioners Claim
The legal presumption established in Section 2, Rule IV, Book
1112, of the Rules and Regulations implementing particularly
2

SECTION 2. Status of employees paid by the month -Employees who are uniformly
paid by the month, irrespective of the number of' working days therein with a salary
of not less than the statutory or established minimum wage, shall be presumed to be
paid for all days in the month whether worked or not.
For this purpose, the monthly minimum wage shall not be less than the statutory
minimum wage multiplied by 365 days divided by twelve.

Labor Law 1
Article 94 (formerly Article 208) of the Labor Code, is merely a
disputable presumption
Respondents Comments
- The Bank maintains that, since its inception or start of
operations in 1954, all monthly-paid employees in the Bank are
paid their monthly salaries without any deduction for unworked
Saturdays, Sundays, legals and special holidays. On the other
hand, it also maintains that, as a matter of fact, 'always
conscious of its employee who has to work, on respondent's
rest days of Saturdays and Sundays or on a legal holiday, an
employee who works overtime on any of said days is paid one
addition regular pay for the day plus 50% of said regular pay
- The Bank further maintains that the holiday pay is intended
only for daily-paid workers.
- The question submitted for arbitration is now moot and
academic.
ISSUE
WON the permanent employees of the bank are entitled to
holiday pay
HELD
YES
- In excluding the union members of herein petitioner from the
benefits of the holiday pay law, public respondent predicated
his ruling on Section 2, Rule IV, Book III of the Rules to
implement Article 94 of the labor Code promulgated by the then
Secretary of labor and Policy Instructions No. 9.
- In Insular Bank of Asia and America Employees' Union
(IBAAEU) vs. Inciong, 7 this Court's Second Division, speaking
through former Justice Makasiar, expressed the view and
declared that the aforementioned section and interpretative
bulletin are null and void, having been promulgated by the then
Secretary of Labor in excess of his rule-making authority. It
was pointed out, inter alia, that in the guise of clarifying the
provisions on holiday pay, said rule and policy instructions in
effect amended the law by enlarging the scope of the
exclusions.
- The questioned Section 2, Rule IV, Book III of the Integrated
Rules and the Secretary's Policy Instruction No. 9 add another
excluded group, namely, 'employees who are uniformly paid by
the month'. While the additional exclusion is only in the form of
a presumption that all monthly paid employees have already
been paid holiday pay, it constitutes a taking away or a
deprivation which must be in the law if it is to be valid. An
administrative interpretation which diminishes the benefits of
labor more than what the statute delimits or withholds is
obviously ultra vires.
Disposition The questioned decision set aside and the award
of the arbitrator reinstated.

SONZA V ABS-CBN BROADCASTING CORPORATION


431 SCRA 587
CARPIO; June 10, 2004
NATURE
Petition for review on certiorari
FACTS
- ABS-CBN signed Agreement with Mel and Jay Management and
Devt Corp (MJMDC), which agreed to provide Sonzas services
exclusively to the network as talent for radio and TV.
- Sonza resigned and complained that network didnt pay his
salaries, separation pay, service incentive leave pay, etc. ABS
filed Motion to Dismiss because there was no employeremployee relationship. ABS continued to remit Sonzas monthly
talent fees.
- Labor Arbiter dismissed complaint because of lack of
jurisdiction. NLRC affirmed Arbiters decision. Sonza filed
certiorari action with CA, which dismissed the case. Hence this
petition.

A2010

- 44 -

Disini

ISSUE
WON there was an employer-employee relationship between
ABS-CBN and Sonza
HELD
NO
- This is the first Court resolution on nature of relationship
between a station and a talent.
- There are 4 elements of employer-employee relationship:
1. Selection of employee
- if Sonza didnt possess his skills, talents and celebrity
status, ABS-CBN would not have entered into agreement
with him but would have hired him through personnel
department
2. Payment of wages
- whatever Sonza received arose from the contract and not
from the employer-employee relation
- the talent fee is so huge that it indicates more a
contractual than an employment relationship
3. Power to dismiss
- ABS-CBN coulnt retrench Sonza because it is obligated to
pay talent fees for duration of contract
4. Control on employee on means and methods
- also called control test; most impt to determine
relationship
- Sonza contends ABS exercised control over means and
methods of his work. Court said ABS merely reserved the
right to modify the program format and airtime schedule.
Its sole concern was the quality of the show and the
ratings. How Sonza appeared, sounded, etc. is outside
control of ABS.
- Sonza contends that ABS exercised control in providing
equipment and crew.
Court said these are not tools
needed by Sonza. What he needed were his talent, skills,
costume.
- Sonza contends that ABS subjected him to rules and
standards. Court said that the rules are the TV and Radio
Code of the Kapisanan ng Broadcaster sa Pilipinas, merely
adopted by ABS as its code of ethics.
It applies to
broadcasters, not just to ABS employees. Besides, these
rules are merely guidelines.
- Sonza said his exclusivity is a form of control by ABS.
Court said exclusivity is a widespread practice in
entertainment industry, as protection of investment in
building up a talent. Besides, the huge talent fees of an
exclusive talent compensates for exclusivity.
- Arbiter ruled that as talent of MJMDC, Sonza is not an
employee of ABS. Sonza insists that MJMDC is a labor-only
contractor and ABS is his employer. In labor-only contract,
there are 3 parties the contractor, employee and the principal
(deemed the real employer). Under this, the contractor is the
agent of the principal. If Sonzas argument was true, then
MJMDC turns to be the agent of both Sonza and ABS. Besides,
in the Agreement, there were only two parties mentioned
Sonza and ABS, with MJMDC as Sonzas agent.
- Sonza argues Policy Instruction No. 40 by Minister of Labor
said the types of employees in broadcast are the station and
program employees. Court said this instruction is a mere
executive issuance not binding on the Court.
- Court also said that Arbiter can decide a case without a formal
trial.
- Sonza argues that treating talents as contractors violates right
to security of tenure. Court said this right exists only if there is
an employer-employee relation. Besides, law also protects
rights of talents to contract. Besides, if hosts were employees,
managers can dictate what hosts will say, and this is not
conducive to press freedom.
- Difference in tax treatment also showed that theres no
employer-employee relation.
- Sonzas claim is based on their agreement. Therefore, action
should not be based on Labor Code but on breach of contract.

Labor Law 1
RIZAL EMPIRE INSURANCE GROUP V NLRC (RUIZ,
CORIA)
150 SCRA 565
PARAS; May 29, 1987

A2010

- 45 -

Disini

punctuality and attendance. Perfection cannot after all be


demanded.
Disposition Petition DISMISSED.

NATURE
Petition for certiorari
FACTS
- August, 1977- Coria was hired by Rizal Empire Insurance
Group(REIG) as a casual employee with a salary of P10.00 a
day.
- January 1, 1978- Coria was made a regular employee, having
been appointed as clerk-typist, with a monthly salary of
P300.00.
- Being a permanent employee, he was furnished a copy of
petitioner company's "General Information, Office Behavior and
Other Rules and Regulations."
- In the same year, without change in his position-designation,
he was transferred to the Claims Department and his salary was
increased to P450.00 a month.
- 1980- he was transferred to the Underwriting Department and
his salary was increased to P580.00 a month plus cost of living
allowance, until he was transferred to the Fire Department as
filing clerk.
- July, 1983- he was made an inspector of the Fire Division with
a monthly salary of P685.00 plus allowances and other benefits.
- Oct. 15, 1983- Coria was dismissed from work, on the grounds
of tardiness and unexcused absences.
- Coria filed a complaint with MOLE
- March 14, 1985- LA Ruiz reinstated him to his position with
back wages.
- REIG appealed to the NLRC but was dismissed on the ground
that the same had been filed out of time. Hence this petition.
ISSUE
WON it is still within the jurisdiction of the SC
HELD
NO
- Under the provisions of the Revised NLRC Rules, the decision
appealed from in this case has become final and executory and
can no longer be subject to appeal.
Ratio
Administrative regulations and policies enacted by
administrative bodies to interpret the law which they are
entrusted to enforce, have the force of law, and are entitled to
great respect (Espanol v. Philippine Veterans Administration,
137 SCRA 314 [1985])..
Reasoning
- The record shows that REIG received a copy of the decision of
the LA on April 1, 1985.
- It filed a Motion for Extension of Time to File Memorandum of
Appeal on April 11, 1985 and filed the Memorandum of Appeal
on April 22, 1985.
- Rule VIII of the Revised Rules of the NLRC on appeal, provides
that decisions or orders of a LA shall be final and executory
unless appealed to the Commission by any or both of the
parties within 10 calendar days from receipt of notice and that
no motion or request for extension of the period within which to
perfect an appeal shall be entertained.
NLRC correctly dismissed REIGs appeal pursuant to said rules.
- The NLRC didnt commit GAD amounting to lack of jurisdiction
in arbitrarily dismissing petitioners' appeal on a technicality.
- SC need not interpret the Revised Rules of the NLRC as they
are clear and explicit and leave no room for interpretation.
- Even on the merits, the ruling of the LA appears to be correct;
the consistent promotions in rank and salary of the private
respondent indicate he must have been a highly efficient
worker, who should be retained despite occasional lapses in

DUNCAN ASSOCIATION OF DETAILMAN-PTGWO V


GLAXO WELLCOME PHILIPPINES INC
438 SCRA 343
TINGA; September 17, 2004
NATURE
Petition for review on certiorari of the decision and resolution of
the Court of Appeals
FACTS
- Petitioner Tecson was hired by respondent Glaxo Wellcome
Philippines, Inc. (Glaxo) as medical representative after Tecson
had undergone training and orientation. Thereafter, Tecson
signed a contract of employment which stipulates, among
others, that he agrees to study and abide by existing company
rules; to disclose to management any existing or future
relationship by consanguinity or affinity with co-employees or
employees of competing drug companies and should
management find that such relationship poses a possible
conflict of interest, to resign from the company.
- The Employee Code of Conduct of Glaxo similarly provides
that an employee is expected to inform management of any
existing or future relationship by consanguinity or affinity with
co-employees or employees of competing drug companies. If
management perceives a conflict of interest or a potential
conflict between such relationship and the employees
employment with the company, the management and the
employee will explore the possibility of a transfer to another
department in a non-counterchecking position or preparation
for employment outside the company after six months.
- Tecson was initially assigned to market Glaxos products in
the Camarines Sur-Camarines Norte sales area. Subsequently,
Tecson entered into a romantic relationship with Bettsy, an
employee of Astra Pharmaceuticals (Astra), a competitor of
Glaxo. Bettsy was Astras Branch Coordinator in Albay. She
supervised the district managers and medical representatives
of her company and prepared marketing strategies for Astra in
that area. Even before they got married, Tecson received
several reminders from his District Manager regarding the
conflict of interest which his relationship with Bettsy might
engender. Still, love prevailed, and Tecson married Bettsy.
- Tecsons superiors informed him that his marriage to Bettsy
gave rise to a conflict of interest. Tecsons superiors reminded
him that he and Bettsy should decide which one of them would
resign from their jobs, although they told him that they wanted
to retain him as much as possible because he was performing
his job well.
- Tecson requested for time to comply with the company policy
against entering into a relationship with an employee of a
competitor company.
He explained that Astra, Bettsys
employer, was planning to merge with Zeneca, another drug
company; and Bettsy was planning to avail of the redundancy
package to be offered by Astra. With Bettsys separation from
her company, the potential conflict of interest would be
eliminated. At the same time, they would be able to avail of the
attractive redundancy package from Astra.
- Tecson again requested for more time resolve the problem.
Tecson applied for a transfer in Glaxos milk division, thinking

Labor Law 1
that since Astra did not have a milk division, the potential
conflict of interest would be eliminated. His application was
denied in view of Glaxos least-movement-possible policy.
Glaxo transferred Tecson to the Butuan City-Surigao CityAgusan del Sur sales area. Tecson asked Glaxo to reconsider its
decision, but his request was denied.
- Tecson sought Glaxos reconsideration regarding his transfer
and brought the matter to Glaxos Grievance Committee. Glaxo,
however, remained firm in its decision and gave Tecson time to
comply with the transfer order. Tecson defied the transfer order
and continued acting as medical representative in the
Camarines Sur-Camarines Norte sales area.
- During the pendency of the grievance proceedings, Tecson
was paid his salary, but was not issued samples of products
which were competing with similar products manufactured by
Astra.
He was also not included in product conferences
regarding such products.
- Because the parties failed to resolve the issue at the
grievance machinery level, they submitted the matter for
voluntary arbitration. Glaxo offered Tecson a separation pay of
one-half month pay for every year of service, or a total of
P50,000.00 but he declined the offer. The National Conciliation
and Mediation Board (NCMB) rendered its Decision declaring as
valid Glaxos policy on relationships between its employees and
persons employed with competitor companies, and affirming
Glaxos right to transfer Tecson to another sales territory.
- Aggrieved, Tecson filed a Petition for Review with the Court of
Appeals assailing the NCMB Decision. The Court of Appeals
promulgated its Decision denying the Petition for Review on the
ground that the NCMB did not err in rendering its Decision. The
appellate court held that Glaxos policy prohibiting its
employees from having personal relationships with employees
of competitor companies is a valid exercise of its management
prerogatives. Tecson filed a Motion for Reconsideration of the
appellate courts Decision, but the motion was denied by the
appellate court.
ISSUES
1. WON the Court of Appeals erred in ruling that Glaxos policy
against its employees marrying employees from competitor
companies is valid, and in not holding that said policy violates
the equal protection clause of the Constitution
2. WON petitioner was constructively dismissed
HELD
1. NO
- Glaxo has a right to guard its trade secrets, manufacturing
formulas, marketing strategies and other confidential programs
and information from competitors, especially so that it and
Astra are rival companies in the highly competitive
pharmaceutical industry.
- The prohibition against personal or marital relationships with
employees of competitor companies upon Glaxos employees is
reasonable under the circumstances because relationships of
that nature might compromise the interests of the company. In
laying down the assailed company policy, Glaxo only aims to
protect its interests against the possibility that a competitor
company will gain access to its secrets and procedures.
- That Glaxo possesses the right to protect its economic
interests cannot be denied. No less than the Constitution
recognizes the right of enterprises to adopt and enforce such a
policy to protect its right to reasonable returns on investments
and to expansion and growth. Indeed, while our laws endeavor
to give life to the constitutional policy on social justice and the
protection of labor, it does not mean that every labor dispute
will be decided in favor of the workers. The law also recognizes
that management has rights which are also entitled to respect
and enforcement in the interest of fair play.
- The challenged company policy does not violate the equal
protection clause of the Constitution as petitioners erroneously
suggest. It is a settled principle that the commands of the equal
protection clause are addressed only to the state or those

A2010

- 46 -

Disini

acting under color of its authority. Corollarily, it has been held in


a long array of U.S. Supreme Court decisions that the equal
protection clause erects no shield against merely private
conduct, however, discriminatory or wrongful. The only
exception occurs when the state in any of its manifestations or
actions has been found to have become entwined or involved in
the wrongful private conduct. Obviously, however, the
exception is not present in this case.
Significantly, the
company actually enforced the policy after repeated requests to
the employee to comply with the policy. Indeed, the application
of the policy was made in an impartial and even-handed
manner, with due regard for the lot of the employee.
- In any event, from the wordings of the contractual provision
and the policy in its employee handbook, it is clear that Glaxo
does not impose an absolute prohibition against relationships
between its employees and those of competitor companies. Its
employees are free to cultivate relationships with and marry
persons of their own choosing. What the company merely
seeks to avoid is a conflict of interest between the employee
and the company that may arise out of such relationships.
- The Court of Appeals also correctly noted that the assailed
company policy which forms part of respondents Employee
Code of Conduct and of its contracts with its employees, such
as that signed by Tecson, was made known to him prior to his
employment. Tecson, therefore, was aware of that restriction
when he signed his employment contract and when he entered
into a relationship with Bettsy. Since Tecson knowingly and
voluntarily entered into a contract of employment with Glaxo,
the stipulations therein have the force of law between them
and, thus, should be complied with in good faith. He is
therefore estopped from questioning said policy.
2. NO
- Constructive dismissal is defined as a quitting, an involuntary
resignation resorted to when continued employment becomes
impossible, unreasonable, or unlikely; when there is a demotion
in rank or diminution in pay; or when a clear discrimination,
insensibility or disdain by an employer becomes unbearable to
the employee. None of these conditions are present in the
instant case. The record does not show that Tecson was
demoted or unduly discriminated upon by reason of such
transfer. Glaxo properly exercised its management prerogative
in reassigning Tecson to the Butuan City sales area. When the
problem could not be resolved after several years of waiting,
Glaxo was constrained to reassign Tecson to a sales area
different from that handled by his wife for Astra. Notably, Glaxo
did not terminate Tecson from employment but only reassigned
him to another area where his home province, Agusan del Sur,
was included.
In effecting Tecsons transfer, Glaxo even
considered the welfare of Tecsons family.
Clearly, the
foregoing dispels any suspicion of unfairness and bad faith on
the part of Glaxo.

SALINAS V NLRC (ATLANTIC GULF AND PACIFIC CO)


319 SCRA 54
PURISIMA; November 24, 1999
FACTS
- Petitioners were employed with Atlantic Gulf and Pacific Co.
(AG & P):
Salinas: 1983-1988 as carpenter/finishing carpenter
Alejandro: 1982-1989 as bulk cement operator, bulk cement
plant/carrier operator & crane driver
Cortez: 1979-1988 as carpenter/forklift operator

Labor Law 1
Samulde: 1982-1989 as lubeman/stationary operator
- Complaints (separate but consolidated by the LA): illegal
dismissal
- Petitioners Claim: They had been covered by a number of
contracts renewed continuously, with periods ranging from five
(5) to nine (9) years, and they performed the same kind of work
through out their employment, and such was usually necessary
and desirable in the trade or business of the respondent
corporation; and their work did not end on a project-to-project
basis, although the contrary was made to appear by the
employer through the signing of separate employment
contracts.
- LA: Dismissed petitions on the ground that the petitioners are
project employees are project employees whose work contracts
with AG & P indicate that they were employed in such category;
that they have been assigned to different work projects, not just
to one and that their work relation with AG & P, relative to
termination, is governed by Policy Instruction No. 20 (rule
governing project employees).
- Appeal to NLRC: Affirmed LAs findings
ISSUES
1. WON the petitioners are project employees
Procedural
2. WON this petition for certiorari was proper
HELD
1. NO
- The petitioners are regular employees.
- The mandate in Article 281 of the Labor Code, which
pertinently prescribes that the 'provisions of written agreement
to the contrary notwithstanding and regardless of the oral
agreements of the parties, an employment shall be deemed to
be regular where the employee has been engaged to perform
activities which are usually necessary or desirable in the usual
business or trade of the employer' and that any employee who
has rendered at least one year of service, whether such service
is continuous or broken shall be considered a regular employee
with respect to the activity in which he is employed and his
employment shall continue while such actually exists,' should
apply in the case of petitioner.
- Failure to report the termination to Public Employment Office
is a clear indication that petitioners were not and are not
project employees. (PI No. 20 requires reports of terminations)
- It is basic and irrefragable rule that in carrying out and
interpreting the provisions of the Labor Code and its
implementing regulations, the workingman's welfare should be
the primordial and paramount consideration. The interpretation
herein made gives meaning and substance to the liberal and
compassionate spirit of the law enunciated in Article 4 of Labor
Code that "all doubts in the implementation and interpretation
of the provisions of the Labor Code including its implementing
rules and regulations shall be resolved in favor of labor".
- It is beyond cavil that petitioners had been providing the
respondent corporation with continuous and uninterrupted
services, except for a day or so gap in their successive
employment contracts. Their contracts had been renewed
several times, with the total length of their services ranging
from five (5) to nine (9) years. Throughout the duration of their
contracts, they had been performing the same kinds of work
(e.g., as lubeman, bulk cement operator and carpenter), which
were usually necessary and desirable in the construction
business of AG & P, its usual trade or business.
- Undoubtedly, periods in the present case have been imposed
to preclude the acquisition of tenurial security by petitioners,
and must be struck down for being contrary to public policy,
morals, good customs or public order.
2. YES
- Anent the issue that the petition should have been brought
under Rule 65 and not under Rule 45 of the Revised Rules of
Court, this rule is not inflexible. In the interest of justice, often
the Court has judiciously treated as special civil actions for

A2010

- 47 -

Disini

certiorari petitions erroneously captioned as petitions for review


on certiorari.
- With regard to the issue on non-exhaustion of administrative
remedies, the Court hold that the failure of petitioners to
interpose a motion for reconsideration of the NLRC decision
before coming to this Court was not a fatal omission. The
exhaustion of administrative remedies doctrine is not a hard
and fast rule and does not apply where the issue is purely a
legal one. A motion for reconsideration as a prerequisite for the
bringing of an action under Rule 65 may be dispensed with
where the issue is purely of law, as in this case. At all events
and in the interest of substantial justice, especially in cases
involving the rights of workers, procedural lapses, if any, may
be disregarded to enable the Court to examine and resolve the
conflicting rights and responsibilities of the parties. This
liberality is warranted in the case at bar, especially since it has
been shown that the intervention of the Court is necessary for
the protection of the herein petitioner(s).
Disposition The questioned Resolution of the NLRC is SET
ASIDE and another one is hereby ENTERED ordering the
respondent corporation to reinstate petitioners without loss of
seniority and with full backwages.

ABELLA V NLRC (QUITCO, DIONELE)


152 SCRA 140
PARAS; July 20, 1987
FACTS
- Petitioner Rosalina Perez Abella leased a farm land in
Ponteverde, Negros Occidental, known as Hacienda DanaoRamona, for a period of ten (10) years, renewable, at her
option, for another ten (10) years. She did renew for another
ten years. During the existence of the lease, she employed
private respondents. Private respondent Ricardo Dionele, Sr.
has been a regular farm worker for 33 years while . On the
other hand, private respondent Romeo Quitco started worked
for 14 years. Upon the expiration of her leasehold rights,
petitioner dismissed private respondents and turned over the
hacienda to the owners thereof, who continued the
management, cultivation and operation of the farm
ISSUE
WON private respondents are entitled to separation pay
HELD
YES
- The closing wasnt due to serious losses or financial reverses.
The Court cited Article 284 (this should be 283) which says:
"Art. 284.
Closure of establishment and reduction of
personnel. The employer may also terminate the
employment of any employee due to the installation of laborsaving devices, redundancy, retrenchment to prevent losses
or the closing or cessation of operation of the establishment
or undertaking unless the closing is for the purpose of
circumventing the provisions of this title, by serving a written
notice on the workers and the Ministry of Labor and
Employment at least one (1) month before the intended date
thereof. In case of termination due to the installation of laborsaving devices or redundancy, the worker affected thereby
shall be entitled to a separation pay equivalent to at least his
one (1) month pay or to at least one (1) month pay for every
year of service, whichever is higher. In case of retrenchment
to prevent losses and in cases of closure or cessation of
operations of establishment or undertaking not due to serious
business losses or financial reverses, the separation pay shall
be equivalent to one (1) month pay or at least one-half (1/2)
month pay for every year of service whichever is higher. A
fraction of at least six (6) months shall be considered one (1)
whole year."
- Petitioner then contends that the aforequoted provision
violates the constitutional guarantee against impairment of

Labor Law 1

A2010

- 48 -

Disini

obligations and contracts, because when she leased Hacienda


Danao-Ramona, neither she nor the lessor contemplated the
creation of the obligation to pay separation pay to workers at
the end of the lease. The Court said that this contention by
petitioner is untenable. The law is clear and to permit such an
argument would mean that the years of service given by the
workers will mean nothing since there is no agreement here
that the new management will be the one to shoulder the
separation pay. The old management, pertaining to Abella in
this case, should give the payment.
- In any event, it is well-settled that in the implementation and
interpretation of the provisions of the Labor Code and its
implementing regulations, the workingman's welfare should be
the primordial and paramount consideration. (Volshel Labor
Union v. Bureau of Labor Relations, 137 SCRA 43 [1985]). It is
the kind of interpretation which gives meaning and substance
to the liberal and compassionate spirit of the law as provided
for in Article 4 of the New Labor Code which states that `all
doubts in the implementation and interpretation of the
provisions of this Code including its implementing rules and
regulations shall be resolved in favor of labor." The policy is to
extend the applicability of the decree to a greater number of
employees who can avail of the benefits under the law, which is
in consonance with the avowed policy of the State to give
maximum aid and protection to labor.
Disposition Petition is DISMISSED.

- Subject of interpretation in the case at bar is Article 943 of the


Labor which was amended by Executive Order No. 2034
- In deciding in favor of the Bisig ng Asian Transmission Labor
Union (BATLU), the Voluntary Arbitrator held that Article 94 of
the Labor Code provides for holiday pay for every regular
holiday, the computation of which is determined by a legal
formula which is not changed by the fact that there are two
holidays falling on one day; and that that the law, as amended,
enumerates ten regular holidays for every year, and should not
be interpreted as authorizing a reduction to nine the number of
paid regular holidays "just because April 9 (Araw ng Kagitingan)
in certain years, like 1993 and 1998, is also Holy Friday or
Maundy Thursday."
- The Court of Appeals upheld the findings of the Voluntary
Arbitrator, holding that the Collective Bargaining Agreement
(CBA) between petitioner and BATLU, the law governing the
relations between them, clearly recognizes their intent to
consider Araw ng Kagitingan and Maundy Thursday, on
whatever date they may fall in any calendar year, as paid legal
holidays during the effectivity of the CBA and that "there is no
condition, qualification or exception for any variance from the
clear intent that all holidays shall be compensated.
- The Court of Appeals further held that "in the absence of an
explicit provision in law which provides for [a] reduction of
holiday pay if two holidays happen to fall on the same day, any
doubt in the interpretation and implementation of the
Labor Code provisions on holiday pay must be resolved
in favor of labor."
- Hence, this petition.

ASIAN TRANSMISSION CORP V CA (BISIG NG ASIAN


TRANSMISSION LABOR UNION)
425 SCRA 478
CARPIO-MORALES; March 15, 2004

ISSUE
WON daily-paid employees are entitled to be paid for two
regular holidays which fall on the same day

NATURE
Petition for certiorari seeking the nullification of the March 28,
2000 Decision of the Court of Appeals
FACTS
- The Department of Labor and Employment (DOLE), through
Undersecretary Cresenciano B. Trajano, issued an Explanatory
Bulletin dated March 11, 1993, wherein it clarified, that
employees are entitled to 200% of their basic wage on April 9,
1993, which, apart from being Good Friday, and, therefore, a
legal holiday, is also Araw ng Kagitingan, which is also a legal
holiday, even if unworked.
- Said bulletin was reproduced on January 23, 1998, when April
9, 1998 was both Maundy Thursday and Araw ng Kagitingan
- Despite the explanatory bulletin, petitioner Asian Transmission
Corporation opted to pay its daily paid employees only 100% of
their basic pay on April 9, 1998.
- Respondent Bisig ng Asian Transmission Labor Union (BATLU)
protested.
- In accordance with Step 6 of the grievance procedure of the
Collective Bargaining Agreement (CBA) existing between
petitioner and BATLU, the controversy was submitted for
voluntary arbitration.
- On July 31, 1998, the Office of the Voluntary Arbitrator
rendered a decision directing petitioner to pay its covered
employees "200% and not just 100% of their regular daily
wages for the unworked April 9, 1998

HELD
YES
- Holiday pay is a legislated benefit enacted as part of the
Constitutional imperative that the State shall afford protection
to labor. Its purpose is not merely "to prevent diminution of the
monthly income of the workers on account of work
interruptions. In other words, although the worker is forced to
take a rest, he earns what he should earn, that is, his holiday
pay."8 It is also intended to enable the worker to participate in
the national celebrations held during the days identified as with
great historical and cultural significance.
- Independence Day (June 12), Araw ng Kagitingan (April 9),
National Heroes Day (last Sunday of August), Bonifacio Day
(November 30) and Rizal Day (December 30) were declared
national holidays to afford Filipinos with a recurring opportunity
to commemorate the heroism of the Filipino people, promote
national identity, and deepen the spirit of patriotism. Labor Day
(May 1) is a day traditionally reserved to celebrate the
contributions of the working class to the development of the
nation, while the religious holidays designated in Executive
Order No. 203 allow the worker to celebrate his faith with his
family.
- As reflected above, Art. 94 of the Labor Code, as amended,
affords a worker the enjoyment of ten paid regular holidays.
The provision is mandatory, regardless of whether an employee
is paid on a monthly or daily basis. Unlike a bonus, which is a
3

ART. 94. Right to holiday pay. (a) Every worker shall be paid his regular daily
wage during regular holidays, except in retail and service establishments regularly
employing less than ten (10) workers; (b) The employer may require an employee to
work on any holiday but such employee shall be paid a compensation equivalent to
twice his regular rate; and (c) As used in this Article, "holiday" includes: New Years
Day, Maundy Thursday, Good Friday, the ninth of April, the first of May, the twelfth of
June, the fourth of July, the thirtieth of November, the twenty-fifth and thirtieth of
December and the day designated by law for holding a general election
4

regular holidays are now:1. New Years Day January 1; 2. Maundy Thursday Movable
Date; 3. Good Friday Movable Date; 4. Araw ng Kagitingan April 9 (Bataan and
Corregidor Day); 5. Labor Day May 1; 6. Independence Day June 12; 7. National
Heroes Day Last Sunday of August; 8. Bonifacio Day November 30; 9. Christmas Day
December 25; 10. Rizal Day December 30

Labor Law 1
management prerogative, holiday pay is a statutory benefit
demandable under the law. Since a worker is entitled to the
enjoyment of ten paid regular holidays, the fact that two
holidays fall on the same date should not operate to
reduce to nine the ten holiday pay benefits a worker is
entitled to receive.
- It is elementary, under the rules of statutory construction, that
when the language of the law is clear and unequivocal,
the law must be taken to mean exactly what it says. In
the case at bar, there is nothing in the law which
provides or indicates that the entitlement to ten days of
holiday pay shall be reduced to nine when two holidays
fall on the same day.
- In any event, Art. 4 of the Labor Code provides that all
doubts in the implementation and interpretation of its
provisions, including its implementing rules and
regulations, shall be resolved in favor of labor. For the
working mans welfare should be the primordial and
paramount consideration.
- Moreover, Sec. 11, Rule IV, Book III of the Omnibus Rules to
Implement the Labor Code provides that "Nothing in the law or
the rules shall justify an employer in withdrawing or reducing
any benefits, supplements or payments for unworked regular
holidays as provided in existing individual or collective
agreement or employer practice or policy.
- From the pertinent provisions of the CBA entered into by the
parties, petitioner had obligated itself to pay for the legal
holidays as required by law.
Disposition Petition is dismissed.

CLEMENTE V GSIS
152 SCRA 500
GUTIERREZ, JR; July 31, 1987
NATURE
Petition to review decision of the Employees Compensation
Commission (ECC) which affirmed decision of GSIS and denied
Clementes claim for death benefits
FACTS
- Carolinas husband, Pedro Clemente was for 10 years a janitor
in the DOH Dagupan City assigned at the Ilocos Norte Skin
Clinic.
- He was hospitalized for 12 days due to his ailment of nephritis,
and was also found to be suffering from Hansens Disease
(portal cirrhosis and leprosy). He died on Nov 14, 1976.
- Petitioner then filed with GSIS a claim for employees
compensation under the Labor Code. This was denied by GSIS
on the ground that such ailments are not occupational diseases
taking into consideration nature of his work. Under Art. 167(L)
of the Labor Code and Sec. 1(b) Rule III of the Amended Rules
on Employees Compensation, for the sickness and the resulting
disability or death to be compensable, sickness must be the
result of an occupational disease listed under Annex A of the
rules; otherwise proof must be shown that the risk of
contracting the disease is increased by the working conditions.
- Petitioner claimed that the ailments were contracted in the
course of employment and were aggravated by his work since
he was in direct contact with persons suffering from different
skin diseases and was exposed to obnoxious dusts and other
dirt.
- ECC also dismissed the claim since there was no substantial
evidence of causal connection and there was evidence that
deceased had already contracted the Hansens before
employment.
ISSUE
WON petitioner is entitled to the compensation
HELD
YES

A2010

Disini

- 49 -

Ratio Strict rules of evidence are NOT applicable in claims for


compensation. The degree of proof required is merely
substantial evidence, which means such relevant evidence as a
reasonable mind might accept as adequate to support a
conclusion. What the law requires is a reasonable workconnection and not a direct causal relation.
- Doubts should be resolved in favor of the claimant-employee.
Reasoning
- The major ailments of the deceased could be traced to
bacterial and viral infections. For instance, in the case of
leprosy, it is known that the source of infection is discharge
from lesions of persons with active cases.
- Petitoners husband worked in a skin clinic and was exposed to
different carriers of diseases. As janitor, he was the employee
most exposed to the dangerous concentration of infected
material, and not being a med practitioner, least likely to know
how to avoid them.
- GSISs conservative stand is not consistent with the liberal
interpretation of the Labor Code and the social justice
guarantee embodied in the Constitution in favor of workers.
Disposition Decision appealed from is set aside and GSIS is
ordered to pay petitioner P12T as death benefits and P1,200 as
attorneys fees.

ACUAV CA
[PAGE 12]
BONIFACIO V GSIS
146 SCRA 276
FERNAN; December 15, 1986
NATURE
Petition for review on certiorari
FACTS
- Lourdes Bonifacio was a classroom teacher assigned to the
district of Bagamanoc, Division of Catanduanes, from August
1965 until she contracted carcinoma of the breast with
metastases to the gastrointestinal tract and lungs which caused
her death on Oct. 5, 1978.
- Thereafter a claim for death benefits under P.D. No. 626, as
amended, was filed by petitioner with the GSIS. The same was
however denied on the ground that the decedent's principal
ailment, carcinoma of the breast with metastases to
gastrointestinal tract and lungs, is not an occupational disease
for her particular work as a teacher, nor is the risk of
contracting said disease increased by her working conditions.
- The Employees Compensation Commission (ECC), on appeal
affirmed the decision of the GSIS.
ISSUES
1. WON the GSIS and the ECC erred in denying petitioners
claim
2. WON the rule that in case of doubt in the implementation
and interpretation of the provisions of the Labor Code, including
its implementing rules and regulations, the same shall be
resolved in favor of the laborer applies in this case
HELD
1. NO
- A compensable sickness means "any illness definitely
accepted as an occupational disease listed by the ECC, or any
illness caused by employment subject to proof by the employee
that the risk of contracting the same is increased by working
conditions. For this purpose, the Commission is empowered to
determine and approve occupational diseases and work-related
illnesses that may be considered compensable based on
peculiar hazards of employment." [Art. 167(1) Labor Code as
amended by P.D. No. 1368, effective May 1, 1978].

Labor Law 1
- Thus, for the sickness or the resulting disability or death to be
compensable, the sickness must be the result of an accepted
occupational disease listed by the ECC, or any other sickness
caused by employment subject to proof by claimant that the
risk of contracting the same is increased by working conditions.
[Sec. 1, Rule 11, Amended Rules on Employees Compensation].
Carcinoma of the breast with metastases to the gastrointestinal
tract and lungs is not listed by the Commission as an
occupational disease.
- The cancer which affected the deceased not being
occupational in her particular employment, it became
incumbent upon petitioner to prove that the decedent's working
conditions increased the risk of her contracting the fatal illness.
This onus petitioner failed to satisfactorily discharge.
- Petitioner's contention that the decision of the ECC totally
ignored the SC's pronouncements on compensation cases is
unmeritorious. The petitioner evidently overlooked that his
claim is now within the ambit of the Labor Code and the rulings
under the old law, Act No. 3428, as amended, no longer control.
- The old law as embodied particularly in Section 43 of RA No.
772 amending Act No. 3812, provided for "the presumption of
compensability and the rule on aggravation of illness, which
favor the employee," and "paved the way for the latitudinarian
or expansive application of the Workmen's Compensation Law
in favor of the employee or worker." The presumption in
essence states that in any proceeding for the enforcement of
the claim for compensation under the Workmen's Compensation
Act "it shall be presumed in the absence of substantial evidence
to the contrary that the claim comes within the provisions of the
said Act, that sufficient notice thereof was given, that the injury
was not occasioned by the willful intention of the injured
employee to bring about the injury or death of himself or of
another, that the injury did not result solely from the
intoxicatiojn of the injured employee while on duty, and that the
contents of verified medical and surgical reports introduced in
evidence by claimants for compensation are correct."
- Thus, under the Workmen's Compensation Law, it is not
necessary for the claimant to carry the burden of proof to
establish his case to the point of demonstration It is not
necessary to prove that employment was the sole cause of the
death or injury suffered by the employee. It is sufficient to show
that the employment had contributed to the aggravation or
acceleration of such death or ailment. Once the disease had
been shown to have arisen in the course of employment, it is
presumed by law, in the absence of substantial evidence to the
contrary, that it arose out of it.
- With this legal presumption in the old law, the burden of proof
shifts to the employer and the employee no longer suffers the
burden of showing causation. Under the present Labor Code,
the "latitudinarian or expansive application of the Workmen's
Compensation Law in favor of the employee or worker" no
longer prevails as the burden of showing proof of causation has
shifted back to the employee particularly in cases of sickness or
injuries which are not accepted or listed as occupational by the
ECC. The Labor Code abolished the presumption of
compensability and the rule on aggravation of illness caused by
the nature of the employment.
2. NO
- While the court does not dispute petitioner's contention that
under the law, in case of doubt in the implementation and
interpretation of the provisions of the Labor Code, including its
implementing rules and regulations, the doubt shall be resolved
in favor of the laborer, the court finds that the same has no
application in this case since the pertinent provisions of the
Labor Code leave no room for doubt either in their
interpretation or application.
Disposition Petition is dismissed and the decisions of the GSIS
and the ECC denying the claim are affirmed

BRAVO V EMPLOYEES' COMPENSATION


COMMISSION

A2010

Disini

- 50 143 SCRA 101


FERNAN; July 22, 1986

NATURE
PETITION for certiorari to review the decision of the Employees'
Compensation Commission
FACTS
- Evelio Bravo was a supervising cartographer engineer at the
Bureau of Coast and Geodetic Survey. As litho-photo engraving
supervisor (another term for a supervising cartographer
engineer?), he was involved in drafting and plate printing,
developing and processing either dry or wet negatives, and
supervising the formulation of lightsensitive lithographic
chemicals from reagent of nitric, phosphoric, oleic acids,
potassium ferricynamide, ammonium hydroxide and ammonium
dichromate in the kithographic laboratory.
- sometime in 1979, he complained of irregular bowel
movement, constipation and abdominal pain. In 1980 he was
admitted to St. Lukes Hospital and was diagnosed with
"adenocarcinoma sigmoid (colon) Duke's C and chronic
periappendicitis". He went through a series of operations and
incurred hospitalization expenses amounting to P8,650.05.
- He did not return to work and retired at the age of 45 under
the provisions of RA 1616. He received P37,002.31 from GSIS.
He filed a claim for disability benefits in the GSIS.
- GSIS: Denied. His diagnosed disease were not occupational
diseases in his particular employment and his working
conditions did not increase the risk of contracting them.
-He sought reconsideration, claiming that his work exposed him
to chemicals. His MFR was denied on the ground that his
exposure to photographic solutions as litho-photo
engraving supervisor had no causal relationship to the
development of his adenocarcinoma considering that
said ailment is traceable to "familial multiple polyposis,
chronic ulcerative colitis, chronic lympho-granuloma
venereum, chronic granuloma inguinale and perhaps
adenoma.
- He appealed to the Employees Compensation Commission,
but died pending the appeal. His widow, Angeles, pursued his
appeal.
- Commission: affirmed GSIS deci. Bravo's ailments were
"too remote to be related causally to his work and
working conditions" at the Bureau of Coast and Geodetic
Survey. His contention that his cancer could be traced to
exposure to photographic solutions was merely
supposition and devoid of medical support.
- Petitioners contention
> while the causes of colonic malignancy are as yet
undetermined, there is a "probability" that the fatal ailment of
Bravo was work connected as shown by the fact that he was
exposed to various chemicals which are generally considered
predisposing factors of cancer (relying on the decision in
Panotes vs. Employees' Compensation Commission where
it was held that the very fact that the cause of a disease is
unknown creates the probability that the working conditions
could have increased the risk of contracting the disease, if not
caused by it) ; that the law merely requires reasonable workconnection because of the liberal interpretation accorded to
social legislation; that under the theory of increased risk, her
husbands cancer of the colon is a compensable disease
because his exposure to chemicals and the "stressful demand"
of his work increased the risk of contracting said ailment; and
that Commission issued the Resolutions Nos. 2610 and 26775
5

Resolution No. 2610 approves the recommendation of the Commission's Technical


Committee on Medical Matters that appealed compensation cases "whose subject
contingencies concern cancer diseases shall be held compensable, in line with
pertinent Supreme Court Decisions, provided that such diseases shall be duly
confirmed by formal reports on biopsies, or opinions of cancer specialists". That
resolution shall be applied prospectively.

Labor Law 1
which
provides
guidelines
for
deciding
on
pending
compensation cases regarding cancer.
- Solicitor Generals reply
> resolutions are just proofs that the Commission is
continuously in involved in its task "to initiate, rationalize, and
coordinate policies of the employees' compensation program."
They do not imply that the law merely requires reasonable
work-connection because that requirement which was
mandated in the repealed Workmen's Compensation Act is
different from the present requirement of clear medical basis
"where before a mere aggravation or presumption of
compensability was sufficient."
ISSUES
WON cancer of the colon and peri-appendicitis which caused
the death of a former litho-photo engraving supervisor are
compensable diseases under the Labor Code
a. WON cancer of the colon and peri-appendicitis are listed
under compensable diseases under the Labor Code and Rule III,
Section IV of the Amended Rules on Employees Compensation
b. WON petitioner could claim benefits through the increased
risk doctrine
HELD
NO
Ratio Article 167, paragraph (1) of the Labor Code and Rule III,
Section IN of the Amended Rules on Employees' Compensation
provide that for a sickness and the resulting disability or death
to be compensable, the said sickness must be an occupational
disease listed under Annex "A" of said Rules, otherwise, the
claimant or employee concerned must prove that the risk of
contracting the disease is increased by the working conditions
(increased risk doctrine)
a. NO
- Both cancer of the colon and peri-appendicitis are not listed as
occupational diseases for Bravo's kind of employment.
b. NO
- Petitioner failed to submit convincing proofs to entitle her to
compensation benefits.
Ratio A claimant who depends on the theory of increased risk
must present substantial proof to show that his ailment was
contracted during his employment. He or she must also submit
proof that the risk of contracting the ailment was increased by
the particular working conditions.
Reasoning
- On reliance on Panotes case: In the Panotes case and the
Cristobal case, both claimants presented conditions of their
employment. In the present case, the petitioner only
enumerated the chemicals to which Bravo was allegedly
exposed as a litho-photo engraving supervisor and rely
on the "probability" that those chemicals caused his
cancer of the colon.
On interpretation in compensation cases
> Strict rules of evidence are not applied in compensation
cases. However, the present scheme and theory of employees'
compensation under the Labor Code requires a clear medical
basis for a claim for benefits to succeed. There are no more
presumptions as to what caused a particular illness because the
determination of compensability is medically and scientifically
oriented.

Resolution No. 2677 amends Resolution No. 2610 by adding to the pertinent
paragraph thereof the phrase "provided that certain predisposing factors that are
medically recognized or proven are present." It also approves the modified guidelines
on cancer of the breast, liver stomach (gastric), lungs and nasopharynx. As regards
"other types of cancer diseases", the guideline states: "An employee's prolonged
exposure to chemicals may predispose him or her to contract and develop other
types of cancer diseases". For cancer cases decided by the Supreme Court, the
guidelines states: "A claim must be resolved in favor of a claimant or appellant if
facts of his or her case on record indicate reasonable work-connection of the disease,
the disease belongs to borderline or 'twilight' cases, and if the cause of the cancer
disease is unknown".

A2010

- 51 -

Disini

On application of the resolutions by the Commission


> they were issued after the death of Evelio, and are applied
prospectively. Even if they were applied, the petitioner did not
submit formal requirements required by said resolutions.
On liberal interpretation due to social legislation
> We are aware of the mandate that social legislation should
be applied in consonance with the principles of social justice
and protection to labor. However, we cannot adopt a sweeping
interpretation of the law in favor of labor lest we engage in
judicial legislation.
Disposition decision of the Employees' Compensation
Commission is hereby affirmed.

PAL V NLRC (IRINEO)


201 SRCA 687
NARVASA; September 24, 1991
FACTS
- On the basis of the findings and recommendations of a Fact
Finding Panel upon investigation, Irineo and 3 other PAL
employees, Damian, Rabasco and Macatol, were prosecuted
and convicted for estafa through falsification of commercial
documents
- All 4 filed motions for reconsideration and/or new trial. Only
one of them, Macatol, was absolved for lack of sufficient
evidence. 12 years later, Macatol filed a complaint for illegal
dismissal which was dismissed by the Labor Arbiter on the
ground of prescription. The NLRC affirmed, contending that the
prescriptive period accrued from the time of his dismissal and
not the termination of the criminal case
- A later appeal with the IAC resulted in the acquittal of Irineo
and Rabasco on grounds of reasonable doubt.
- 17 years after his dismissal, Irineo filed a complaint against
PAL for reinstatement and backwages, claiming the termination
was illegal. The Labor Arbiters decision decreed his
reinstatement without loss of seniority rights, payment of
backwages and moral damages of P300k. The Arbiter overruled
the defense of prescription and held that since there was a PAL
circular which placed any employee charged with any crime
inimical to the companys interest under preventive suspension,
and a standing order by the CIR forbidding the dismissal of any
PAL employee without court authority, Irineos dismissal merely
amounted to suspension. The Arbiter rendered a judgment
terminating Irineos suspension with backwages and moral
damages.
- PAL appealed to the NLRC but failed to obtain a reversal of the
Arbiters decision. Hence this petition for certiorari.
ISSUE
WON the NLRC gravely abused discretion amounting to lack or
excess of jurisdiction and arbitrarily exercised power without
due regard for the rule of law
HELD
YES
- That there should be care and solicitude in the protection and
vindication of the rights of workingmen cannot be gainsaid; but
that care and solicitude cannot justify disregard of relevant
facts or eschewal of rationality in the construction of the text of
applicable rules in order to arrive at a disposition in favor of an
employee who is perceived as otherwise deserving of sympathy
and commiseration.
- The letter to Irineo from then PAL president Benigno Toda
clearly indicated, For being involved in the irregular refund of
tickets in the international service to the damage and prejudice
of the company, you are dismissed from the service effective
immediately. For the Arbiter and the NLRC to construe this as
mere suspension would be illogical.
-Their reliance on PAL circular to justify their decision,
construing this as a complete foreclosure of any alternative

Labor Law 1
action on PALs part was unfounded. To further support their
decision they refer said CIR standing order which had been
imposed in relation to a pending labor dispute with the CIR.
However, having ended when the parties entered into a CBA 2
years before Irineos dismissal, the standing order was no
longer relevant to the event.
- Irineos assertion only after 17 years meant he slept on his
rightshis claim is thus time-barred.
- Premises considered, it appears that the NLRCs conclusions
are flawed by errors serious as to constitute grave abuse of
discretion
Disposition Court GRANTS the petition and issues the writ of
certiorari prayed for.

MANNING INTERNATIONAL CORPORATION V NLRC


(BENEDICTO)
195 SCRA 155
NARVASA; March 13, 1991
NATURE
Petition for certiorari to review
FACTS
- Francisco Benedictoa.k.a. Lazaro Benedicto, according to his
passportwas hired by a foreign firm, Abdulasis & Mohamed A.
Aljomaih Co., thru its Philippine representative. Manning
International Corporation, as a truck driver for its establishment
in Riyadh, Saudi Arabia.
Benedicto was engaged for a
stipulated term of two (2) years. He left for Saudi Arabia on
December 1, 1980 to fulfil his employment contract.
- Some months before the expiration of his contract with
Abdulasis, etc., Benedicto was involved in a vehicular accident,
was injured, and in consequence, lost both his legs. From the
date of the accident, February 2, 1982, he was confined at a
hospital in Saudi Arabia until sometime when his employment
was terminated. He was repatriated to the Philippines in August
1982.
- Benedicto filed a complaint with POEA for the recovery of his
salary for the unexpired portion of his contract, insurance
benefits and projected cost of medical expenses amounting to
P25,000.00
- POEA dismissed Benedictos claim upon finding that
complainant was legally terminated from employment
because of his disability. However, Manning and Abdulasis
were ordered to provide compensation benefits for serviceconnected illness, injuries or death.
- MFR to NLRC affirmed the decision of POEA
- Judgment became final and executory. Benedicto moved for
computation of the amounts due him, and in substantiation,
submitted receipts evidencing his actual medical expenses. His
former employers opposed the motion on the ground that the
medical expenses referred to another person, Lazaro Benedicto
but the Administrator overruled the objection and pointed out
that the names Lazaro and Francisco Benedicto both referred to
one person, and directed the issuance of an alias writ of
execution to enforce payment of P12,000 as total and
permanent disability benefits and P19,450.00 as hospitalization
and medical expenses for 120 days or a total of P31,450.00.
- MFR was filed to NLRC to protest the limitation of the award to
him of medical expenses to a period of 120 days.
- NLRC set aside the the POEA Order and on considerations of
equity and social justice as well as the theory of medical
treatment should not be stopped until Benedictos injury or

A2010

- 52 -

Disini

disability is healed and entered a new judgment increasing the


amount to be paid by employers.
- Petition for certiorari
ISSUES
1. WON the new judgment of the NLRC is void ab initio,
insofar as it attempts to vary the disposition of the final and
executory decision of the POE Administrator
2. WON the challenged decision of NLRC is without legal basis
and unjust
HELD
1. YES
Ratio When a final judgment becomes executory, it thereby
becomes immutable and unalterable, The judgment may no
longer be modified in any respect, even if the modification is
meant to correct what is perceived to be an erroneous
conclusion of fact or law, and regardless of whether the
modifications is attempted to be made by the Court rendering it
or by the highest Court of the land. The only recognized
exceptions are the correction of clerical errors or the making of
so-called nunc pro tunc entries which cause no prejudice to any
party, and, of course, where the judgment is void.
2. YES
Ratio Considerations of equity and social justice cannot
prevail over against the expressed provision of the labor laws
allowing dismissal of employees for cause and without any
provision for separation pay.
Disposition Contested Decision ANNULLED AND SET ASIDE
and REINSTATING and AFFIRMING the Order of the POE
Administrator.

RELIANCE & INSURANCE CO INC V NLRC (RELIANCE


SURETY & INSURANCE EMPLOYEES UNION)
193 SCRA 365
SARMIENTO; January 25 1991
NATURE
Petition for certiorari whether or not strikers have been found to
have staged an illegal strike may be reinstated to work.
FACTS
- The manager of Reliance Surety Insurance Co (RSIC) effected
a change in the seating arrangement of its personnel to avoid
productivity loss due to personal and non-work-related
conversations, calls and visits.
- Isagani Rubio, Rosalinda Macapagal, Glene Molina, and Severa
Cansino protested the transfer of their tables because said
change was without prior notice and was just to harass them as
union members. When the manager insisted, a heated
discussion ensued, during which Rubio and companions insulted
the manager and supervisors.
- The 4 employees were asked to explain within 48 hours why
no disciplinary action should be taken against them for
misconduct, insubordination, and gross disrespect. Tension rose
in the office as Rubio continued to refuse to stay at his
designated place, and Molina and Macapagal still levelled
insults to those who testified against them. Hence, Rubio and
companions were placed under preventive suspension on 3
February 1987 and ultimately dismissed after investigation on 3
March 1987.
- 6 March 1987, the Reliance Surety & Insurance Employees
Union (union) filed in behalf of the dismissed employees with
the NLRC, against the RSIC a complaint for illegal dismissal
including the charge of unfair labor practice.

Labor Law 1

A2010

- 53 -

Disini

- Unions claim: The company was guilty of unfair labor practice


because it effected transfer and changes in the seating
arrangement to pressure or intimidate union members; because
it interfered in the union members' exercise of their right to
self-organization by forcing them to undertake overtime work
even on a non-working Saturday and in times when there were
scheduled union meetings to prevent them from attending the
same; and because, thru its manager and assistant managers,
it caused the resignation and withdrawal of union members
from the union.
- Pending trial, the union filed with the DOLE a notice of strike
predicated on unfair labor practices by the company. RSIC was
given notice of strike and a telegram from DOLE for initial
conciliation conference both to be held on the same date. But
even before the initial conference could take place, the union in
the morning of 17 March 1987 struck and picketed the company
premises, which obstructed the free ingress to and egress from
its premises, thereby preventing its officials and employees
from doing their usual duties.
- RSIC them filed with the NLRC a petition to declare the strike
illegal due to the defiance of the 30 or 15 day cooling-offperiod, disregard of the legal requirement to furnish the
department with the results of the strike vote at least 7 days
before the strike and failure to furnish a written notice of the
meeting to declare a strike to the BLR or the Regional Office
- Labor Arbiter found the strike to be illegal. NLRC affirmed with
modification upon appeal holding that although the strike was
illegal, dismissal was not the proper penalty. It said that the
strikers should be reinstated without backwages due to the
unions belief that the company was committing unfair labor
practice. (Ferrer v. Court of Industrial Relations 6 and Almira v.
BF Goodrich Philippines, Inc)
- Petitioners claim: NLRC was guilty of grave abuse of
discretion.

strike alone does not make the action illegal, which would
justify the dismissal of strikers.
- The Court reiterates that good faith is still a valid defense
against claims of illegality of a strike. We do find, however, not
a semblance of good faith here, but rather, plain arrogance,
pride, and cynicism of certain workers.
- WRT respondent, Isagani Rubio, what militates against his
readmission to the firm is the fact that he had accepted the
sum of P2,448.80 "in full satisfaction of the . . . Decision" (of the
Labor Arbiter).
- The sympathy of the Court is on the side of the laboring
classes, not only because the Constitution imposes sympathy
but because of the one-sided relation between labor and
capital. The Court must take care, however, that in the contest
between labor and capital, the results achieved are fair and in
conformity with the rules. We will not accomplish that objective
here by approving the act of the National Labor Relations
Commission which we hold to constitute a grave abuse of
discretion.
Disposition petition is GRANTED.

ISSUE
1. WON the strike was illegal
2. WON the petition should be granted

FACTS
- UPSU is a union of supervisory employees. The union filed a
petition for certification election on behalf of the route
managers at Pepsi-Cola Products Philippines, Inc. However, its
petition was denied by the med-arbiter and, on appeal, by the
Secretary of Labor and Employment, on the ground that the
route managers are managerial employees and, therefore,
ineligible for union membership under the first sentence of Art.
245 of the Labor Code, which says, Managerial employees are
not eligible to join, assist or form any labor organization.
Supervisory employees shall not be eligible for membership in a
labor organization of the rank-and-file employees but may join,
assist or form separate labor organizations of their own.

HELD
1. YES
- The strike in question was illegal, for failure of the striking
personnel to observe legal strike requirements, to wit: (1) as to
the fifteen-day notice; (2) as to the two-thirds required vote to
strike done by secret ballot; (3) as to submission of the strike
vote to the Department of Labor at least seven days prior to the
strike.
- NLRC also found that certain strikers harassed non-striking
employees, called company officers names, and committed acts
of violence (as a result of which, criminal charges were brought
with the fiscal's office.)
- The strike itself was prompted by no actual, existing unfair
labor practice committed by the petitioner. In effecting a
change in the seating arrangement, the petitioner merely
exercised a reasonable prerogative employees could not validly
question, much less assail as an act of unfair labor practice.
Rearranging furniture cannot justify a four-month-long strike. As
to the private respondent's charges of harassment, the
Commission found none, and as a general rule, the Court is
bound by its findings of fact.
2. YES
- The strike that was illegal in more ways than one, the
reinstated union officers were clearly in bad faith, and to
reinstate them without loss of seniority rights, is to reward them
for an act public policy does not sanction.
- The Ferrer and Almira cases did not involve illegal strikes. In
Ferrer was a defective strike, one conducted in violation of the
thirty-day "cooling-off" period, but one carried out in good faith
"to offset what petitioners were warranted in believing in good
faith to be unfair labor practices [committed by] Management.
What Almira on the other hand declared was that a violent

PHILIPPINE AIRLINES, INC. (PAL) V NLRC


[PAGE 25]
DUNCAN ASSOCIATION V GLAXO WELLCOME PHILS
[PAGE 43]
UNITED PEPSI-COLA SUPERVISORY UNION (UPSU) V
LAGUESMA
288 SCRA 15
MENDOZA; March 25, 1998

ISSUE
1. WON the route managers at Pepsi-Cola Products Philippines,
Inc. are managerial employees
2. WON the first sentence of Art. 245 of the Labor Code,
prohibiting managerial employees from forming, assisting or
joining any labor organization, is constitutional in light of Art. III,
Sec.8 of the Constitution. The right of the people, including
those employed in the public and private sectors, to form
unions, associations, or societies for purposes not contrary to
law shall not be abridged.
HELD
1. YES
- Their job descriptions clearly reveal so. They also fall under
this category under the purview of art. 212. The term
manager generally refers to anyone who is responsible for
subordinates and other organization resources. As a class,
managers constitute three levels of a pyramid.
- What distinguishes them from the rank-and file employees is
that they act in the interest of the employer in supervising such
rank-and-file employees
- Managerial employees may therefore be said to fall into two
distinct categories: the managers per se, who compose the

Labor Law 1
former group described above, and the supervisors who form
the latter group. Whether they belong to the first or second
category, managers, vis--vis employers, are, likewise,
employees
2. NO
- As already stated, whether they belong to the first category
(managers per se) or the second category (supervisors),
managers are employees. Nonetheless, in the United States,
as Justice Puno's separate opinion notes, supervisors have no
right to form unions. They are excluded from the definition of
the term "employee" in 2(3) of the Labor-Management
Relations Act of 1947.
- Commission intended the absolute right to organize of
government workers, supervisory employees, and security
guards to be constitutionally guaranteed. By implication, no
similar absolute constitutional right to organize for labor
purposes should be deemed to have been granted to top-level
and middle managers. As to them the right of self-organization
may be regulated and even abridged conformably to Art. III, 8.
- Types of Managerial Employees:
> FIRST-LINE MANAGERS The lowest level in an organization
at which individuals are responsible for the work of others is
called first-line or first-level management. First-line managers
direct operating employees only; they do not supervise other
managers. Example of first-line managers are the foreman or
production supervisor in a manufacturing plant, the technical
supervisor in a research department, and the clerical supervisor
in a large office. First-level managers are often called
supervisors.
> MIDDLE MANAGERS The term middle management can refer
to more than one level in an organization. Middle managers
direct the activities of other managers and sometimes also
those of operating employees. Middle managers principal
responsibilities are to direct the activities that implement their
organizations policies and to balance the demands of their
superiors with the capacities of their subordinates. A plant
manager in an electronics firm is an example of a middle
manager.
> TOP MANAGERS Composed of a comparatively small group
of executives, top management is responsible for the overall
management of the organization. It establishes operating
policies and guides the organizations interactions with its
environment. Typical titles of top managers are chief
executive officer, president, and senior vice-president.
Actual titles vary from one organization to another and are not
always a reliable guide to membership in the highest
management classification.
Disposition petition is DISMISSED

SONZA V ABS-CBN BROADCASTING CORPORATION


[PAGE 42]
ASIATIC DEVELOPMENT CORP V BROGADA
495 SCRA 166
CORONA; July 14, 2006
NATURE
Petition for review on certiorari
FACTS
- Respondents Wellington and Flordeliza Brogada are the
parents of Fermin B. Brogada who was allegedly employed by
petitioner Asiatic Development Corporation from July 1994 up to
his death in November 14, 1996.
- Respondents filed with the SSC a petition for social security
coverage and payment of contributions in order to avail of the
benefits accruing from the death of Fermin. They alleged that
Fermin worked as survey aide under Engr. Bienvenido Orense,
petitioners geodetic engineer. Fermin was working on a project
with Engr. Orense for one of petitioners clients when he was
shot and killed.

A2010

- 54 -

Disini

- Petitioner denied its liability on the ground that there was no


employer-employee relationship between it and Fermin. It
claimed that Fermin was the employee of Engr. Orense.
- SSS held that Fermin was an employee and was subject to the
compulsory coverage. On appeal, the SSC resolution was
affirmed by the CA.
ISSUE
WON an employer-employee relationship exists
HELD
- The question of WON an employer-employee relationship
exists is a question of fact. In petitions for review on certiorari
under Rule 45, only questions of law may be raised by the
parties and passed upon by this Court. Factual findings of
quasi-judicial bodies, when adopted and confirmed by the CA
and if supported by substantial evidence, are accorded respect
and even finality by this Court. While this Court has recognized
several exceptions to this rule, none of these exceptions finds
application here.
- Both the SSC and CA found that Fermin was petitioners
employee. Thus, petitioner is liable for unpaid social security
contributions.
- Petitioners claims are a mere reiteration of arguments
unsuccessfully raised before the SSC and the CA.
No
compelling reason whatsoever is shown by petitioner for this
Court to reverse the SSCs findings and conclusions, as affirmed
by the CA.
Disposition Petition is DENIED.

VILLAVILLA V CA (SSS, MERCADO, COSUCO)


212 SCRA 488
BELLOSILLO; August 11, 1992
FACTS
- Arturo Villavilla, son of petitioners, was employed as
"tripulante" (crew member) of the fishing boat "F/B Saint
Theresa" from 1974 until September 11, 1977, when the boat
sank off Isla Binatikan, Taytay, Palawan. Arturo was not among
the known survivors of that sinking and had been missing since
then.
- On November 20, 1979, petitioners Andres Villavilla and Ester
Gadiente Villavilla, parents of Arturo, filed a petition with the
Social Security Commission against Reynaldo Mercado and
Marcelino Cosuco, owners of the ill-fated fishing boat, for death
compensation benefits of Arturo whom respondents failed to
register as their employee.
- On May 29, 1981, the Social Security System (SSS) filed a
petition in intervention alleging that records from the SSS
Production Department showed that "F/B Saint Theresa", owned
by Marcelino Cosuco and operated by Reynaldo Mercado, was a
registered member-employer, and that in the event petitioners
succeeded in proving the employment of Arturo with private
respondents, the latter should be held liable in damages
equivalent to the benefits due the petitioners for failure to
report Arturo for coverage pursuant to Sec. 24 (a) of the Social
Security Act, as amended.
- Respondent Cosuco filed his answer denying all allegations in
the petition and claiming that he already sold the fishing boat to
respondent Mercado on December 10, 1975, and from then on
he did not participate anymore in the operation and
management of the boat nor in the hiring of its crewmembers.
- Meanwhile respondent Mercado was declared in default for
failure to file his answer.
- After petitioners had presented their evidence and rested their
case, respondent Cosuco filed a motion to dismiss (demurrer to
evidence) on the ground of res judicata and lack of cause of
action.
- Respondent Social Security Commission issued an Order
dismissing the petition for lack of cause of action.

Labor Law 1
- The Court of Appeals affirmed the questioned Order of
respondent Commission there being no reversible error.
ISSUE
1. WON there was an employer-employee relationship between
petitioners' deceased son, Arturo Villavilla, and herein private
respondents
2. WON private respondents are liable for death compensation
benefits of Arturo Villavilla
3. WON there was a violation of the Social Security Act, as
amended, by private respondents for not registering Arturo
Villavilla with the System as their employee as mandated by
law
HELD
1. NO
- The records disclose that the relationship between Mercado
and the crew members of the ship headed by its skipper, Capt.
Pedro Matibag, is one positively showing the existence of a joint
venture. This is clearly revealed in the testimonies of Capt.
Pedro Matibag and Gil Chua, a crew member, both witnesses for
petitioners.
- The arrangement between the boat owner and the crew
members, one of whom was petitioners' son, partook of the
nature of a joint venture: the crew members did not receive
fixed compensation as they only shared in their catch; they
ventured to the sea irrespective of the instructions of the boat
owners, i.e., upon their own best judgment as to when, how
long, and where to go fishing; the boat owners did not hire
them but simply joined the fishing expedition upon invitation of
the ship master, even without the knowledge of the boat owner.
In short, there was neither right of control nor actual exercise of
such right on the part of the boat owner over his crew
members.
- It is clear that there was no employer-employee relationship
between petitioner's son Arturo and private respondent
Mercado, much less private respondent Cosuco. As such, Arturo
could not be made subject of compulsory coverage under the
Social Security Act; hence, private respondents cannot be said
to have violated said law when they did not register him with
the Social Security System. A fortiori, respondent as well as
intervenor are not answerable to petitioners for any death
benefits under the law.
- Culled from the foregoing, the inexorable conclusion is that
respondent Court of Appeals did not err in sustaining the
judgment of respondent Social Security Commission.
- It may not be amiss to mention that while petitioners merely
raise factual questions which are not proper under Rule 45 of
the Rules of Court, We nevertheless went to great lengths in
dissecting the facts of this case if only to convince Us that
petitioners, who are pauper litigants and seeking claims under a
social legislation, have not been denied its benefits. For, We are
not unaware that in this jurisdiction all doubts in the
implementation and interpretation of provisions of social
legislations should be resolved in favor of the working class.
But, alas, justice is not fully served by sustaining the contention
of the poor simply because he is poor. Justice is done by
properly applying the law regardless of the station in life of the
contending parties.

MIGUEL V JCT GROUP INC


453 SCRA 529
PANGANIBAN; March 16, 2005
NATURE
Petition for Review on Certiorari under Rule 45 of the Rules of
Court CA decision on the ground of grave abuse of discretion
because it annulled and set aside decisions of the labor arbiter
and NLRC
FACTS

A2010

- 55 -

Disini

- 1984 > Glorious Sun Garment Manufacturing Company was a


garment exporter until it folded up and, thereafter, De Soliel
Apparel Manufacturing Corporation and American Inter-Fashion
Corporation took over Glorious Suns manufacturing facilities
and absorbed its employees (petitioners Miguel et al)
- 1986 > PCGG sequestered De Soleil and AIFC
- 1989 > JCT Group, Inc. and De Soleil executed a Management
and Operating Agreement for servicing De Soleils export quota
to ensure its rehabilitation and preserve its viability and
profitability
- 1990 > De Soleil ceased business operations thus terminating
employment
- 1993 > complaints for illegal dismissal and payment of
backwages before NLRC against De Soleil, AIFC, PCGG, Glorious
Sun, JCT, Nemesio Co and Vicente Cuevas III. But JCT and
Cuevas filed a motion to dismiss due to lack of jurisdiction
because of the absence of employer-employee relationship
between them and petitioners.
- 1995 > Without resolving the motion to dismiss, Labor Arbiter
Sampang rendered (1)
Declaring De Soleil, AIFC, PCGG,
Glorious Sun, JCT, Nemesio Co and Cuevas jointly and severally
guilty of illegal dismissal and to pay complainants backwages,
separation pay, service incentive leave pay, 13th month pay,
unpaid salaries as computed by the Research and Information
Unit (2) Declaring De Soleil et al liable for the payment of
attorneys fees (10% of the total awards or P3,691,743.06)
- Because of the huge amount [monetary award, inclusive of
attorneys fees, aggregated P41,313,094.98 computed by
Research and Information Unit], JCT and Cuevas and Glorious
Sun filed separate motions with NLRC for reduction of the
appeal bond . NLRC reduced it to P5,000,000.00 for each
respondent. Again, they filed a motion for reconsideration of
said order by way of further reduction of the bond to
P500,000.00. CA and SCA denied their motions
- 1995 > Meanwhile, Glorious Sun and JCT et al et al appealed
the labor arbiters decision to NLRC and petitioners filed a
motion to dismiss the appeals bec of failure to post a bond as
required in A223 LC
- 1996 > NLRC absolved Glorious Sun and dismissed JCT et als
appeal and sent to CA for appropriate disposition
- CA: reversed NLRC decision and remanded the case to the
labor arbiter bec it found no factual basis for the ruling that JCT
had become the employer of petitioners after the cessation of
operations of Glorious Sun and failed to explain Cuevas liability
in solidum with AIF, De Soleil and JCT; hence this petition by
Miguel et al
ISSUE
WON CA committed grave abuse of discretion amounting in
ruling to remand the case to the labor arbiter because of lack of
factual findings to prove employer-employee relationship
between JCT et al and Miguel et al which would be the basis of
the liability of JCT et al
HELD
NO.
Instead, NLRC and the Labor arbiter abused their
discretion when they ruled in favor of the petitioners without
determining
the
existence
of
an
employer-employee
relationship between them and respondents because it was
silent on why JCT and Cuevas were held liable.
Doctrine Grave abuse of discretion implies such capricious and
whimsical exercise of judgment as to be equivalent to lack or
excess of jurisdiction.
That is, power is arbitrarily or
despotically exercised by reason of passion, prejudice, or
personal hostility; and caprice is so patent or so gross as to
amount to an evasion of a positive duty, or to a virtual refusal
to perform the duty enjoined or to act at all in contemplation of
law.
Reasoning
- LABOR ARBITER: made no determination whether there was
employer-employee relationship and, if so, whether JCT et al
assumed the obligations of Miguel et als previous employers.

Labor Law 1
There is no dispute that given the nature of their functions and
length of services, were regular employees. But the question is:
who was/were their employer/s?
- MOA: does not appear that JCT became the employer of
Miguel et al by virtue of this
- NLRC: silent on JCT being the employer of Miguel et al after
Glorious Sun ceased operations, save for its conclusion that
they were absorbed by, or their work continued under JCT and
did not state the reason for liability in solidum of Cuevas.
Computation of the monetary award totaling P37,557,317.08
(exclusive of attorneys fees) covers a period starting on initial
employment (with Glorious Sun) some dating back to 1978
- Saballa v NLRC > This Court has previously held that judges
and arbiters should draw up their decisions and resolutions with
due care, and make certain that they truly and accurately
reflect their conclusions and their final dispositions. The same
thing goes for the findings of fact made by the NLRC, as it is a
settled rule that such findings are entitled to great respect and
even finality when supported by substantial evidence;
otherwise, they shall be struck down for being whimsical and
capricious and arrived at with grave abuse of discretion. It is a
requirement of due process and fair play that the parties to a
litigation be informed of how it was decided, with an
explanation of the factual and legal reasons that led to the
conclusions of the court. A decision that does not clearly and
distinctly state the facts and the law on which it is based leaves
the parties in the dark as to how it was reached and is
especially prejudicial to the losing party, who is unable to
pinpoint the possible errors of the court for review by a higher
tribunal.
Obiter
- employer-employee relationship test:
1) power to select employees
2) who pays for their wages
3) who has the power to dismiss them, and
4) who exercises control in the methods and the results by
which the work is accomplished
*** The last factor, the control test, is the most important.
Disposition Petition is DENIED and the assailed Decision
AFFIRMED

WACK-WACK GOLF & COUNTRY CLUB V NLRC


(CAGASAN, DOMINGUEZ, BSMI)
456 SCRA 280
CALLEJO, SR; April 15, 2005
NATURE
Petition for review decision of CA
FACTS
- A large portion of the Wack Wack (WW) clubhouse (including
the kitchen) was destroyed by fire, and because of this, the
management had to suspend operations of the Food and
Beverage department, requiring the suspension of 54
employees. The Employees Union found the suspension as
arbitrary and constitutive of union-busting, and went on strike.
The parties soon after entered into an amicable settlement,
whereby a special separation benefit/ retirement package was
formulated. The same provides for, among other things, a 1
month separation pay for every year of service, and be
considered on priority basis for employment by concessionaires
and/or contractors, and even by the club, upon full resumption
of operations.
- The package was availed of by 3 employees (Cagasan,
Dominguez, and Baluyot), who received large sums of money as
separation pay.
Soon after, WW entered into a Management Contract with
Business Staffing and Management Inc (BSMI), whereby the
latter will provide management services for WW.
Cagasan and Dominguez filed their application for employment
with BMSI. They, by reason of the priority given by the

A2010

- 56 -

Disini

separation package, were rehired on probationary status by


BMSI.
- WW also engaged other contractors in the operations of the
club (like janitorial services, Finance and accounting services).
Because of the various management service contracts, BMSI
made an organizational analysis and manpower evaluation to
streamline its operations. It found the positions of Cagasan and
Domiguez redundant, and subsequently terminated them.
Cagasan and Dominguez then filed complaints in the NLRC for
illegal dismissal against WW. NLRC ordered reinstatement
ISSUES
1. WON BMSI is an independent contractor (which will answer
the question as WON there was an employer-employee
relationship)
2. WON the employees were illegally dismissed
HELD
1. YES
Reasoning
- An independent contractor is one who undertakes job
contracting, i.e., a person who: (a) carries on an independent
business and undertakes the contract work on his own account
under his own responsibility according to his own manner and
method, free from the control and direction of his employer or
principal in all matters connected with the performance of the
work except as to the results thereof; and (b) has substantial
capital or investment in the form of tools, equipments,
machineries, work premises and other materials which are
necessary in the conduct of the business. Jurisprudence shows
that determining the existence of an independent contractor
relationship, several factors may be considered, such as, but
not necessarily confined to, whether or not the contractor is
carrying on an independent business; the nature and extent of
the work; the skill required; the term and duration of the
relationship; the right to assign the performance of specified
pieces of work; the control and supervision of the work to
another; the employers power with respect to the hiring, firing,
and payment of the contractors workers; the control of the
premises; the duty to supply premises, tools, appliances,
materials and labor; and the mode, manner and terms of
payment.
- There is indubitable evidence showing that BSMI is an
independent contractor, engaged in the management of
projects, business operations, functions, jobs and other kinds of
business ventures, and has sufficient capital and resources to
undertake its principal business. It had provided management
services to various industrial and commercial business
establishments.
- In December 1993, Labor Sec. Laguesma, in a case,
recognized BSMI as an independent contractor. As a legitimate
job contractor, there can be no doubt as to the existence of an
employer-employee relationship between the contractor and
the workers.
Thus, there is no employer-employee relation between WW and
the workers.
2. NO
Ratio
As there was no employer-employee relationship
between WW and the complainants, there can be no illegal
dismissal.
Reasoning
- the complainants (private respondents herein) were validly
terminated upon their option to take the separation package
provided by WW. Thus, the same have no cause of action
against WW.
Disposition
Petition granted. CA and NLRC decisions set
aside

PHILIPPINE GLOBAL COMMUNICATIONS INC V DE


VERA
459 SCRA 260

Labor Law 1

A2010

GARCIA; June 7, 2005


NATURE
petition for review on certiorari
FACTS
- Petitioner Philippine Global Communications, Inc. (PhilCom), is
a corporation engaged in the business of communication
services and allied activities, while respondent Ricardo De Vera
is a physician by profession whom petitioner enlisted to attend
to the medical needs of its employees.
- It appears that on 15 May 1981, De Vera, via a letter dated 15
May 1981, offered his services to the petitioner, therein
proposing his plan of works required of a practitioner in
industrial medicine.
- The parties agreed and formalized respondents proposal in a
document denominated as RETAINERSHIP CONTRACT which will
be for a period of one year subject to renewal, it being made
clear therein that respondent will cover the retainership the
Company previously had with Dr. K. Eulau and that
respondents retainer fee will be at P4,000.00 a month. Said
contract was renewed yearly. The retainership arrangement
went on from 1981 to 1994 with changes in the retainers fee.
However, for the years 1995 and 1996, renewal of the contract
was only made verbally. On December 1996 Philcom, thru a
letter bearing on the subject boldly written as TERMINATION
RETAINERSHIP CONTRACT, informed De Vera of its decision to
discontinue the latters retainers contract with the Company
effective at the close of business hours of December 31, 1996
because management has decided that it would be more
practical to provide medical services to its employees through
accredited hospitals near the company premises.
- On 22 January 1997, De Vera filed a complaint for illegal
dismissal before the National Labor Relations Commission
(NLRC), alleging that that he had been actually employed by
Philcom as its company physician since 1981 and was
dismissed without due process. He averred that he was
designated as a company physician on retainer basis for
reasons allegedly known only to Philcom. He likewise professed
that since he was not conversant with labor laws, he did not
give much attention to the designation as anyway he worked on
a full-time basis and was paid a basic monthly salary plus fringe
benefits, like any other regular employees of Philcom.
- On 21 December 1998, Labor Arbiter Ramon Valentin C. Reyes
came out with a decision dismissing De Veras complaint for
lack of merit, on the rationale that as a retained physician
under a valid contract mutually agreed upon by the parties, De
Vera was an independent contractor and that he was not
dismissed but rather his contract with [PHILCOM] ended when
said contract was not renewed after December 31, 1996.
NLRC reversed (the word used is modified) that of the Labor
Arbiter, on a finding that De Vera is Philcoms regular
employee and accordingly directed the company to reinstate
him to his former position without loss of seniority rights and
privileges and with full backwages from the date of his
dismissal until actual reinstatement.
- Court of Appeals modified NLRCs decision that of the NLRC by
deleting the award of traveling allowance, and ordering
payment of separation pay to De Vera in lieu of reinstatement.
ISSUES
WON an employer-employee
petitioner and respondent

relationship

exists

between

HELD
NO
- De Vera was an independent contractor beinf the retained
physician of petitioner company.
- In a long line of decisions, the Court, in determining the
existence of an employer-employee relationship, has invariably
adhered to the four-fold test, to wit: the selection and
engagement of the employee; the payment of wages; the

- 57 -

Disini

power of dismissal; and the power to control the employees


conduct, or the so-called control test, considered to be the
most important element.
- Applying the four-fold test to this case, we initially find that it
was respondent himself who sets the parameters of what his
duties would be in offering his services to petitioner in the letter
which he sent to petitioner.
- The letter was substantially the basis of the labor arbiters
finding that there existed no employer-employee relationship
between petitioner and respondent, in addition to the following
factual settings:
- The fact that the complainant was not considered an
employee was recognized by the complainant himself in a
signed letter, the tenor of which indicated that the complainant
was proposing to extend his time with the respondent and
seeking additional compensation for said extension. This shows
that the respondent PHILCOM did not have control over the
schedule of the complainant as it [is] the complainant who is
proposing his own schedule and asking to be paid for the same.
This is proof that the complainant understood that his
relationship with the respondent PHILCOM was a retained
physician and not as an employee. If he were an employee he
could not negotiate as to his hours of work.
- De Veras service for the respondent was covered by a
retainership contract [which] was renewed every year from
1982 to 1994. Upon reading the contract dated September 6,
1982, signed by the complainant himself (Annex C of
Respondents Position Paper), it clearly states that is a
retainership contract. The retainer fee is indicated thereon and
the duration of the contract for one year is also clearly indicated
in paragraph 5 of the Retainership Contract. The complainant
cannot claim that he was unaware that the contract was good
only for one year, as he signed the same without any
objections. The complainant also accepted its renewal every
year thereafter until 1994. As a literate person and educated
person, the complainant cannot claim that he does not know
what contract he signed and that it was renewed on a year to
year basis.
- The labor arbiter added the indicia, not disputed by
respondent, that from the time he started to work with
petitioner, he never was included in its payroll; was never
deducted any contribution for remittance to the Social Security
System (SSS); and was in fact subjected by petitioner to the ten
(10%) percent withholding tax for his professional fee, in
accordance with the National Internal Revenue Code, matters
which are simply inconsistent with an employer-employee
relationship.
- Clearly, the elements of an employer-employee relationship
are wanting in this case. We may add that the records are
replete with evidence showing that respondent had to bill
petitioner for his monthly professional fees It simply runs
against the grain of common experience to imagine that an
ordinary employee has yet to bill his employer to receive his
salary.
- We note, too, that the power to terminate the parties
relationship was mutually vested on both. Either may terminate
the arrangement at will, with or without cause.Finally,
remarkably absent from the parties arrangement is the
element of control, whereby the employer has reserved the
right to control the employee not only as to the result of the
work done but also as to the means and methods by which the
same is to be accomplished.
- Here, petitioner had no control over the means and methods
by which respondent went about performing his work at the
company premises. He could even embark in the private
practice of his profession, not to mention the fact that
respondents work hours and the additional compensation
therefor were negotiated upon by the parties. In fine, the
parties themselves practically agreed on every terms and
conditions of respondents engagement, which thereby negates
the element of control in their relationship. For sure, respondent

Labor Law 1
has never cited even a single instance when petitioner
interfered with his work.
Disposition petition is GRANTED and the challenged decision
of the Court of Appeals REVERSED and SET ASIDE. The 21
December 1998 decision of the labor arbiter is REINSTATED.

SONZA V ABS-CBN
[PAGE 42]
JARDIN V NLRC (PHILJAMA INTL)
326 SCRA 299
QUISUMBING; February 23, 2000
NATURE
Special civil action for certiorari seeks to annul the decision of
public respondent promulgated on October 28, 1994, in NLRC
NCR CA No. 003883-92, and its resolution dated December 13,
1994 which denied petitioners motion for reconsideration.
FACTS
- Petitioners were drivers of private respondent, Philjama
International Inc., a domestic corporation engaged in the
operation of "Goodman Taxi." Petitioners used to drive private
respondents taxicabs every other day on a 24-hour work
schedule under the boundary system. Under this arrangement,
the petitioners earned an average of P400.00 daily.
Nevertheless, private respondent admittedly regularly deducts
from petitioners daily earnings the amount of P30.00
supposedly for the washing of the taxi units. Believing that the
deduction is illegal, petitioners decided to form a labor union to
protect their rights and interests.
- Upon learning about the plan of petitioners, private
respondent refused to let petitioners drive their taxicabs when
they reported for work on August 6, 1991, and on succeeding
days. Petitioners suspected that they were singled out because
they were the leaders and active members of the proposed
union. Aggrieved, petitioners filed with the labor arbiter a
complaint against private respondent for unfair labor practice,
illegal dismissal and illegal deduction of washing fees. In a
decision dated August 31, 1992, the labor arbiter dismissed said
complaint for lack of merit.
- On appeal, the NLRC (public respondent herein), in a decision
dated April 28, 1994, reversed and set aside the judgment of
the labor arbiter. The labor tribunal declared that petitioners
are employees of private respondent, and, as such, their
dismissal must be for just cause and after due process.
- Private respondents first motion for reconsideration was
denied. Remaining hopeful, private respondent filed another
motion for reconsideration. This time, public respondent, in its
decision dated October 28, 1994, granted aforesaid second
motion for reconsideration. It ruled that it lacks jurisdiction over
the case as petitioners and private respondent have no
employer-employee relationship. It held that the relationship of
the parties is leasehold which is covered by the Civil Code
rather than the Labor Code.
ISSUE
WON the NLRC committed grave abuse of discretion in
entertaining the motion for reconsideration and in holding that
there is no employer-employee relationship in the boundary
system.
HELD
YES
Ratio Only one motion for reconsideration from the same party
is allowed before the NLRC in line with the policy of assisting
the parties in obtaining an expeditious and inexpensive
settlement of labor cases. When the NLRC entertained the

A2010

- 58 -

Disini

second motion for reconsideration, it therefore committed


grave abuse of discretion.
Reasoning
- The phrase "grave abuse of discretion amounting to lack or
excess of jurisdiction" has settled meaning in the jurisprudence
of procedure. It means such capricious and whimsical exercise
of judgment by the tribunal exercising judicial or quasi-judicial
power as to amount to lack of power.
- In this case before us, private respondent exhausted
administrative remedy available to it by seeking reconsideration
of public respondents decision dated April 28, 1994, which
public respondent denied. With this motion for reconsideration,
the labor tribunal had ample opportunity to rectify errors or
mistakes it may have committed before resort to courts of
justice can be had. Thus, when private respondent filed a
second motion for reconsideration, public respondent should
have forthwith denied it in accordance with Rule 7, Section 14
of its New Rules of Procedure which allows only one motion for
reconsideration from the same party, thus:
"SEC. 14. Motions for Reconsideration. --- Motions for
reconsideration of any order, resolution or decision of the
Commission shall not be entertained except when based on
palpable or patent errors, provided that the motion is under
oath and filed within ten (10) calendar days from receipt of
the order, resolution or decision with proof of service that a
copy of the same has been furnished within the reglementary
period the adverse party and provided further, that only one
such motion from the same party shall be entertained."
[Emphasis supplied]
- The rationale for allowing only one motion for reconsideration
from the same party is to assist the parties in obtaining an
expeditious and inexpensive settlement of labor cases. For
obvious reasons, delays cannot be countenanced in the
resolution of labor disputes. The dispute may involve no less
than the livelihood of an employee and that of his loved ones
who are dependent upon him for food, shelter, clothing,
medicine, and education. It may as well involve the survival of a
business or an industry.
- As correctly pointed out by petitioner, the second motion for
reconsideration filed by private respondent is indubitably a
prohibited pleading which should have not been entertained at
all. Public respondent cannot just disregard its own rules on the
pretext of "satisfying the ends of justice", especially when its
disposition of a legal controversy ran afoul with a clear and long
standing jurisprudence in this jurisdiction as elucidated in the
subsequent discussion. Clearly, disregarding a settled legal
doctrine enunciated by this Court is not a way of rectifying an
error or mistake. In our view, public respondent gravely abused
its discretion in taking cognizance and granting private
respondents second motion for reconsideration as it wrecks the
orderly procedure in seeking reliefs in labor cases.
Obiter
- There is another compelling reason why we cannot leave
untouched the flip-flopping decisions of the public respondent.
As mentioned earlier, its October 28, 1994 judgment is not in
accord with the applicable decisions of this Court. The labor
tribunal reasoned out as follows:
- Four-fold test for employer-employee relations:
(1) the selection and engagement of the employee;
(2) the payment of wages;
(3) the power of dismissal; and
(4) the power of control the employees conduct.
- NLRC found that the boundary system is a leasehold system
which takes it out of the ordinary notion of control over
employees conduct.
- The SC iterated its ruling that the relationship between
jeepney owners/operators on one hand and jeepney drivers on
the other under the boundary system is that of employeremployee and not of lessor-lessee.
- The SC explained that in the lease of chattels, the lessor loses
complete control over the chattel leased although the lessee
cannot be reckless in the use thereof, otherwise he would be

Labor Law 1
responsible for the damages to the lessor. In the case of
jeepney owners/operators and jeepney drivers, the former
exercise supervision and control over the latter. The
management of the business is in the owners hands. The
owner as holder of the certificate of public convenience must
see to it that the driver follows the route prescribed by the
franchising authority and the rules promulgated as regards its
operation.
- As consistently held by this Court, termination of employment
must be effected in accordance with law. The just and
authorized causes for termination of employment are
enumerated under Articles 282, 283 and 284 of the Labor Code.
The requirement of notice and hearing is set-out in Article 277
(b) of the said Code. Hence, petitioners, being employees of
private respondent, can be dismissed only for just and
authorized cause, and after affording them notice and hearing
prior to termination. In the instant case, private respondent had
no valid cause to terminate the employment of petitioners.
Neither were there two (2) written notices sent by private
respondent informing each of the petitioners that they had
been dismissed from work. These lack of valid cause and failure
on the part of private respondent to comply with the twin-notice
requirement underscored the illegality surrounding petitioners
dismissal.
- Under the law, an employee who is unjustly dismissed from
work shall be entitled to reinstatement without loss of seniority
rights and other privileges and to his full backwages, inclusive
of allowances, and to his other benefits or their monetary
equivalent computed from the time his compensation was
withheld from him up to the time of his actual reinstatement It
must
be
emphasized,
though,
that
recent
judicial
pronouncements distinguish between employees illegally
dismissed prior to the effectivity of Republic Act No. 6715 on
March 21, 1989, and those whose illegal dismissals were
effected after such date. Thus, employees illegally dismissed
prior to March 21, 1989, are entitled to backwages up to three
(3) years without deduction or qualification, while those illegally
dismissed after that date are granted full backwages inclusive
of allowances and other benefits or their monetary equivalent
from the time their actual compensation was withheld from
them up to the time of their actual reinstatement. The
legislative policy behind Republic Act No. 6715 points to "full
backwages" as meaning exactly that, i.e., without deducting
from backwages the earnings derived elsewhere by the
concerned employee during the period of his illegal dismissal.
Considering that petitioners were terminated from work on
August 1, 1991, they are entitled to full backwages on the basis
of their last daily earnings.

MANILA GOLF & COUNTRY CLUB INC V IAC


(LLAMAR)
337 SCRA 207
NARVASA; September 27, 1994
NATURE
Petition for review
FACTS
- three separate proceedings, all initiated by or on behalf of
herein private respondent and his fellow caddies:
1) filed with the Social Security Commission (SSC) via petition of
17 persons who styled themselves "Caddies of Manila Golf and
Country
Club-PTCCEA
(Philippine
Technical,
Clerical,
Commercial Employees Association) for coverage and
availment of benefits under the Social Security Act. It alleged

A2010

- 59 -

Disini

that although the petitioners were employees of the Manila Golf


and Country Club, a domestic corporation, the latter had not
registered them as such with the SSS.
2) a certification election case filed with the Labor Relations
Division of the Ministry of Labor by the PTCCEA on behalf of the
same caddies- it was resolved in favor of the petitioners
3) a compulsory arbitration case initiated before the Arbitration
Branch of the Ministry of Labor by the same labor organizationit was dismissed for lack of merit by Labor Arbiter on the ground
that there was no employer-employee relationship between the
petitioning caddies and the respondent Club
- In the case before the SSC, the Club filed answer praying for
the dismissal of the petition, alleging in substance that the
petitioners, caddies by occupation, were allowed into the Club
premises to render services as such to the individual members
and guests playing the Club's golf course and who themselves
paid for such services; that as such caddies, the petitioners
were not subject to the direction and control of the Club as
regards the manner in which they performed their work; and
hence, they were not the Club's employees.
- Subsequently, all but two (Fermin Llamar and Raymundo
Jomok) of the 17 petitioners of their own accord withdrew their
claim for social security coverage, avowedly coming to realize
that indeed there was no employment relationship between
them and the Club. The Commission dismissed the petition for
lack of merit:. . . that the caddy's fees were paid by the golf
players themselves and not by respondent club...While
respondent club promulgates rules and regulations on the
assignment, deportment and conduct of caddies the same are
designed to impose personal discipline among the caddies but
not to direct or conduct their actual work. In fact, a golf player
is at liberty to choose a caddy of his preference regardless of
the respondent club's group rotation system and has the
discretion on whether or not to pay a caddy...This lends
credence to respondent's assertion that the caddies are never
their employees in the absence of two elements, namely, (1)
payment of wages and (2) control or supervision over them.
- From this Resolution appeal was taken to the IAC by the union
representing Llamar and Jomok. The appeal ascribed two errors
to the SSC: (1) refusing to suspend the proceedings to await
judgment by the Labor Relations Division of National Capital
Regional Office in the certification election on the precise issue
of the existence of employer-employee relationship between
the respondent club and the appellants, it being contended that
said issue was "a function of the proper labor office"; and (2)
adjudicating that self same issue a manner contrary to the
ruling of the Director of the Bureau of Labor Relations, which
"has not only become final but (has been) executed or
(become) res adjudicata."
- IAC: declared Fermin Llamar an employee of the Manila
Gold and Country Club, ordering that he be reported as such for
social security coverage and paid any corresponding benefits,
but it conspicuously ignored the issue of res adjudicata raised in
said second assignment.
- The questioned employer-employee relationship between the
Club and Fermin Llamar passed the so-called "control test,"
establishment in the case i.e., "whether the employer
controls or has reserved the right to control the employee not
only as to the result of the work to be done but also as to the
means and methods by which the same is to be accomplished,"
the Club's control over the caddies encompassing:
(a) the promulgation of no less than 24 rules and regulations
just about every aspect of the conduct that the caddy must
observe, or avoid, when serving as such, any violation of any
which could subject him to disciplinary action, which may
include suspending or cutting off his access to the club
premises; (b) the devising and enforcement of a group
rotation system whereby a caddy is assigned a number which
designates his turn to serve a player; (c) the club's
"suggesting" the rate of fees payable to the caddies.

Labor Law 1
ISSUE
WON persons rendering caddying services for members of golf
clubs and their guests in said clubs' courses or premises are the
employees of such clubs and therefore within the compulsory
coverage of the Social Security System (SSS)
HELD
NO
Ratio The Court does not agree that said facts necessarily or
logically point to such a relationship, and to the exclusion of any
form of arrangements, other than of employment, that would
make the respondent's services available to the members and
guest of the petitioner. As long as it is, the list made in the
appealed decision detailing the various matters of conduct,
dress, language, etc. covered by the petitioner's regulations,
does not, in the mind of the Court, so circumscribe the actions
or judgment of the caddies concerned as to leave them little or
no freedom of choice whatsoever in the manner of carrying out
their services.
Reasoning
- In the very nature of things, caddies must submit to some
supervision of their conduct while enjoying the privilege of
pursuing their occupation within the premises and grounds of
whatever club they do their work in. For all that is made to
appear, they work for the club to which they attach themselves
on sufference but, on the other hand, also without having to
observe any working hours, free to leave anytime they
please, to stay away for as long they like. It is not
pretended that if found remiss in the observance of said rules,
any discipline may be meted them beyond barring them
from the premises which, it may be supposed, the Club
may do in any case even absent any breach of the rules,
and without violating any right to work on their part. All
these considerations clash frontally with the concept of
employment. The IAC would point to the fact that the Club
suggests the rate of fees payable by the players to the caddies
as still another indication of the latter's status as employees. It
seems to the Court, however, that the intendment of such fact
is to the contrary, showing that the Club has not the
measure of control over the incidents of the caddies'
work and compensation that an employer would
possess. In the final analysis, petitioner has no was of
compelling the presence of the caddies as they are not
required to render a definite number of hours of work on
a single day. Even the group rotation of caddies is not
absolute because a player is at liberty to choose a caddy of his
preference regardless of the caddy's order in the rotation.
Obiter (on issue of res judicata)
- That same issue of res adjudicata, ignored by the IAC beyond
bare mention thereof, as already pointed out, is now among the
mainways of the private respondent's defenses to the petition
for review.
- Because the same question of employer-employee relationship
has been dragged into three different fora, willy-nilly and in
quick succession, it has birthed controversy as to which of the
resulting adjudications must now be recognized as decisive. On
the one hand, there is the certification case where the decision
found for the existence of employer-employee relationship
between the parties; on the other, the compulsory arbitration
case which was dismissed for lack of merit on the ground that
there existed no such relationship between the Club and the
private respondent.
- It is well settled that for res adjudicata, or the principle of bar
by prior judgment, to apply, the following essential requisites
must concur: (1) there must be a final judgment or order; (2)
said judgment or order must be on the merits; (3) the court
rendering the same must have jurisdiction over the subject
matter and the parties; and (4) there must be between the two
cases identity of parties, identity of subject matter and identity
of cause of action.

A2010

- 60 -

Disini

- A certification proceedings is not a "litigation" in the sense in


which the term is commonly understood, but mere investigation
of a non-adversary, fact-finding character, in which the
investigating agency plays the part of a disinterested
investigator seeking merely to ascertain the desires of the
employees as to the matter of their representation.
- In any case, this Court is not inclined to allow private
respondent the benefit of any doubt as to which of the
conflicting ruling just adverted to should be accorded primacy,
given the fact that it was he who actively sought them
simultaneously, as it were, from separate fora. Accordingly, the
IAC is not to be faulted for ignoring private respondent's
invocation of res adjudicata; on contrary, it acted correctly in
doing so.
Disposition Reversed and set aside, it being hereby declared
that the private respondent, Fermin Llamar, is not an employee
of petitioner Manila Golf and Country Club and that petitioner is
under no obligation to report him for compulsory coverage to
the Social Security System.

FELIX V BUENASEDA
240 SCRA 139
KAPUNAN; January 17, 1995
NATURE
Petition for review on certiorari
FACTS
- Petitioner Dr. Alfredo Felix joined the National Center for
Mental Health (NCMH) as a resident physician and after only 3
years, was promoted to Senior Resident Physician, a position he
held until the Ministry of Health reorganized the NCMH in Jan.
1988, pursuant to E.O. 119. Under the said reorganization, Felix
was appointed to the position of Sr. Resident Physician in a
temporary capacity immediately after he and other employees
allegedly tendered their courtesy resignations to the Sec. of
Health. Felix was later promoted to the position of Medical
Specialist 1 (Temporary Status) which was renewed the
following year.
- In 1988, the Dept. of Health or DoH issued Dept. Order (D.O.)
347, which required board certification as a prerequisite for
renewal of specialist positions in various med. centers, hospitals
and agencies and specifically provided that specialists working
in various branches of DoH be recognized as Fellows of their
respective societies and/or Diplomates of their specialty
boards or both, the purpose of which was to upgrade the quality
of specialists in DoH hospitals by requiring them to pass
rigorous theoretical and clinical exams given by recognized
specialty boards.
- (Then) Sec. of Health Alfredo Bengzon issued D.O. 478
(amending Sec.4 of D.O. 347) which provided for an extension
of appointments of Medical Specialists in cases where
termination of those who failed to meet the requirement for
board certification might result in disruption of hospital
services. The said order provided, among others, that:
xxxxx 2. Medical specialists recommended for extension of
appointment shall meet the following minimum criteria:
a. DOH medical specialist certified
b. Has been in the service of the Department at least three
(3) years prior to December 1988
c. Has applied or taken the specialty board examination.
- In 1991, after reviewing petitioners service record and
performance, the Medical Credentials Committee of the NCMH
recommended non-renewal of his appointment as Medical

Labor Law 1
Specialist 1, informing him of its decision on Aug. 22, 1991. He
was, however, allowed to continue in the service, and receive
his salary even after being informed of the termination of his
appointment.
- On Nov. 25, 1991, the Chiefs of Service held an emergency
meeting to discuss the petitioners case. In the meeting, the
overall consensus among the dept. heads was for petitioners
non-renewal where his poor performance, frequent tardiness
and inflexibility were pointed as among the factors responsible
for the recommendation not to renew his appointment. The
matter was referred to the CSC, which ruled that the temporary
appointment can be terminated any time and that any renewal
of such appointment is within the discretion of the appointing
authority. Consequently, petitioner was advised by hospital
authorities to vacate his cottage. Refusing to comply, petitioner
filed a petition with the Merit System Protection Board (MSPB)
complaining about the alleged harassment and questioning the
non-renewal of his appointment, the MSPB, however, dismissed
his complaint for lack of merit.
- This decision was appealed to the Civil Service Commission
(CSC) which dismissed the same. The MFR was also denied by
the CSC hence this appeal.
Petitioners claims
1. CSC erred in holding that by submitting his courtesy
resignation and accepting his temporary appointment,
petitioner had effectively divested himself of his security of
tenure, considering the circumstances of such courtesy
resignation and acceptance of appointment.
2. Respondent commission erred in not declaring that the
conversion of the permanent appointment of petitioner to
temporary was done in bad faith in the guise of reorganization
and thus invalid, being violative of the petitioners right of
security of tenure.
Respondent CSCs claims
1. The petitioners temporary appointments after the
reorganization pursuant to E.O. 119 were valid and did not
violate his constitutional right to security of tenure.
2. Petitioner is guilty of estoppel or laches, having acquiesced
to such temporary appointments from 1988-1991
3. The respondent CSC did not act with grave abuse of
discretion in affirming the petitioners non-renewal of his
appointment to the NCMH.
ISSUE
WON petitioners removal from a permanent position (Medical
Specialist 1), as a result of the reorganization of the DoH, was
void for being violative of the constitutional provision on
security of tenure
HELD
NO
Reasoning
- A residency or resident physician position in a medical
specialty is never a permanent one. Residency connotes
training and temporary status. Promotion to the next postgraduate year is based on merit and performance determined
by periodic evaluations and examinations of knowledge, skills
and bedside manner. Under this system, residents, especially
those in university teaching hospitals enjoy their right to
security of tenure only to the extent that they periodically make
the grade. While physicians (or consultants) of specialist rank
are not subject to the same stringent evaluation procedures,
specialty societies require continuing education as a
requirement for accreditation in good standing, in addition to
peer review processes based on performance, mortality and
morbidity audits, feedback from residents, interns and medical
students and research output. The nature of the contracts
of resident physicians meets traditional tests for
determining employer employee relationships, but
because the focus of residency is training, they are
neither here nor there. Moreover, stringent standards and
requirements for renewal of specialist rank positions or for

A2010

- 61 -

Disini

promotion to the next postgraduate residency year are


necessary because lives are ultimately at stake.
- From the position of senior resident physician, which he held
at the time of the government reorganization, the next logical
step in the stepladder process was obviously his promotion to
the rank of Medical Specialist 1, a position which he apparently
accepted. Such status, however, clearly carried with it certain
professional responsibilities including the responsibility of
keeping up with the minimum requirements of specialty rank,
the responsibility of keeping abreast with current knowledge in
his specialty and in Medicine in general, and the responsibility
of completing board certification requirements within a
reasonable period of time. The evaluation made by petitioner's
peers and superiors clearly showed that he was deficient in a lot
of areas, in addition to the fact that at the time of his nonrenewal, he was not even board-certified.
- As respondent CSC has correctly pointed out, the appointment
was for a definite and renewable period which, when it was not
renewed, did not involve a dismissal but an expiration of the
petitioners term.
*On estoppel by laches:
- Public policy and convenience demand that any claim to any
position in the civil service, permanent, temporary or otherwise,
or any claim to a violation of the constitutional provision on
security of tenure be made within a reasonable period of time.
The failure to assert a claim or the voluntary acceptance of
another position in government, obviously without reservation,
leads to a presumption that the civil servant has either given up
his claim or has already settled into the new position. This is
the essence of laches which is the failure or neglect, for an
unreasonable and unexplained length of time to do that which,
by exercising due diligence, could or should have been done
earlier; it is the negligence or omission to assert a right within a
reasonable time, warranting a presumption that the party
entitled to assert it either has abandoned it or declined to
assert it.
- Petitioner made no attempt to oppose earlier renewals of his
temporary Specialist 1 contracts, clearly demonstrating his
acquiescence to - if not his unqualified acceptance of - the
promotion (albeit of a temporary nature). Whatever objections
petitioner had against the earlier change from the status of
permanent senior resident physician to temporary senior
resident physician were neither pursued nor mentioned at or
after his designation as Medical Specialist 1 (Temporary). He is
therefore estopped from insisting upon a right or claim which he
had plainly abandoned when he, from all indications,
enthusiastically accepted the promotion. His negligence to
assert his claim within a reasonable time, coupled with his
failure to repudiate his promotion to a temporary position,
warrants a presumption that he either abandoned (his claim) or
declined to assert it.
Disposition Petition dismissed for lack of merit

R TRANSPORT CORP V EJANDRA


428 SCRA 724
CORONA; May 20, 2004
NATURE
Petition for review of the decision of the CFI of Iloilo
FACTS
- Rogelio Ejandra worked for petitioner bus company as a driver.
- On Jan 31 1996, he was apprehended for obstruction of traffic.
His license was confiscated. He reported this to his manager,
Oscar Pasquin, who gave him P500 to redeem the license. He
was able to retrieve the license after a week since the
apprehending officer turned it in only then.
- On feb 8, 1996, he reported for work. The company said they
were reviewing if they were going to allow him drive again.
Also, he was being blamed for damage to the bus. Ejandra said
the bus was damaged during the week he wasnt able to drive.

Labor Law 1
- Petitioner, on the other hand, claims that Ejandra is a habitual
absentee and has abandoned his job. To belie private
respondents allegation that his license had been confiscated,
petitioner asserted that, had it been true, he should have
presented an apprehension report and informed petitioner of
his problems with the LTO. But he did not. Petitioner further
argued that private respondent was not an employee because
theirs was a contract of lease and not of employment, with
petitioner being paid on commission basis
- The labor arbiter ruled in favor of Ejandra. It was held that he
didnt abandon his work, since there was valid reason for his 1
week absence. He also was not afforded due process. NLRC and
CA affirmed.
ISSUES
1. WON there was an employee employer relationship
2. WON Ejandra was dismissed for a just cause
HELD
1. YES
- Petitioner is barred to negate the existence of an employeremployee relationship. He has invoked rulings on the right of an
employer to dismiss an employee for just cause. The power to
dismiss an employee is one of the indications that there was
such relationship. Also, A97 of the Labor Code says that
employees can be paid in form of commissions.
2. NO
- To constitute abandonment, two elements must concur: (1)
the failure to report for work or absence without valid or
justifiable reason and (2) a clear intention to sever the
employer-employee relationship. Petitioner did not fulfill the
requisites. First, Ejandras absence was justified since his
license wasnt release until after a week. Second, Ejandra did
not want to sever their relationship when he got his license
back. Third, labor arbiter Yulo correctly observed that, if private
respondent really abandoned his work, petitioner should have
reported such fact to the nearest Regional Office of the
Department of Labor and Employment in accordance with
Section 7, Rule XXIII, Book V of Department Order No. 9, series
of 1997 (Rules Implementing Book V of the Labor Code).
Petitioner made no such report.
- In addition, he wasnt also given due process by not giving him
notice and hearing.
Disposition Decision reversed

SONZA V ABS-CBN
[PAGE 42]
INSULAR LIFE V NLRC (BASIAO)
179 SCRA 459
NARVASA; November 15, 1989
NATURE
Petition for certiorari and prohibition to review the resolution of
the NLRC.
FACTS
- In 1968, Insular Life Assurance Co., Ltd. (Company) and
Melecio T. Basiao entered into a contract by w/c Basiao was
"authorized to solicit w/in the Phils applications for insurance
policies and annuities in accordance with the existing rules and
regulations" of the Company; he would receive "compensation,
in the form of commissions . . . ", and the "rules in Rate Book
and its Agent's Manual, as well as all its circulars and those
which may from time to time be promulgated by it . . ." were
made part of said contract.
- The contract also contained provisions governing the relations
of the parties, the duties of the Agent, the acts prohibited to
him, and the modes of termination of the agreement, viz.:
"RELATION WITH THE COMPANY. The Agent shall be free to
exercise his own judgment as to time, place and means of

A2010

- 62 -

Disini

soliciting insurance. Nothing herein contained shall therefore


be construed to create the relationship of employee and
employer between the Agent and the Company. However, the
Agent shall observe and conform to all rules and regulations
which the Company may from time to time prescribe.
"TERMINATION. The Company may terminate the contract at
will, without any previous notice to the Agent, for or on
account of . . . (explicitly specified causes) . . .
- in April 1972, the parties entered into another contract - an
Agency Manager's Contract, while Basiao concurrently
fulfilled his commitments under the first contract with the
Company.
- In May 1979, the Company terminated the Agency Manager's
Contract. After vainly seeking a reconsideration, Basiao sued
the Company in a civil action and this (he claimed) prompted
the latter to terminate also his engagement under the first
contract and to stop payment of his commissions starting April
1, 1980.
- Basiao filed w/ the Ministry of Labor a complaint against the
Company and its president. The complaint sought to recover
commissions allegedly unpaid, plus attorney's fees. The
respondents claim: Ministry had no jurisdiction over Basiao's
claim, asserting that he was not the Company's employee, but
an independent contractor and that the Company had no
obligation to him for unpaid commissions under the terms and
conditions of his contract.
- The Labor Arbiter found for Basiao. He ruled that the
underwriting agreement had established an employer-employee
relationship between him and the Company, and this conferred
jurisdiction on the Ministry of Labor to adjudicate his claim. Said
official's decision directed payment of his unpaid commissions
". . . equivalent to the balance of the first year's premium
remaining unpaid, at the time of his termination, of all the
insurance policies solicited by . . . (him) in favor of the
respondent company . . ." plus 10% attorney's fees.
- This decision was, on appeal by the Company, affirmed by the
NLRC.
ISSUE
WON Basiao had become the Company's employee by virtue of
the contract invoked by him, thereby placing his claim for
unpaid commissions within the original and exclusive
jurisdiction of the Labor Arbiter under the provisions of Section
217 of the Labor Code
HELD
NO
- Basiao was not an employee of the petitioner, but a
commission agent, an independent contractor whose claim for
unpaid commissions should have been litigated in an ordinary
civil action. The Labor Arbiter erred in taking cognizance of, and
adjudicating, said claim, being without jurisdiction to do so, as
did the respondent NLRC in affirming the Arbiter's decision. This
conclusion renders it unnecessary and premature to consider
Basiao's claim for commissions on its merits.
-Control test" (Viana vs. Alejo Al-Lagadan, 1956):
"In determining the existence of employer-employee
relationship, the following elements are generally considered,
namely: (1) the selection and engagement of the employee;
(2) the payment of wages; (3) the power of dismissal; and
(4) the power to control the employees' conduct although the latter is the most important element (35 Am. Jur.
445). . . ,"
- However, not every form of control that the hiring party
reserves to himself over the conduct of the party hired in
relation to the services rendered may be accorded the effect of
establishing an employer-employee relationship between them
in the legal or technical sense of the term.
- Logically, the line should be drawn between rules that merely
serve as guidelines towards the achievement of the mutually
desired result without dictating the means or methods to be
employed in attaining it, and those that control or fix the

Labor Law 1
methodology and bind or restrict the party hired to the use of
such means. The first, which aim only to promote the result,
create no employer-employee relationship unlike the second,
which address both the result and the means used to achieve it.
- Rules and regulations governing the conduct of the business
are provided for in the Insurance Code and enforced by the
Insurance Commissioner. It is, therefore, usual and expected for
an insurance company to promulgate a set of rules to guide its
commission agents in selling its policies that they may not run
afoul of the law and what it requires or prohibits. Of such a
character are the rules which prescribe the qualifications of
persons who may be insured, subject insurance applications to
processing and approval by the Company, and also reserve to
the Company the determination of the premiums to be paid and
the schedules of payment. None of these really invades the
agent's contractual prerogative to adopt his own selling
methods or to sell insurance at his own time and convenience,
hence cannot justifiably be said to establish an employeremployee relationship between him and the company.
- Mafinco Trading Corporation v Ople: a person engaged to
sell soft drinks for another, using a truck supplied by the latter,
but with the right to employ his own workers, sell according to
his own methods subject only to prearranged routes, observing
no working hours fixed by the other party and obliged to secure
his own licenses and defray his own selling expenses, all in
consideration of a peddler's discount given by the other party
for at least 250 cases of soft drinks sold daily, was not an
employee but an independent contractor.
- Investment Planning Corporation of the Philippines v
SSS: there was no employer-employee relationship between a
commission agent and an investment company, but that the
former was an independent contractor where said agent and
others similarly placed were: (a) paid compensation in the form
of commissions based on percentages of their sales, any
balance of commissions earned being payable to their legal
representatives in the event of death or registration; (b)
required to put up performance bonds; (c) subject to a set of
rules and regulations governing the performance of their duties
under the agreement with the company and termination of their
services for certain causes; (d) not required to report for work
at any time, nor to devote their time exclusively to working for
the company nor to submit a record of their activities, and who,
finally, shouldered their own selling and transportation
expenses.
- Sara v NLRC: one who had been engaged by a rice miller to
buy and sell rice and palay without compensation except a
certain percentage of what he was able to buy or sell, did work
at his own pleasure without any supervision or control on the
part of his principal and relied on his own resources in the
performance of his work, was a plain commission agent, an
independent contractor and not an employee.

ALMIREZ V INFINITE LOOP TECHNOLOGY


CORPORATION
481 SCRA 364
CARPIO-MORALES; January 31, 2006
FACTS
- Almirez was hired as a Refinery Senior Process Design
Engineer for a specific project by respondent Infinite Loop
Technology Corporation through its General Manager Rubino.
- September 30, 1999 Details were furnished to Almirez
regarding her designation in the company as well as the scope
of her services. The scope of the services was to commence on
October 18, 1999 and had a guarantee of 12 continuous
months.
> The senior process design engineer was to work together
with the Process Design Consultant in performing the scope
of the services which included preparing the process terms of
reference or basis of design for the BPSD Petroleum Refinery,
to review and revise as necessary the existing conceptual

A2010

- 63 -

Disini

process block diagram of the process flow scheme,


implement new process technologies, participate in
discussions, make recommendation reports to the company
management team, represent the company in meetings and
perform other related works.
> She was to be paid a professional fee of US$2,000 per
month (net of tax) to be 50/50 split in US dollars or equivalent
peso every 15th of the month. She also had other benefits
and bonuses along with equipment such as a laptop
computer.
- Almirez received a total of P77,000 the following amounts on
the dates indicated:
- 11-23-09 P20,000 (Salary for Nov. 1-15)
12-02-99 P8,000 (Salary for Nov. 15-30)
12-15-99 P2,000 (Full payment for Nov. 15-30
salary)
P10,000 (Salary for Dec. 1-15,)
1-17-00 P12,000 (Salary for January 1-15)
1-16-00 P12,500 (Salary for January 16-31)
1-20-00 P12,500 (Salary for January 1-15)
- Almirez then wrote a letter to the company, expressing her
disappointment because she was receiving less than expected.
She hdadexpected the amount to be net of taxes but she was
receiving less because of SSS deductions and tax deductions.
She asked that her SSS dues be not deducted from her salary
because she was voluntarily paying such obligations on her
own. She further stated that she was willing to render her
services at Infinite Loop based on the contract and that she was
willing to serve as technical consultant on other relevant
projects.
- Rabino said that Almirezs letter was different from what they
had previously agreed upon. According to him, the BPSD
project, like any other project, could be deferred and that since
the engineering design for the proposed project was not yet
available, it would be prudent to suspend the professional
services of Almirez as Senior Process Design Engineer effective
February 2007.
- Almirez, through counsel, wrote a letter to Rabino, asking that
she be properly compensated with the total amount of her
contract because the contract stated that her tenure would last
for 12 months but she had already been suspended by February
of 2000. Almirez also noted that she had been paid only
P74,229.17 which is way below the amount promised to her of
US$2,000 a month net of tax.
- Rabino responded by explaining to Almirez that the company
and its partner corporations were all experiencing financial
difficulties with their projects and asked her to bear with
them.
- December 12, 2000 Almirez filed a complaint against Infinite
Loop for breach of contract of employment. Infinite Loop
countered by saying that the NLRC had no jurisdiction to hear
the case because there was no employer-employee relationship
and the contract was one of services, not employment.
- The Labor Arbiter ruled that there was an existence of an
employer-employee relationship and ordered Infinite Loop to
pay Almirez US$24,000 in its peso equivalent less advances of
P77,000. Infinite Loop appealed to the NLRC but the appeal
was dismissed.
- The Court of Appeals found that the primary cause of Almirezs
action was that for a sum of money on account of an alleged
breach of contract to pay a professional fee. It held that there
was no employer-employee relationship so the NLRC and the
Labor Arbiter have no jurisdiction over the said case. Thus
Almirezs petition was dismissed.
ISSUE
WON there was
relationship
HELD
1. NO

an existence of an employer-employee

Labor Law 1
Ratio
Under the control test, an employer-employee
relationship exists where the person for whom the services are
performed reserves the right to control not only the end
achieved but also the manner and means to be used in
reaching the end.
Reasoning
- The Court has consistently held a four tier test to evaluate the
existence of an employer-employee relationship which include:
1) manner of selection of engagement, 2) payment of wages, 3)
presence or absence of power of dismissal and 4) presence or
absence of power of control.
- The last test is known as the control test and is regarded as
the most crucial and determinative indicator of the presence of
absence of an employer-employee relationship.
- There is no showing of a controlling power of Infinite Loop over
Almirez. They only specified what she needed to achieve but
now how she was go on about it.
- The company had hired her based on her expertise but the
company naturally had to appraised of the work progress.
- The deduction for SSS and tax do not bolster Almirezs
contention that there was an employee-employer relationship.
However, only one pay slip was issued (Januaryb 16-31, 2000)
and the rest were in cash vouchers. As such, the payslip cannot
be considered as proof of an employer-employee relationship.
- The use of the word salary is not determinative of such a
relationship either.
Salary is defined as remuneration for
services given. The contract details her salary and it serves
between the parties was the law governing them. But the
contract, as pointed out earlier, is bereft of proof of control of
Infinite Loop over Almirez.
Disposition Petition is denied for lack of merit with costs
against petitioner.

SEVILLA V CA (TOURIST WORLD SERVICES)


160 SCRA 171
SARMIENTO; April 15, 1998
NATURE
Appeal by certiorari
FACTS
- On the strength of a contract, Tourist World Service Inc. (TWS)
leased the premises belonging to Mrs. Segundina Noguera for
the formers use as a branch office. Lina Sevilla bound herself
solidarily liable with TWS for the prompt payment of the
monthly rentals thereon.
- When the branch office was opened, it was run by appellant
Sevilla payable to TWS by any airline for any fare brought in on
the efforts of Sevilla, 4% was to go to Sevilla and 3% was to be
withheld by TWS.
- TWS appears to have been informed that Sevilla was
connected with a rival firm, the Philippine Travel Bureau, and,
since the branch office was anyhow losing, the TWS considered
closing down its office.
- This was firmed up by two resolutions of the TWS board of
directors to abolish the office of the manager and VP of the
branch office and authorizing the corporate secretary to receive
the properties in the said branch office.
- The corporate secretary went to the branch office, and finding
the premises locked and being unable to contact Sevilla,
padlocked the premises to protect the interests of TWS.
- When neither Sevilla nor her employees could enter the locked
premises, she filed a complaint against TWS with a prayer for
the issuance of a mandatory preliminary injunction.
- The trial court dismissed the case holding that TWS, being the
true lessee, was within its prerogative to terminate the lease
and padlock the premises. It likewise found that Sevilla was a
mere employee of TWS and as such, was bound by the acts of
her employer.
- The CA affirmed. Hence this petition.

A2010

- 64 -

Disini

ISSUE
1. WON there was an employer-employee relation between
TWS and Sevilla
2. WON the padlocking of the premises by TWS without the
knowledge and consent of Sevilla entitled the latter to the relief
of damages prayed for
HELD
1. NO. It was a principal-agent relationship.
Ratio In this jurisdiction, there has been no uniform test to
determine the existence of an employer-employee relation. In
general, We have relied on the so-called right of control test,
where the person for whom the services are performed
reserves a right to control not only the end to be achieved but
also the means to be used in reaching such end. In addition, the
existing economic conditions prevailing between the
parties, like the inclusion of the employee in the payrolls, are
also considered in determining the existence of an employeremployee relationship.
Reasoning
- Sevilla was not subject to control by TWS either as to the
result of the enterprise or as to the means used in connection
therewith.
- Under the contract of lease, Sevilla bound herself in solidum
for the rental payments; an arrangement that would belie the
claims of a master-servant relationship for a true employee
cannot be made to part with his own money in pursuance of his
employers business, or otherwise assume liability thereof.
- Sevilla was not in the companys payroll. She retained 4% in
commissions from airline bookings, the remaining 3% going to
TWS. Unlike an employee who usually earns a fixed salary, she
earned compensation in fluctuating amounts depending on her
booking successes.
- The fact that Sevilla has been designated branch manager
does not make her, ergo, TWS employee. Employment is
determined by the right of control test and certain economic
parameters. Titles are weak indicators.
- When Sevilla agreed to man TWS Ermita branch office, she
did so pursuant to a contract of agency. It is the essence of this
contract that the agent renders services in representation or
on behalf of another. In the case at bar, Sevilla solicited airline
fares, but she did so for and on behalf of her principal, TWS.
2. YES
Ratio For its unwarranted revocation of the contact of agency,
TWS should be sentenced to pay damages.
Reasoning
- Sevilla had acquired a personal stake in the business itself,
and necessarily, in the equipment pertaining thereto.
- Sevilla was not a stranger to that contract of lease having
been explicitly named therein as third party in charge of rental
payments. She could not be ousted from possession summarily
as one would eject an interloper.
- The Court is satisfied with the chronicle of events, there was
indeed some malevolent design to put the petitioner Sevilla in a
bad light following the disclosures that she had worked for a
rival firm.
Disposition REVERSED.

INSULAR LIFE ASSURANCE CO LTD V NLRC (DELOS


REYES)
287 SCRA 476
BELLOSILLO; March 12, 1998
NATURE
Petition for review on certiorari of the decision of the NLRC
FACTS
- On 21 August 1992 petitioner entered into an agency contract
with respondent Pantaleon de los Reyes authorizing the latter to
solicit within the Philippines applications for life insurance and

Labor Law 1
annuities for which he would be paid compensation in the form
of commissions. The contract was prepared by petitioner in its
entirety and De los Reyes merely signed his conformity thereto.
It contained the stipulation that no employer-employee
relationship shall be created between the parties and that the
agent shall be free to exercise his own judgment as to time,
place and means of soliciting insurance. De los Reyes however
was prohibited by petitioner from working for any other life
insurance company, and violation of this stipulation was
sufficient ground for termination of the contract. Aside from
soliciting insurance for the petitioner, private respondent was
required to submit to the former all completed applications for
insurance within ninety (90) consecutive days, deliver policies,
receive and collect initial premiums and balances of first year
premiums, renewal premiums, deposits on applications and
payments on
policy loans. Private respondent was also bound to turn over to
the company immediately any and all sums of money collected
by him.
- On 1 March 1993 petitioner and private respondent entered
into another contract where the latter was appointed as Acting
Unit Manager under its office atthe Cebu DSO V. As such, the
duties and responsibilities of De los Reyes included the
recruitment, training, organization and development within his
designated territory of a sufficient number of qualified,
competent and trustworthy underwriters, and to supervise and
coordinate the sales efforts of the underwriters in the active
solicitation of new business and in the furtherance of the
agency's assigned goals. It was similarly provided in the
management contract that the relation of the acting unit
manager and/or the agents of his unit to the company shall be
that of independent contractor. If the appointment was
terminated for any reason other than for cause, the acting unit
manager would be reverted to agent status and assigned to any
unit. As in the previous agency contract, De los Reyes together
with his unit force was granted freedom to exercise judgment
as to time, place and means of soliciting insurance. Aside from
being granted override commissions, the acting unit manager
was given production bonus, development allowance and a unit
development financing scheme euphemistically termed
"financial assistance" consisting of payment to him of a free
portion of P300.00 per month and a validate portion of
P1,200.00. While the latter amount was deemed as an advance
against expected commissions, the former was not and would
be freely given to the unit manager by the company only upon
fulfillment by him of certain manpower and premium quota
requirements. The agents and underwriters recruited and
trained by the acting unit manager would be attached to the
unit but petitioner reserved the right to determine if such
assignment would be made or, for any reason, to reassign them
elsewhere. Aside from soliciting insurance, De los Reyes was
also expressly obliged to participate in the company's
conservation program, i.e., preservation and maintenance of
existing insurance policies, and to accept moneys duly
receipted on agent's receipts provided the same were turned
over to the company. As long as he was unit manager in an
acting capacity, De los Reyes was prohibited from working for
other life insurance companies or with the government. He
could not also accept a managerial or supervisory position in
any firm doing business in the Philippines without the written
consent of petitioner.
- Private respondent worked concurrently as agent and Acting
Unit Manager until he was notified by petitioner on 18
November 1993 that his services were terminated effective 18
December 1993. He filed a complaint before the Labor Arbiter
on the ground that he was illegally dismissed and that he was
not paid his salaries and separation pay.
ISSUE
WON there exist an employer-employee relationship between
petitioner and respondent

A2010

- 65 -

Disini

HELD
YES
- It is axiomatic that the existence of an employer-employee
relationship cannot be negated by expressly repudiating it in
the management contract and providing therein that the
"employee" is an independent contractor when the terms of the
agreement clearly show otherwise. For, the employment status
of a person is defined and prescribed by law and not by what
the parties say it should be. In determining the status of the
management contract, the "four-fold test" on employment
earlier mentioned has to be applied.
(a) selection and engagement of employee
> Petitioner contends that De los Reyes was ever required to go
through the pre-employment procedures and that the
probationary employment status was reserved only to
employees of petitioner. On this score, it insists that the first
requirement of selection and engagement of the employee was
not met. A look at the provisions of the contract shows that
private respondent was appointed as Acting Unit Manager only
upon recommendation of the District Manager. This indicates
that private respondent was hired by petitioner because of the
favorable endorsement of its duly authorized officer. But, this
approbation could only have been based on the performance of
De los Reyes as agent under the agency contract so that there
can be no other conclusion arrived under this premise than the
fact that the agency or underwriter phase of the relationship of
De los Reyes with petitioner was nothing more than a trial or
probationary period for his eventual appointment as Acting Unit
Manager of petitioner. Then, again, the very designation of the
appointment of private respondent as "acting" unit manager
obviously implies a temporary employment status which may
be made permanent only upon compliance with company
standards such as those enumerated under the management
contract.
(b) payment of wages
> Petitioner points out that respondent was compensated
strictly on commission basis, the amount of which was totally
dependent on his total output. But, the manager's contract,
speaks differently. It unquestionably demonstrate that the
performance requirement imposed on De los Reyes was
applicable quarterly while his entitlement to the free portion
(P300) and the validated portion (P1,200) was monthly starting
on the first month of the twelve (12) months of the
appointment. Thus, it has to be admitted that even before the
end of the first quarter and prior to the so-called quarterly
performance evaluation, private respondent was already
entitled to be paid both the free and validated portions of the
UDF every month because his production performance could
not be determined until after the lapse of the quarter involved.
This indicates quite clearly that the unit manager's quarterly
performance had no bearing at all on his entitlement at least to
the free portion of the UDF which for all intents and purposes
comprised the salary regularly paid to him by petitioner. Thus it
cannot be validly claimed that the financial assistance
consisting of the free portion of the UDF was purely dependent
on the premium production of the agent. Be that as it may, it is
worth considering that the payment of compensation by way of
commission does not militate against the conclusion that
private respondent was an employee of petitioner. Under Art.
97 of the Labor Code, "wage" shall mean "however designated,
capable of being expressed in terms of money, whether fixed or
ascertained on a time, task, price or commission basis . . . .".
(c) power of dismissal and power of control
> petitioner asserts that its termination of De los Reyes was but
an exercise of its inherent right as principal under the contracts
and that the rules and guidelines it set forth in the contract
cannot, by any stretch of the imagination, be deemed as an
exercise of control over the private respondent as these were
merely directives that fixed the desired result without dictating
the means or method to be employed in attaining it. The
management contract, however, prescribes reveals that the
company practically dictates the manner by which their jobs are

Labor Law 1
to be carried out particularly exclusivity of service, control of
assignments and removal of agents under private respondent's
unit, collection of premiums, furnishing of company facilities
and materials as well as capital described as Unit Development
Fund.
- These are but hallmarks of the management system in which
herein private respondent worked. This obtaining, there is no
escaping the conclusion that private respondent Pantaleon de
los Reyes was an employee of herein petitioner.
Disposition Petition denied.

CHAVEZ V NLRC (SUPREME PACKAGING INC, LEE)


448 SCRA 478
CALLEJO, SR; January 17, 2005
NATURE
Petition for review on certiorari of the Resolution[1] dated
December 15, 2000 of the Court of Appeals (CA) reversing its
Decision dated April 28, 2000 finding private respondents guilty
of illegal dismissal.
FACTS
- The respondent company, Supreme Packaging, Inc. engaged
the services of the petitioner, Pedro Chavez, as truck driver on
October 25, 1984. The respondent company furnished the
petitioner with a truck.
- Sometime in 1992, the petitioner expressed to respondent
Alvin Lee, respondent companys plant manager, his (the
petitioners) desire to avail himself of the benefits that the
regular employees were receiving such as overtime pay,
nightshift differential pay, and 13th month pay, among others.
Although he promised to extend these benefits to the
petitioner, respondent Lee failed to actually do so.
- On February 20, 1995, the petitioner filed a complaint for
regularization with the Regional Arbitration Branch No. III of the
NLRC in San Fernando, Pampanga. Before the case could be
heard, respondent company terminated the services of the
petitioner. Consequently, on May 25, 1995, the petitioner filed
an amended complaint against the respondents for illegal
dismissal, unfair labor practice and non-payment of overtime
pay, nightshift differential pay, 13th month pay, among others.
The case was docketed as NLRC Case No. RAB-III-02-6181-95.
- The respondents, for their part, denied the existence of an
employer-employee relationship between the respondent
company and the petitioner. They averred that the petitioner
was an independent contractor as evidenced by the contract of
service which he and the respondent company entered
into. The relationship of the respondent company and the
petitioner was allegedly governed by this contract of service.
- The respondents insisted that the petitioner had the sole
control over the means and methods by which his work was
accomplished. He paid the wages of his helpers and exercised
control over them. As such, the petitioner was not entitled to
regularization because he was not an employee of the
respondent company. The respondents, likewise, maintained
that they did not dismiss the petitioner. Rather, the severance
of his contractual relation with the respondent company was
due to his violation of the terms and conditions of their
contract.
ISSUE
WON there existed an employer-employee relationship between
the respondent company and the petitioner.
HELD
YES
- The elements to determine the existence of an employment
relationship are: (1) the selection and engagement of the
employee; (2) the payment of wages; (3) the power of
dismissal; and (4) the employers power to control the

A2010

- 66 -

Disini

employees conduct.[11] The most important element is the


employers control of the employees conduct, not only as to
the result of the work to be done, but also as to the means and
methods to accomplish it.[12] All the four elements are
present in this case.
- Of the four elements of the employer-employee relationship,
the control test is the most important. Although the
respondents denied that they exercised control over the
manner and methods by which the petitioner accomplished his
work, a careful review of the records shows that the latter
performed his work as truck driver under the respondents
supervision and control. Their right of control was manifested
by the following attendant circumstances:
1.
The truck driven by the petitioner belonged to
respondent company;
2.
There was an express instruction from the respondents
that the truck shall be used exclusively to deliver respondent
companys goods; [19]
3.
Respondents directed the petitioner, after completion
of each delivery, to park the truck in either of two specific
places only, to wit: at its office in Metro Manila at 2320
Osmea Street, Makati City or at BEPZ, Mariveles, Bataan;[20]
and
4.
Respondents determined how, where and when the
petitioner would perform his task by issuing to him gate
passes and routing slips. [21]
- These circumstances, to the Courts mind, prove that the
respondents exercised control over the means and methods by
which the petitioner accomplished his work as truck driver of
the respondent company.
- The contract of service indubitably established the existence
of an employer-employee relationship between the respondent
company and the petitioner. It bears stressing that the
existence of an employer-employee relationship cannot
be negated by expressly repudiating it in a contract and
providing therein that the employee is an independent
contractor when, as in this case, the facts clearly show
otherwise. Indeed, the employment status of a person is
defined and prescribed by law and not by what the
parties say it should be.[22]
- The employer-employee relationship being established, the
Court rules that private respondent is guilty of illegal dismissal.

SAN MIGUEL CORP V ABELLA


461 SCRA 392
CARPIO-MORALES; June 28 2005
NATURE
Special Civil Action in the Supreme Court. Certiorari
FACTS
- Petitioner San Miguel Corporation (SMC) and Sunflower MultiPurpose Cooperative (Sunflower), entered into a one-year
Contract of Services commencing on January 1, 1993, to be
renewed on a month to month basis until terminated by either
party. The pertinent provisions of the contract are:
1. The cooperative agrees and undertakes to perform and/or
provide for the company, on a non-exclusive basis for a
period of one year the following services for the Bacolod
Shrimp Processing Plant:
A. Messengerial/Janitorial
B. Shrimp Harvesting/Receiving
C. Sanitation/Washing/Cold Storage
4. There is no employer-employee relationship between the
company and the cooperative, or the cooperative and any of
its members, or the company and any members of the
cooperative. The cooperative is an association of selfemployed members, an independent contractor, and an
entrepreneur. It is subject to the control and direction of the
company only as to the result to be accomplished by the
work or services herein specified, and not as to the work

Labor Law 1
herein contracted. The cooperative and its members
recognize that it is taking a business risk in accepting a fixed
service fee to provide the services contracted for and its
realization of profit or loss from its undertaking, in relation to
all its other undertakings, will depend on how efficiently it
deploys and fields its members and how they perform the
work and manage its operations.
- Pursuant to the contract, Sunflower engaged private
respondents to, as they did, render services at SMCs Bacolod
Shrimp Processing Plant at Sta. Fe, Bacolod City. The contract
was deemed renewed by the parties every month after its
expiration on January 1, 1994 and private respondents
continued to perform their tasks until September 11, 1995. In
July 1995, private respondents filed a complaint before the
NLRC, Regional Arbitration Branch No. VI, Bacolod City, praying
to be declared as regular employees of SMC, with claims for
recovery of all benefits and privileges enjoyed by SMC rank and
file employees. Private respondents subsequently filed on
September 25, 1995 an Amended Complaint to include illegal
dismissal as additional cause of action following SMCs closure
of its Bacolod Shrimp Processing Plant on September 15,
1995which resulted in the termination of their services. SMC
filed a Motion for Leave to File Attached Third Party Complaint
dated November 27, 1995 to implead Sunflower as Third Party
Defendant which was, by Order of December 11, 1995, granted
by Labor Arbiter Ray Alan T. Drilon. In the meantime, on
September 30, 1996, SMC filed before the Regional Office at
Iloilo City of the Department of Labor and Employment (DOLE) a
Notice of Closure of its aquaculture operations effective on even
date, citing serious business losses. By Decision of September
23, 1997, Labor Arbiter Drilon dismissed private respondents
complaint for lack of merit.
- Private respondents appealed to the NLRC. By Decision of
December 29, 1998, the NLRC dismissed the appeal for lack of
merit, it finding that third party respondent Sunflower was an
independent contractor in light of its observation that [i]n all
the activities of private respondents, they were under the actual
direction, control and supervision of third party respondent
Sunflower, as well as the payment of wages, and power of
dismissal. By Decision of February 7, 2001, the appellate court
reversed the NLRC decision and accordingly found for private
respondents. Justifying its reversal of the findings of the labor
arbiter and the NLRC, the appellate court reasoned:Although
the terms of the non-exclusive contract of service between SMC
and [Sunflower] showed a clear intent to abstain from
establishing an employer-employee relationship between SMC
and [Sunflower] or the latters members, the extent to which
the parties successfully realized this intent in the light of the
applicable law is the controlling factor in determining the real
and actual relationship between or among the parties.There
being a finding of labor-only contracting, liability must be
shouldered either by SMC or [Sunflower] or shared by both (See
Tabas vs. California Manufacturing, Inc., supra, p. 502). SMC
however should be held solely liable for [Sunflower] became
non-existent with the closure of the aquaculture business of
SMC.
ISSUE
1. WON the respondents are employees of SMC
2. WON the retrenchment was valid and consequently, whether
the respondents are entitled to relief
HELD
1. YES
- Since private respondents who were engaged in shrimp
processing performed tasks usually necessary or desirable in
the aquaculture business of SMC, they should be deemed
regular employees of the latter and as such are entitled to all
the benefits and rights appurtenant to regular employment.
They should thus be awarded differential pay corresponding to
the difference between the wages and benefits given them and
those accorded SMCs other regular employees. Respecting the

A2010

- 67 -

Disini

private respondents who were tasked with janitorial and


messengerial duties, this Court quotes with approval the
appellate courts ruling thereon:
- Those performing janitorial and messengerial services however
acquired regular status only after rendering one-year service
pursuant to Article 280 of the Labor Code. Although janitorial
and messengerial services are considered directly related to the
aquaculture business of SMC, they are deemed unnecessary in
the conduct of its principal business; hence, the distinctionThe
law of course provides for two kinds of regular employees,
namely: (1) those who are engaged to perform activities which
are usually necessary or desirable in the usual business or trade
of the employer; and (2) those who have rendered at least one
year of service, whether continuous or broken, with respect to
the activity in which they are employed.
- The test to determine the existence of independent
contractorship is whether one claiming to be an independent
contractor has contracted to do the work according to his own
methods and without being subject to the control of the
employer, except only as to the results of the work. As for those
of private respondents who were engaged in janitorial and
messengerial tasks, they fall under the second category and are
thus entitled to differential pay and benefits extended to other
SMC regular employees from the day immediately following
their first year of service.
- In legitimate labor contracting, the law creates an employeremployee relationship for a limited purpose, i.e., to ensure that
the employees are paid their wages. The principal employer
becomes jointly and severally liable with the job contractor,
only for the payment of the employees wages whenever the
contractor fails to pay the same. Other than that, the principal
employer is not responsible for any claim made by the
employees.[50]
- In labor-only contracting, the statute creates an employeremployee relationship for a comprehensive purpose: to prevent
a circumvention of labor laws. The contractor is considered
merely an agent of the principal employer and the latter is
responsible to the employees of the labor-only contractor as if
such employees had been directly employed by the principal
employer.[51]
- The Contract of Services between SMC and Sunflower shows
that the parties clearly disavowed the existence of an
employer-employee relationship between SMC and private
respondents. The language of a contract is not, however,
determinative of the parties relationship; rather it is the totality
of the facts and surrounding circumstances of the case.[52] A
party cannot dictate, by the mere expedient of a unilateral
declaration in a contract, the character of its business, i.e.,
whether as labor-only contractor or job contractor, it being
crucial that its character be measured in terms of and
determined by the criteria set by statute
2. SMC has thus proven substantial business reverses justifying
retrenchment of its employees.
- In the case at bar, company losses were duly established by
financial documents audited by Joaquin Cunanan & Co. showing
that the aquaculture operations of SMCs Agribusiness Division
accumulated losses amounting to P145,848,172.00 in 1992
resulting in the closure of its Calatrava Aquaculture Center in
Negros Occidental, P11,393,071.00 in 1993 and P80,325,608.00
in 1994 which led to the closure of its San Fernando Shrimp
Processing Plant in Pampanga and the Bacolod Shrimp
Processing Plant in 1995. For termination due to retrenchment
to be valid, however, the law requires that written notices of the
intended retrenchment be served by the employer on the
worker and on the DOLE at least one (1) month before the
actual date of the retrenchment in order to give employees
some time to prepare for the eventual loss of their jobs, as well
as to give DOLE the opportunity to ascertain the verity of the
alleged cause of termination. Private respondents,
however, were merely verbally informed on September
10, 1995 by SMC Prawn Manager Ponciano Capay that
effective the following day or on September 11, 1995,

Labor Law 1
they were no longer to report for work as SMC would be
closing its operations. Where the dismissal is based on
an authorized cause under Article 283 of the Labor Code
but the employer failed to comply with the notice
requirement, the sanction should be stiff as the
dismissal process was initiated by the employers
exercise of his management prerogative, as opposed to
a dismissal based on a just cause under Article 282 with
the same procedural infirmity where the sanction to be
imposed upon the employer should be tempered as the
dismissal process was, in effect, initiated by an act
imputable to the employee. In light of the factual
circumstances of the case at bar, the Court awards P50,000.00
to each private respondent as nominal damages.The grant of
separation pay as an incidence of termination of employment
due to retrenchment to prevent losses is a statutory obligation
on the part of the employer and a demandable right on the part
of the employee. Private respondents should thus be awarded
separation pay equivalent to at least one (1) month pay or to at
least one-half month pay for every year of service, whichever is
higher, as mandated by Article 283 of the Labor Code or the
separation pay awarded by SMC to other regular SMC
employees that were terminated as a result of the
retrenchment, depending on which is most beneficial to private
respondents.Considering that private respondents were not
illegally dismissed, however, no backwages need be awarded.
It is well settled that backwages may be granted only when
there is a finding of illegal dismissal.[80] The appellate court thus
erred in awarding backwages to private respondents. What was
involved in that case was one of illegal dismissal

LOPEZ V METROPOLITAN WATERWORKS AND


SEWERAGE SYSTEM
462 SCRA 428
TINGA; June 30, 2005
NATURE
Petition for the review of the decision of the CA
FACTS
- By virtue of an Agreement, petitioners were engaged by the
MWSS as collectors-contractors, wherein the former agreed to
collect from the concessionaires of MWSS, charges, fees,
assessments of rents for water, sewer and/or plumbing services
which the MWSS bills from time to time.
- In 1997, MWSS entered into a Concession Agreement with
Manila Water Service, Inc. and Benpress-Lyonnaise, wherein the
collection
of bills
was
transferred
to
said
private
concessionaires, effectively terminating the contracts of service
between petitioners and MWSS.
- Regular employees of the MWSS were paid their retirement
benefits, but not petitioners. Instead, they were refused said
benefits, MWSS relying on a resolution of the CSC that contractcollectors of the MWSS are not its employees and therefore not
entitled to the benefits due regular government employees.
- Petitioners filed a complaint with the CSC which denied their
claims, stating that petitioners were engaged by MWSS through
a contract of service, which explicitly provides that a bill
collector-contractor is not an MWSS employee. Relying on Part
V of CSC Memorandum Circular No. 38, Series of 1993, the CSC
stated that contract services/job orders are not considered
government services, which do not have to be submitted to the
CSC for approval, unlike contractual and plantilla appointments.
Moreover, it found that petitioners were unable to show that
they have contractual appointments duly attested by the CSC.
In addition, the CSC stated that petitioners, not being
permanent employees of MWSS and not included in the list
submitted to the concessionaire, are not entitled to severance
pay. Petitioners claims for retirement benefits and terminal
leave pay were likewise denied.

A2010

- 68 -

Disini

- Petitioners sought reconsideration of the CSC Resolution,


which was however denied
- Petitioners filed a petition for review with the Court of Appeals
which affirmed the ruling of the CSC.
ISSUE
WON petitioners were employees of the
consequently, entitled to the benefits they claim

MWSS

and,

HELD
YES
- The Court has invariably affirmed that it will not hesitate to tilt
the scales of justice to the labor class for no less than the
Constitution dictates that the State . . . shall protect the rights
of workers and promote their welfare. It is committed to this
policy and has always been quick to rise to defense in the rights
of labor, as in this case.
- Protection to labor, it has been said, extends to all of
labor local and overseas, organized and unorganized, in the
public and private sectors.[52] Besides, there is no reason not to
apply this principle in favor of workers in the government. The
government, including government-owned and controlled
corporations, as employers, should set the example in
upholding the rights and interests of the working class.
- For purposes of determining the existence of employeremployee relationship, the Court has consistently adhered to
the four-fold test, namely: (1) whether the alleged employer has
the power of selection and engagement of an employee; (2)
whether he has control of the employee with respect to the
means and methods by which work is to be accomplished; (3)
whether he has the power to dismiss; and (4) whether the
employee was paid wages. Of the four, the control test is the
most important element.
- A review of the circumstances surrounding the case reveals
that petitioners are employees of MWSS. MWSS wielded its
power of selection when it contracted with the individual
petitioners, undertaking separate contracts or agreements. The
same goes true for the power to dismiss. Although termed as
causes for termination of the Agreement, a review of the same
shows that the grounds indicated therein can similarly be
grounds for termination of employment.
- On the issue of remuneration, MWSS claims that the
compensation received by petitioners does not fall under the
definition of wages as provided in Section 2(i) of P.D. 1146. This
assertion, however, simply begs the question. The provision is a
simple statement of meaning, operating on the a priori premise
or presumption that the recipient is already classified as an
employee, and does not lay down any basis or standard for
determining who are employees and who are not.
- On the other hand, relevant and appropriate is the definition of
wages in the Labor Code, namely, that it is the remuneration,
however designated, for work done or to be done, or for
services rendered or to be rendered. The commissions due
petitioners were based on the bills collected as per the schedule
indicated in the Agreement. Significantly, MWSS granted
petitioners benefits usually given to employees, to wit: COLA,
meal, emergency, and traveling allowances, hazard pay, cash
gift, and other bonuses. Petitioners rendered services to MWSS
for which they were paid and given similar benefits due the
other employees of MWSS.
- Now the aspect of control. MWSS makes an issue out of the
proviso in the Agreement that specifically denies the existence
of employer-employee relationship between it and petitioners. It
is axiomatic that the existence of an employer-employee
relationship cannot be negated by expressly repudiating it in an
agreement and providing therein that the employee is not an
MWSS employee when the terms of the agreement and the
surrounding circumstances show otherwise. The employment
status of a person is defined and prescribed by law and not by
what the parties say it should be.
- In addition, the control test merely calls for the existence of
the right to control, and not the exercise thereof. It is not

Labor Law 1
essential for the employer to actually supervise the
performance of duties of the employee, it is enough that the
former has a right to wield the power.
- Other manifestations of control are evident from the records.
The power to transfer or reassign employees is a management
prerogative exclusively enjoyed by employers. In this case,
MWSS had free reign over the transfer of bill collectors from one
branch to another. MWSS also monitored the performance of
the petitioners and determined their efficiency ratings.
Disposition Petition was GRANTED IN PART. The Decision of
the Court of Appeals in C.A.G.R. SP No. 55263, as well as the
Civil Service Commissions Resolutions Nos. 991384 and
992074, were REVERSED and SET ASIDE. MWSS is ordered to
pay terminal leave pay and separation pay and/or severance
pay to each of herein petitioners on the basis of
remunerations/commissions, allowances and bonuses each
were actually receiving at the time of termination of their
employment as contract collectors of MWSS. The case was
remanded to the Civil Service Commission for the computation
of the above awards and the appropriate disposition in
accordance with the pronouncements in this Decision.

LAZARO V SSS (LAUDATO)


435 SCRA 472
TINGA; July 30, 2004
NATURE
Petition for Review under ROC Rule 45, assailing the CA
Decision, which affirmed two rulings of the Social Security
Commission (SSC)
FACTS
- Private respondent Rosalina M. Laudato filed a petition before
the SSC for social security coverage and remittance of unpaid
monthly social security contributions against her three
employers. Among the respondents was herein petitioner
Angelito L. Lazaro, proprietor of Royal Star Marketing, which is
engaged in the business of selling home appliances. Laudato
alleged that despite her employment as sales supervisor of the
sales agents for Royal Star from April of 1979 to March of 1986,
Lazaro had failed during the said period, to report her to the
SSC for compulsory coverage or remit Laudato's social security
contributions.
- Lazaro denied that Laudato was a sales supervisor of Royal
Star, averring instead that she was a mere sales agent whom
he paid purely on commission basis. Lazaro also maintained
that Laudato was not subjected to definite hours and conditions
of work. As such, Laudato could not be deemed an employee of
Royal Star.
- SSC ruled in favor of Laudato. Applying the "control test," it
held that Laudato was an employee of Royal Star, and ordered
Royal Star to pay the unremitted social security contributions of
Laudato in the amount of P5,007.35, together with the penalties
totaling P22,218.54. In addition, Royal Star was made liable to
pay damages to the SSC in the amount of P15,680.07 for not
reporting Laudato for social security coverage, pursuant to
Section 24 of the Social Security Law. Lazaro's MR was denied,
prompting him to file a petition for review with the CA.
However, the CA affirmed the finding that Laudato was an
employee of Royal Star, and hence entitled to coverage under
the Social Security Law.
- Lazaro's Argument: that Laudato was not qualified for social
security coverage, as she was not an employee of Royal Star,
her income dependent on a generation of sales and based on
commissions; that Royal Star had no control over Laudato's
activities, and that under the so-called "control test," Laudato
could not be deemed an employee.
ISSUE
WON Laudato is an employee of Royal Star

A2010

- 69 -

Disini

HELD
YES
Doctrine For the purposes of coverage under the Social
Security Act, the determination of employer-employee
relationship warrants the application of the "control test," that
is, whether the employer controls or has reserved the right to
control the employee, not only as to the result of the work
done, but also as to the means and methods by which the same
is accomplished.
- The fact that Laudato was paid by way of commission does not
preclude the establishment of an employer-employee
relationship. In Grepalife v. Judico, the Court upheld the
existence of an employer-employee relationship between the
insurance company and its agents, despite the fact that the
compensation that the agents on commission received was not
paid by the company but by the investor or the person insured.
- Neither does it follow that a person who does not observe
normal hours of work cannot be deemed an employee. In
Cosmopolitan Funeral Homes, Inc. v. Maalat, the Supreme Court
declared that there was an employer-employee relationship,
noting that "[the] supervisor, although compensated on
commission basis, [is] exempt from the observance of normal
hours of work for his compensation is measured by the number
of sales he makes.
- The determination of an employer-employee relationship
depends heavily on the particular factual circumstances
attending the professional interaction of the parties. SC sees no
reversible error in the findings of fact of the courts below. Both
SSC and CA found that Laudato was a sales supervisor and not
a mere agent. As such, Laudato oversaw and supervised the
sales agents of the company, and thus was subject to the
control of management as to how she implements its policies
and its end results. This is proven by several documentary
evidence.
Disposition Petition is DENIED. CA Decision AFFIRMED. Costs
against petitioner.

ALMIREZ V INFINITE LOOP TECHNOLOGY


CORPORATION
[PAGE 57]
LAZARO V SSS (LAUDATO)
[PAGE 61]
DOMASIG V NLRC (CATA GARMENTS)
261 SCRA 779 (96)
PADILLA; September 16, 1996
NATURE
Petition for certiorari under Rule 65 of the Rules of Court to
nullify and set aside the Resolution of respondent National
Labor Relations Commission remanding the records of the case
to the arbitration branch of origin for further proceedings.

FACTS
- Complaint was instituted by Eddie Domasig against
respondents Cata Garments Corporation, a company engaged
in garments business and its owner/manager Otto Ong and
Catalina Co for illegal dismissal, unpaid commission and other
monetary claim(s).
- Complainant alleged that he started working with the
respondent on July 6, 1986 as Salesman; three (3) years ago,
because of a complaint against respondent by its workers, the
company changed its name to Cata Garments Corporation; and
that on August 29, 1992, he was dismissed when respondent

Labor Law 1
learned that he was being pirated by a rival corporation which
offer he refused.
- The Labor Arbiter held that complainant was illegally
dismissed and entitled to reinstatement and backwages as well
as underpayment of salary; 13th month pay; service incentive
leave and legal holiday. The Arbiter also awarded complainant
his claim for unpaid commission in the amount of P143,955.00.
- NLRC remanded the case for further proceedings.
- Petitioners Claim
> Petitioner claims he was an employee, and that he was
illegally dismissed.
- Respondents Comments
> Respondents claim that Domasig was a mere commission
worker, and not a regular employee (which would warrant
backwages).
ISSUE
WON Domasig is a regular employee (this case is under the
topic of proof of employment)
HELD
YES, Domasig is a regular employee.
Ratio Substantial evidence is sufficient as a basis for judgment
on the existence of employer-employee relationship.
Reasoning
- Proof beyond reasonable doubt is not required as a basis for
judgment on the legality of an employers dismissal of an
employee, nor even preponderance of evidence for that matter,
substantial evidence being sufficient. Any competent and
relevant evidence to prove the relationship may be admitted.
- Substantial evidence
> relevant evidence as a reasonable mind might accept as
adequate to support a conclusion, and its absence is not shown
by stressing that there is contrary evidence on record, direct or
circumstantial, for the appellate court cannot substitute its own
judgment or criterion for that of the trial court in determining
wherein lies the weight of evidence or what evidence is entitled
to belief.
> In a business establishment, an identification card is usually
provided not only as a security measure but mainly to identify
the holder thereof as a bona fide employee of the firm that
issues it. Together with the cash vouchers covering petitioners
salaries for the months stated therein, these matters constitute
substantial evidence adequate to support a conclusion that
petitioner was indeed an employee of private respondent.
> The list presented by private respondents would even support
petitioners allegation that, aside from a monthly salary of
P1,500.00, he also received commissions for his work as a
salesman of private respondents.
- Having been in the employ of private respondents
continuously for more than one year, under the law, petitioner
is considered a regular employee.
Disposition The decision of the labor arbiter dated 19 May
1993 is REINSTATED and AFFIRMED.

ABANTE V LAMADRID
430 SCRA 368
YNARES-SANTIAGO; May 28, 2004
NATURE
Petition for review assailing the Decision of the CA which
affirmed the Resolution of the NLRC
FACTS
- Petitioner was employed by respondent company Lamadrid
Bearing and Parts Corporation sometime in June 1985 as a
salesman covering the whole area of Mindanao. His average
monthly income was more or less P16,000.00, but later was
increased to approximately P20,269.50. Aside from selling the
merchandise of respondent corporation, he was also tasked to
collect payments from his various customers. Sometime in

A2010

- 70 -

Disini

1998, petitioner encountered five customers/clients with bad


accounts.
- Petitioner was confronted by respondent Lamadrid over the
bad accounts and warned that if he does not issue his own
checks to cover the said bad accounts, his commissions will not
be released and he will lose his job. Not contented with the
issuance of the foregoing checks as security for the bad
accounts, respondents "tricked" petitioner into signing two
documents, which he later discovered to be a Promissory Note
and a Deed of Real Estate Mortgage.
- Due to financial difficulties, petitioner inquired about his
membership with the SSS in order to apply for a salary loan. To
his dismay, he learned that he was not covered by the SSS and
therefore was not entitled to any benefit. While doing his usual
rounds as commission salesman, petitioner was handed by his
customers a letter from the respondent company warning them
not to deal with petitioner since it no longer recognized him as
a commission salesman. Petitioner thus filed a complaint for
illegal dismissal with money claims against respondent
company and its president, Jose Lamadrid, before the NLRC.
ISSUE
1. WON an employer-employee relationship exists between
plaintiff and respondent company
2. WON respondent intimidated and tricked plaintiff into
providing security for the bad accounts
HELD
1. NO
Ratio To ascertain the existence of an employer-employee
relationship, jurisprudence has invariably applied the four-fold
test, namely: (1) the manner of selection and engagement; (2)
the payment of wages; (3) the presence or absence of the
power of dismissal; and (4) the presence or absence of the
power of control. Of these four, the last one is the most
important. Under the control test, an employer-employee
relationship exists where the person for whom the services are
performed reserves the right to control not only the end
achieved, but also the manner and means to be used in
reaching that end. Where a person who works for another does
so more or less at his own pleasure and is not subject to definite
hours or conditions of work, and in turn is compensated
according to the result of his efforts and not the amount
thereof, no relationship of employer-employee exists.
Reasoning
- Petitioner Abante was a commission salesman who received
3% commission of his gross sales. No quota was imposed on
him by the respondent. He was not required to report to the
office at any time or submit any periodic written report on his
sales performance and activities. He was not designated by
respondent to conduct his sales activities at any particular or
specific place. He pursued his selling activities without
interference or supervision from respondent company and
relied on his own resources to perform his functions.
Respondent company did not prescribe the manner of selling
the merchandise; he was left alone to adopt any style or
strategy to entice his customers. Moreover, petitioner was free
to offer his services to other companies engaged in similar or
related marketing activities as evidenced by the certifications
issued by various customers.
2. NO
Ratio While petitioner may have been coerced into executing
force to issue the said documents, it may equally be true that
petitioner did so in recognition of a valid financial obligation. He
who claims that force or intimidation was employed upon him
lies the onus probandi. He who asserts must prove.
Disposition The decision of the CA is AFFIRMED in toto.

R TRANSPORT CORP V EJANDRA


[PAGE 55]
MANILA ELECTRIC COMPANY V QUISUMBING

Labor Law 1
[PAGE 19]
MANILA ELECTRIC CO V BENAMIRA
302 SCRA 173
AUSTRIA-MARTINEZ; July 14, 2005
NATURE
Petition for review on certiorari of the Court of Appeals decision
FACTS
- The individual respondents are licensed security guards
formerly employed by Peoples Security, Inc. (PSI) and deployed
as such at MERALCOs head office in Ortigas Avenue, Pasig,
Metro Manila. On November 30, 1990, the security service
agreement between PSI and MERALCO was terminated.
Immediately thereafter, fifty-six of PSIs security guards,
including herein eight individual respondents, filed a complaint
for unpaid monetary benefits against PSI and MERALCO.
Meanwhile, the security service agreement between respondent
Armed Security & Detective Agency, Inc., (ASDAI) and MERALCO
took effect on December 1, 1990. In the agreement, ASDAI was
designated as the AGENCY while MERALCO was designated as
the COMPANY.
- Subsequently, the individual respondents were absorbed by
ASDAI and retained at MERALCOs head office.
- Asuncion rendered a decision in NLRC-NCR Case No. 0502746-90 in favor of the former PSI security guards, including
the individual respondents.
- Less than a month later, or on July 21, 1992, the individual
respondents filed another complaint for unpaid monetary
benefits, this time against ASDAI and MERALCO.
- On July 25, 1992, the security service agreement between
respondent Advance Forces Security & Investigation Services,
Inc. (AFSISI) and MERALCO took effect, terminating the previous
security service agreement with ASDAI. Except as to the
number of security guards, the amount to be paid the agency,
and the effectivity of the agreement, the terms and conditions
were substantially identical with the security service agreement
with ASDAI.
- The individual respondents amended their complaint to
implead AFSISI as party respondent. They again amended their
complaint to allege that AFSISI terminated their services on
August 6, 1992 without notice and just cause and therefore
guilty of illegal dismissal.
- The individual respondents alleged that: MERALCO and ASDAI
never paid their overtime pay, service incentive leave pay,
premium pay for Sundays and Holidays, P50.00 monthly
uniform allowance and underpaid their 13th month pay; on July
24, 1992, when the security service agreement of ASDAI was
terminated and AFSISI took over the security functions of the
former on July 25, 1992, respondent security guard Benamira
was no longer given any work assignment when AFSISI learned
that the former has a pending case against PSI, in effect,
dismissing him from the service without just cause; and, the
rest of the individual respondents were absorbed by AFSISI but
were not given any assignments, thereby dismissing them from
the service without just cause.
- ASDAI denied in general terms any liability for the claims of
the individual respondents, claiming that there is nothing due
them in connection with their services.
- On the other hand, MERALCO denied liability on the ground of
lack of employer-employee relationship with individual
respondents. It averred that the individual respondents are the
employees of the security agencies it contracted for security
services; and that it has no existing liability for the individual
respondents claims since said security agencies have been
fully paid for their services per their respective security service
agreement.
- For its part, AFSISI asserted that: it is not liable for illegal
dismissal since it did not absorb or hire the individual
respondents, the latter were merely hold-over guards from

A2010

- 71 -

Disini

ASDAI; it is not obliged to employ or absorb the security guards


of the agency it replaced since there is no provision in its
security service agreement with MERALCO or in law requiring it
to absorb and hire the guards of ASDAI as it has its own guards
duly trained to service its various clients.
- After the submission of their respective evidence and position
papers, Labor Arbiter Pablo C. Espiritu, Jr. rendered a Decision
holding ASDAI and MERALCO jointly and solidarily liable to the
monetary claims of individual respondents and dismissing the
complaint against AFSISI. Individual respondents partial appeal
assailed solely the Labor Arbiters declaration that ASDAI is
their employer. They insisted that AFSISI is the party liable for
their illegal dismissal and should be the party directed to
reinstate them.
For its part, MERALCO attributed grave abuse of discretion on
the part of the Labor Arbiter in failing to consider the absence of
employer-employee relationship between MERALCO and
individual respondents.
- On the other hand, ASDAI took exception from the Labor
Arbiters finding that it is the employer of the individual
respondents and therefore liable for the latters unpaid
monetary benefits.
- The NLRC affirmed in toto the decision of the Labor Arbiter.
The individual respondents filed a motion for partial
reconsideration but it was denied by the NLRC.
The individual respondents filed a petition for certiorari before
the SC. They insisted that they were absorbed by AFSISI and the
latter effected their termination without notice and just cause.
- After the submission of the responsive pleadings and
memoranda, we referred the petition, in accordance with St.
Martin Funeral Homes vs. NLRC,[15] to the CA which, on
September 27, 2000, modified the decision of the NLRC by
declaring MERALCO as the direct employer of the individual
respondents.
- The CA held that: MERALCO changed the security agency
manning its premises three times while engaging the services
of the same people, the individual respondents; MERALCO
employed a scheme of hiring guards through an agency and
periodically entering into service contract with one agency after
another in order to evade the security of tenure of individual
respondents; individual respondents are regular employees of
MERALCO since their services as security guards are usually
necessary or desirable in the usual business or trade of
MERALCO and they have been in the service of MERALCO for no
less than six years; an employer-employee relationship exists
between MERALCO and the individual respondents because: (a)
MERALCO had the final say in the selection and hiring of the
guards, as when its advice was proved to have carried weight in
AFSISIs decision not to absorb the individual respondents into
its workforce; (b) MERALCO paid the wages of individual
respondents through ASDAI and AFSISI; (c) MERALCOs
discretion on matters of dismissal of guards was given great
weight and even finality since the record shows that the
individual respondents were replaced upon the advice of
MERALCO; and, (d) MERALCO has the right, at any time, to
inspect the guards, to require without explanation the
replacement of any guard whose behavior, conduct or
appearance is not satisfactory and ASDAI and AFSISI cannot pull
out any security guard from MERALCO without the latters
consent; and, a labor-only contract existed between ASDAI and
AFSISI and MERALCO, such that MERALCO is guilty of illegal
dismissal without just cause and liable for reinstatement of
individual respondents to its workforce.
ISSUES
1. WON there existed an employer-employee relationship
2. WON individual respondents cannot be considered as regular
employees as the duties performed by them as security guards
are not necessary in the conduct of MERALCOs principal
business which is the distribution of electricity.
3. WON MERALCO has a liability over the dismissed guards

Labor Law 1
HELD
1. It is a settled rule that in the exercise of the Supreme Courts
power of review, the Court is not a trier of facts and does not
normally undertake the re-examination of the evidence
presented by the contending parties during the trial of the case
considering that the findings of facts of the CA are conclusive
and binding on the Court.
However, jurisprudence has
recognized several exceptions in which factual issues may be
resolved by this Court.
- In the present case, the existence of an employer-employee
relationship is a question of fact which is well within the
province of the CA. Nonetheless, given the reality that the CAs
findings are at odds to those of the NLRC, the Court is
constrained to look deeper into the attendant circumstances
obtaining in the present case, as appearing on record.
The individual respondents never alleged in their complaint in
the Labor Arbiter, in their appeal in the NLRC and even in their
petition for certiorari in the CA that MERALCO was their
employer. They have always advanced the theory that AFSISI is
their employer. A perusal of the records shows it was only in
their Memorandum in the CA that this thesis was presented and
discussed for the first time. We cannot ignore the fact that this
position of individual respondents runs contrary to their earlier
submission in their pleadings filed in the Labor Arbiter, NLRC
and even in the petition for certiorari in the CA that AFSISI is
their employer and liable for their termination. As the object of
the pleadings is to draw the lines of battle, so to speak,
between the litigants and to indicate fairly the nature of the
claims or defenses of both parties, a party cannot subsequently
take a position contrary to, or inconsistent, with his pleadings.
Moreover, it is a fundamental rule of procedure that higher
courts are precluded from entertaining matters neither alleged
in the pleadings nor raised during the proceedings below, but
ventilated for the first time only in a motion for reconsideration
or on appeal. The individual respondents are bound by their
submissions that AFSISI is their employer and they should not
be permitted to change their theory. Such a change of theory
cannot be tolerated on appeal, not due to the strict application
of procedural rules but as a matter of fairness. A change of
theory on appeal is objectionable because it is contrary to the
rules of fair play, justice and due process.
- Thus, the CA should not have considered the new theory
offered by the individual respondents in their memorandum.
- The present petition for review on certiorari is far from novel
and, in fact, not without precedence. We have ruled in Social
Security System vs. Court of Appeals that:
...The guards or watchmen render their services to private
respondent by allowing themselves to be assigned by said
respondent, which furnishes them arms and ammunition, to
guard and protect the properties and interests of private
respondent's clients, thus enabling that respondent to fulfill
its contractual obligations. Who the clients will be, and under
what terms and conditions the services will be rendered, are
matters determined not by the guards or watchmen, but by
private respondent. On the other hand, the client companies
have no hand in selecting who among the guards or
watchmen shall be assigned to them. It is private respondent
that issues assignment orders and instructions and exercises
control and supervision over the guards or watchmen, so
much so that if, for one reason or another, the client is
dissatisfied with the services of a particular guard, the client
cannot himself terminate the services of such guard, but has
to notify private respondent, which either substitutes him
with another or metes out to him disciplinary measures. That
in the course of a watchman's assignment the client
conceivably issues instructions to him, does not in the least
detract from the fact that private respondent is the employer
of said watchman, for in legal contemplation such instructions
carry no more weight than mere requests, the privity of
contract being between the client and private respondent, not
between the client and the guard or watchman. Corollarily,
such giving out of instructions inevitably spring from the

A2010

- 72 -

Disini

client's right predicated on the contract for services entered


into by it with private respondent.
- In the matter of compensation, there can be no question at all
that the guards or watchmen receive compensation from
private respondent and not from the companies or
establishments whose premises they are guarding. The fee
contracted for to be paid by the client is admittedly not equal to
the salary of a guard or watchman; such fee is arrived at
independently of the salary to which the guard or watchman is
entitled under his arrangements with private respondent.
- Neither does the petitioner have any power to dismiss the
security guards. In fact, We fail to see any evidence in the
record that it wielded such a power. It is true that it may
request the agency to change a particular guard. But this,
precisely, is proof that the power lies in the hands of the
agency.
- Since the petitioner has to deal with the agency, and not the
individual watchmen, on matters pertaining to the contracted
task, it stands to reason that the petitioner does not exercise
any power over the watchmen's conduct. Always, the agency
stands between the petitioner and the watchmen; and it is the
agency that is answerable to the petitioner for the conduct of its
guards.
- In this case, the terms and conditions embodied in the security
service agreement between MERALCO and ASDAI expressly
recognized ASDAI as the employer of individual respondents.
- Under the security service agreement, it was ASDAI which (a)
selected, engaged or hired and discharged the security guards;
(b) assigned them to MERALCO according to the number agreed
upon; (c) provided the uniform, firearms and ammunition,
nightsticks, flashlights, raincoats and other paraphernalia of the
security guards; (d) paid them salaries or wages; and, (e)
disciplined and supervised them or principally controlled their
conduct.
The agreement even explicitly provided that
[n]othing herein contained shall be understood to make the
security guards under this Agreement, employees of the
COMPANY, it being clearly understood that such security guards
shall be considered as they are, employees of the AGENCY
alone. Clearly, the individual respondents are the employees
of ASDAI.
- Needless to stress, for the power of control to be present, the
person for whom the services are rendered must reserve the
right to direct not only the end to be achieved but also the
means for reaching such end. Not all rules imposed by the
hiring party on the hired party indicate that the latter is an
employee of the former. Rules which serve as general
guidelines towards the achievement of the mutually desired
result are not indicative of the power of control.
- Verily, the security service agreements in the present case
provided that all specific instructions by MERALCO relating to
the discharge by the security guards of their duties shall be
directed to the agency and not directly to the individual
respondents. The individual respondents failed to show that the
rules of MERALCO controlled their performance.
- Moreover, ASDAI and AFSISI are not labor-only contractors.
There is labor only contract when the person acting as
contractor is considered merely as an agent or intermediary of
the principal who is responsible to the workers in the same
manner and to the same extent as if they had been directly
employed by him. On the other hand, job (independent)
contracting is present if the following conditions are met: (a)
the contractor carries on an independent business and
undertakes the contract work on his own account under his own
responsibility according to his own manner and method, free
from the control and direction of his employer or principal in all
matters connected with the performance of the work except to
the result thereof; and (b) the contractor has substantial capital
or investments in the form of tools, equipment, machineries,
work premises and other materials which are necessary in the
conduct of his business.[29] Given the above distinction and
the provisions of the security service agreements entered into

Labor Law 1
by petitioner with ASDAI and AFSISI, we are convinced that
ASDAI and AFSISI were engaged in job contracting.
2. YES
- The individual respondents can not be considered as regular
employees of the MERALCO for, although security services are
necessary and desirable to the business of MERALCO, it is not
directly related to its principal business and may even be
considered unnecessary in the conduct of MERALCOs principal
business, which is the distribution of electricity.
- Furthermore, the fact that the individual respondents filed
their claim for unpaid monetary benefits against ASDAI is a
clear indication that the individual respondents acknowledge
that ASDAI is their employer.
- We cannot give credence to individual respondents insistence
that they were absorbed by AFSISI when MERALCOs security
service agreement with ASDAI was terminated. The individual
respondents failed to present any evidence to confirm that
AFSISI absorbed them into its workforce. Thus, respondent
Benamira was not retained in his post at MERALCO since July
25, 1992 due to the termination of the security service
agreement of MERALCO with ASDAI. As for the rest of the
individual respondents, they retained their post only as holdover guards until the security guards of AFSISI took over their
post on August 6, 1992.
- In the present case, respondent Benamira has been offdetail for seventeen days while the rest of the individual
respondents have only been off- detail for five days when
they amended their complaint on August 11, 1992 to include
the charge of illegal dismissal. The inclusion of the charge of
illegal dismissal then was premature. Nonetheless, bearing in
mind that ASDAI simply stopped giving the individual
respondents any assignment and their inactivity clearly
persisted beyond the six-month period allowed by Article 286 of
the Labor Code, the individual respondents were, in effect,
constructively dismissed by ASDAI from employment, hence,
they should be reinstated.
3. YES, as an indirect employer.
- The fact that there is no actual and direct employer-employee
relationship between MERALCO and the individual respondents
does not exonerate MERALCO from liability as to the monetary
claims of the individual respondents.
When MERALCO
contracted for security services with ASDAI as the security
agency that hired individual respondents to work as guards for
it, MERALCO became an indirect employer of individual
respondents pursuant to Article 107 of the Labor Code, which
reads:
ART.
107. Indirect employer - The provisions of the
immediately preceding Article shall likewise apply to any
person, partnership, association or corporation which, not
being an employer, contracts with an independent contractor
for the performance of any work, task, job or project.
- When ASDAI as contractor failed to pay the individual
respondents, MERALCO as principal becomes jointly and
severally liable for the individual respondents wages, under
Articles 106 and 109 of the Labor Code, which provide:
ART.
106. Contractor or subcontractor. - Whenever an
employer enters into a contract with another person for the
performance of the former[s] work, the employees of the
contractor and of the latter[s] subcontractor, if any, shall be
paid in accordance with the provisions of this Code.
In the event that the contractor or subcontractor fails to pay
the wages of his employees in accordance with this Code, the
employer shall be jointly and severally liable with his
contractor or subcontractor to such employees to the extent
of the work performed under the contract, in the same
manner and extent that he is liable to employees directly
employed by him.
ART. 109. Solidary liability - The provisions of existing laws to
the contrary notwithstanding, every employer or indirect
employer shall be held responsible with his contractor or
subcontractor for any violation of any provision of this Code.
For purpose of determining the extent of their civil liability

A2010

- 73 -

Disini

under this Chapter, they shall be considered as direct


employers.
- ASDAI is held liable by virtue of its status as direct employer,
while MERALCO is deemed the indirect employer of the
individual respondents for the purpose of paying their wages in
the event of failure of ASDAI to pay them.
This statutory
scheme gives the workers the ample protection consonant
with labor and social justice provisions of the 1987 Constitution.
- However, as held in Mariveles Shipyard Corp. vs. Court of
Appeals, the solidary liability of MERALCO with that of ASDAI
does not preclude the application of Article 1217 of the Civil
Code on the right of reimbursement from his co-debtor by the
one who paid, which provides:
ART. 1217. Payment made by one of the solidary debtors
extinguishes the obligation. If two or more solidary debtors
offer to pay, the creditor may choose which offer to accept.
- He who made the payment may claim from his co-debtors only
the share which corresponds to each, with the interest for the
payment already made. If the payment is made before the debt
is due, no interest for the intervening period may be demanded.
When one of the solidary debtors cannot, because of his
insolvency, reimburse his share to the debtor paying the
obligation, such share shall be borne by all his co-debtors, in
proportion to the debt of each.
- ASDAI may not seek exculpation by claiming that MERALCOs
payments to it were inadequate for the individual respondents
lawful compensation. As an employer, ASDAI is charged with
knowledge of labor laws and the adequacy of the compensation
that it demands for contractual services is its principal concern
and not any others.[35]
Disposition
present petition is GRANTED.
The assailed
Decision, dated September 27, 2000, of the CA is REVERSED
and SET ASIDE. The Decision of the Labor Arbiter dated January
3, 1994 and the Resolution of the NLRC dated April 10, 1995 are
AFFIRMED with the MODIFICATION that the joint and solidary
liability of ASDAI and MERALCO to pay individual respondents
monetary claims for underpayment of actual regular hours and
overtime hours rendered, and premium pay for holiday and rest
day, as well as attorneys fees, shall be without prejudice to
MERALCOs right of reimbursement from ASDAI.

SAN MIGUEL CORP V ABELLA


[PAGE 59]

BIG AA MANUFACTURER V ANTONIO


484 SCRA 392
QUISUMBING; March 3, 2006
NATURE
Petition for review on certiorari of a decision of CA
FACTS
- Petitioner Big AA Manufacturer is a sole proprietorship
registered in the name of its proprietor, Enrico E. Alejo.
Respondents filed a complaint for illegal lay-off and illegal
deductions
- Respondents
> That as regular employees, they worked from 8:00 a.m. to
5:00 p.m. at petitioners premises using petitioners tools and
equipment and they received P250 per day. Eutiquio was
employed as carpenter-foreman from 1991-99; Jay as carpenter
from 1993-99; Felicisimo as carpenter from 1994-99; and
Leonardo, Sr. also as carpenter from 1997-99; That they were

Labor Law 1
dismissed without just cause and due process; hence, their
prayer for reinstatement and full backwages.
- Petitioner Big AA Manufacturer
> That it is a sole proprietorship registered in the name of
Enrico Alejo and engaged in manufacturing office furniture, but
it denied that respondents were its regular employees. It
claimed that Eutiquio Antonio was one of its independent
contractors who used the services of the other respondents. It
said that its independent contractors were paid by results and
were responsible for the salaries of their own workers.
Allegedly, there was no employer-employee relationship
between petitioner and respondents. But it allowed respondents
to use its facilities to meet job orders. It also denied that
respondents were laid-off by Big AA Manufacturer, since they
were project employees only. It added that since Eutiquio
Antonio had refused a job order of office tables, their
contractual relationship ended.
- Labor Arbiter ruled againstpetitioners. Both appealed to NLRC.
Respondents appealed for not ordering their reinstatement to
their former positions. The NLRC modified the Labor Arbiters
decision. It ordered petitioner to reinstate respondents to their
former positions or to pay them separation pay in case
reinstatement was no longer feasible, with full backwages in
either case. The NLRC ruled that respondents were regular
employees, not independent contractors. It further held that
petitioner failed to justify its reason for terminating respondents
and its failure to comply with the due process requirements. CA
affirmed NLRC ruling.
ISSUES
1. WON respondents were regular employees
2. WON respondents were illegally dismissed
HELD
1. YES
- Respondents were employed for more than 1 year and their
work as carpenters was necessary or desirable in petitioners
usual trade or business of manufacturing office furniture. Under
Art. 280 of the Labor Code, the applicable test to determine
whether an employment should be considered regular or nonregular is the reasonable connection between the particular
activity performed by the employee in relation to the usual
business or trade of the employer.
- True, certain forms of employment require the performance of
usual or desirable functions and exceed 1 year but do not
necessarily result to regular employment under Art. 280 of the
Labor Code. Some specific exceptions include project or
seasonal employment. Yet, in this case, respondents cannot be
considered project employees. Petitioner had neither shown
that respondents were hired for a specific project the duration
of which was determined at the time of their hiring nor
identified the specific project or phase thereof for which
respondents were hired.
Obiter on Requirements for an Independent contractor: a) he
carries a distinct and independent business, b) possesses
substantial capital or investment in tools, equipment,
machinery or work premises, c) he does not work within another
employer/companys premises using the latters tools and
materials, and d) he is not under the control and supervision of
an employer or company
2. YES
- The consistent rule is that the employer must affirmatively
show rationally adequate evidence that the dismissal was for a
justifiable cause, failing in which would make the termination
illegal, as in this case.
- Contrary to petitioners claim of abandonment as a valid just
cause for termination, herein respondents did not abandon their
work. Petitioner failed to prove that (1) not only of respondents
failure to report for work or absence without valid reason, but
(2) also of respondents clear intention to sever employeremployee relations as manifested by some overt acts.

A2010

- 74 -

Disini

- By filing the complaint for illegal dismissal within two days of


their dismissal and by seeking reinstatement in their position
paper, respondents manifested their intention against severing
their employment relationship with petitioner and abandoning
their jobs. It is settled that an employee who forthwith protests
his layoff cannot be said to have abandoned his work
Disposition Petition denied.

COCA-COLA OTTLERS PHILS INC V NLRC


(CANONICATO)
307 SCRA 131
BELLOSILLO; May 17, 1999
FACTS
- On April 7, 1986 Coca-Cola entered into a contract of janitorial
services with Bacolod Janitorial Services (BJS) as an
independent contractor.
- Private respondent Ramon Canonicato was hired as a janitor
by the Bacolod Janitorial Services (BJS). He was assigned at the
Coca Cola Bottlers, Inc. considering his familiarity with its
premises, having been previous casual employee there.
- Goaded by information that COCA COLA employed previous
BJS employees who filed a complaint against the company for
regularization pursuant to a compromise agreement,
Canonicato submitted a similar complaint against COCA COLA
to the Labor Arbiter on 8 June 1993 and consequently did not
report for work.
- On September 28,1993, BJS sent him a letter advising him to
report to work within 3 days from receipt, otherwise he would
be terminated.
- (there was no express mention of a termination but based on
the fact I assume he did not return to work and was terminated)
- On July 23, 1993, respondent filed with the Labor Arbiter a
complaint for illegal dismissal and underpayment of wages. He
included BJS therein as a co-respondent. The Labor Arbiter
dismissed the complaint and ruled that a) there was no
employer-employee relationship between Canonicato and Coca
Cola (b) BJS was a legitimate job contractor, hence, any liability
of COCA COLA as to Canonicato's salary or wage differentials
was solidary with BJS in accordance with pars. 1 and 2 of Art.
106, Labor Code; (c) COCA COLA and BJS must jointly and
severally pay Canonicato his wage differentials amounting to
P2,776.80 and his 13th month salary of P1,068.00, including ten
(10%) percent attorney's fees in the sum of P384.48.
- The NLRC rejected the decision of the Labor Arbiter on the
ground that the janitorial services of Canonicato were found to
be necessary in the usual trade of Coca Cola. In so holding,
NLRC applied Art.280 of the Labor Code and declared that
Canonito was a regular employee of Coca-Cola. Its motion for
reconsideration having been denied, Coca Cola filed this
petition.
ISSUE
WON Canonito was a regular employee of Coca-cola and thus
malking Coca-Cola liable for illegal dismissal
HELD
NO
- In Kimberly Independent Labor Union v. Drilon where the Court
took judicial notice of the practice adopted in several
government and private institutions and industries of hiring
janitorial services on an "independent contractor basis." In this
respect, although janitorial services may be considered directly
related to the principal business of an employer, as with every
business, we deemed them unnecessary in the conduct of the
employer's principal business.
- This judicial notice, of course, rests on the assumption that the
independent contractor is a legitimate job contractor so that
there can be no doubt as to the existence of an employeremployee relationship between contractor and the worker. In
this situation, the only pertinent question that may arise will no

Labor Law 1
longer deal with whether there exists an employment bond but
whether the employee may be considered regular or casual as
to deserve the application of Art. 280 of the Labor Code.
- It was error therefore for the NLRC to apply Art. 280 of the
Labor Code in determining the existence of an employment
relationship of the parties herein, especially in light of our
explicit holding in Singer Sewing Machine Company v. Drion
that The Court agrees with the petitioner's argument that Article
280 is not the yardstick for determining the existence of an
employment relationship because it merely distinguishes
between two kinds of employees, i.e., regular employees and
casual employees, for purposes of determining the right of an
employee to certain benefits, to join or form a union, or to
security of tenure. Article 280 does not apply where the
existence of an employment relationship is in dispute
- In determining the existence of an employer-employee
relationship it is necessary to determine whether the following
factors are present: (a) the selection and engagement of the
employee; (b) the payment of wages; (c) the power to dismiss;
and, (d) the power to control the employee's conduct. Notably,
these are all found in the relationship between BJS and
Canonicato and not between Canonicato and petitioner COCA
COLA. As the Solicitor-General manifested
- BJS satisfied all the requirements of a job-contractor under the
law, namely, (a) the ability to carry on an independent business
and undertake the contract work on its own account under its
own responsibility according to its manner and method, free
from the control and direction of its principal or client in all
matters connected with the performance of the work except as
to the results thereof; and, (b) the substantial capital or
investment in the form of tools, equipment, machinery, work
premises, and other materials which are necessary in the
conduct of its business.
- All told, there being no employer-employee relationship
between Canonicato and COCA COLA, the latter cannot be
validly ordered to reinstate the former and pay him back wages.

PAL V NLRC (STELLAR INDUSTRIAL SERVICES INC)


298 SCRA 430
PANGANIBAN; November 9, 1998
NATURE
Special civil action for certiorari, seeking to nullify the July 13,
1994 Decision and the June 27, 1996 Resolution of the National
Labor Relations Commission, which held Philippines Airlines, Inc.
liable for separation pay.
FACTS
- Sometime in 1977, PAL, a local air carrier, entered into a
service agreement with STELLAR, a domestic corporation
engaged, among others, in the business of job contracting
janitorial services. Pursuant to their service agreement, which
was impliedly renewed year after year, STELLAR hired workers
to perform janitorial and maintenance services for PAL. The
employees were assigned at PAL's various premises under the
supervision of STELLAR's supervisors/foremen and timekeepers.
The workers were also furnished by STELLAR with janitorial
supplies, such as vacuum cleaner and polisher.
- On December 31, 1990, the service agreement between PAL
and STELLAR expired. PAL then called for [the] bidding of its
janitorial requirements. This notwithstanding, STELLAR exerted
efforts to maintain its janitorial contract with PAL which, in the
meantime, allowed Manuel Parenas and others to work at the
PAL's premises.
- Subsequently, in a letter dated October 31, 1990, PAL formally
informed STELLAR that the service agreement would no longer
be renewed effective November 16, 1991, since PAL's janitorial
requirements were bidded to three other job contractors.
Alleging that they were illegally dismissed, the aforenamed
individual private respondents filed, from January to June 1992,

A2010

- 75 -

Disini

five complaints against PAL and STELLAR for illegal dismissal


and for payment of separation pay.
- Labor Arbiter Manuel P. Asuncion rendered on October 29,
1993 a Decision which held PAL liable for the separation pay of
terminated individual respondents.
- In its Decision affirming the ruling of the labor arbiter,
Respondent Commission held petitioner, as an indirect
employer, jointly and severally liable with STELLAR for
separation pay. First, the individual private respondent's work,
although not directly related to the business of petitioner, was
necessary and desirable for the maintenance of the petitioner's
premises and airplanes. Second, the individual private
respondents were retained for thirteen long years, despite the
fact that the contract, which petitioner had entered into
STELLAR in 1977, was only for one year.
On reconsideration, the NLRC modified its earlier Decision by
absolving STELLAR of liability, thereby making PAL solely
responsible for the award decreed by the labor arbiter. It held
that, first, petitioner was the employer of the individual private
respondents, for it engaged in labor-only contracting with
STELLAR. This was shown by the failure of petitioner to refute
the factual finding that it continued to employ the individual
private respondents after the expiration of the service contract
on December 31, 1990. Second, the individual private
respondents' admission in their Complaint that they were
employees of STELLAR was not conclusive, as the existence of
an employer-employee relation was a question of law that could
not be the subject of stipulation. Respondent Commission
concluded that their dismissal was without just and valid cause.
Because they were no longer seeking reinstatement, petitioner
was liable for separation pay.
ISSUES
1. WON the individual private respondents are
employees of PAL
2. WON petitioner is liable to them for separation pay

regular

HELD
1. No employer-employee relation between complainants and
petitioner.
Ratio
a) Janitorial service agreement is not labor-only
contacting AND
b) Extension of service contract is not a source of employeremployee relation.
Reasoning
a) Prohibited labor-only contracting is defined in Article 106 of
the Labor Code as follows:
There is "labor-only" contracting where the person supplying
workers to an employer does not have substantial capital or
investment in the form of tools, equipment, machineries, work
premises, among others, and the workers recruited and placed
by such persons are performing activities which are directly
related to the principal business of such employer. In such
cases, the person or intermediary shall be considered merely as
an agent of the employer who shall be responsible to the
workers in the same manner and extent as if the latter were
directly employed by him.
- This definition covers any person who undertakes to supply
workers to an employer, where such person:
(1) Does not have substantial capital or investment in the form
of tools, equipment, [machinery], work premises and other
materials; and
(2) The workers recruited and placed by such person are
performing activities which are directly related to the principal
business or operations of the employer in which workers are
habitually employed.
- On the other hand, permissible job contracting requires the
following conditions:
(1) The contractor carries on an independent business and
undertakes the contract work on his own account under his own
responsibility according to his own manner and method, free
from the control and direction of his employer or principal in all

Labor Law 1
matters connected with the performance of the work except as
to the results thereof; and
(2) The contractor has substantial capital or investment in the
form of tools, equipment, [machinery], work premises, and
other materials which are necessary in the conduct of his
business.
- The employee-employer relation existed between the
individual private respondents and STELLAR, not PAL. STELLAR
possessed these earmarks of an employer:
(1) the power of selection and engagement of employees
(2) the payment of wages
(3) the power of dismissal, and
(4) the power to control the employee's conduct
- A contract of employment existed between STELLAR and the
individual private respondents, proving that it was said
corporation which hired them. It was also STELLAR which
dismissed them, as evidenced by Complainant Parenas'
termination letter, which was signed by Carlos P. Callanga, vice
president
for
operations
and
comptroller
of
STELLAR. Likewise, they worked under STELLAR's own
supervisors, Rodel Pagsulingan, Napoleon Parungao and Renato
Topacio. STELLAR even had its own collective bargaining
agreement with its employees, including the individual private
respondents. Moreover, PAL had no power of control and
dismissal over them.
- In fact, STELLAR claims that it falls under the definition of an
independent job contractor. Thus, it alleges that it has sufficient
capital in the form of tools and equipment, like vacuum
cleaners and polishers, and substantial capitalization as proven
by its financial statements. Further, STELLAR has clients other
than petitioner, like San Miguel Corporation, Hongkong and
Shanghai Bank, Eveready, Benguet Management Corporation
and Japan Airlines.
- All these circumstances establish that STELLAR undertook said
contract on its account, under its own responsibility, according
to its own manner and method, and free from the control and
direction of the petitioner. Where the control of the principal is
limited only to the result of the work, independent job
contracting exists. The janitorial service agreement between
petitioner and STELLAR is definitely a case of permissible job
contracting.
b) What actually happened was that PAL and STELLAR impliedly
renewed, as they had previously done before, their service
agreement until PAL's janitorial requirements were bidded to
other job contractors. This explains why the individual private
respondents remained working at PAL's premises even after
December 31, 1990.
- It is evident that petitioner was engaged in permissible job
contracting and that the individual private respondents, for the
entire duration of their employ, were employees not of
petitioner but of STELLAR. In legitimate job contracting, no
employer-employee relation exists between the principal and
the job contractor's employees. The principal is responsible to
the job contractor's employees only for the proper payment of
wages. But in labor-only contracting, an employer-employee
relation is created by law between the principal and the laboronly contractor's employees, such that the former is responsible
to such employees, as if he or she had directly employed them.
Besides, the Court has already taken judicial notice of the
general practice adopted in several government and private
institutions of securing janitorial services on an independent
contractor basis.
2. NO, STELLAR is the one liable for separation pay.
Ratio Despite the protestations of STELLAR, the service
agreement was not a project because its duration was not
determined or determinable.
Reasoning
- In order to avoid liability for separation pay, STELLAR argues
that it terminated the services of the individual private
respondents for a just and valid cause: the completion of a
specific project. Thus, they are not entitled to separation pay.

A2010

- 76 -

Disini

- The Court is not convinced. The position of STELLAR that


individual private respondents were its project employees is
totally unfounded. A regular employee is distinguished from a
project employee by the fact that the latter is employed to
carry out a specific project or undertaking, the duration or
scope of which was specified at the time the employees were
engaged. A "project" has reference to a particular job or
undertaking that may or may not be within the regular or usual
business of the employer. In either case, the project must be
distinct, separate and identifiable from the main business of the
employer, and its duration must be determined or
determinable.
- While the service agreement may have had a specific term,
STELLAR disregarded it, repeatedly renewed the service
agreement, and continued hiring the individual private
respondents for thirteen consecutive years. Had STELLAR won
the bidding, the alleged "project" would have never ended. In
any event, the aforesaid stipulations in the employment
contract are not included in Articles 282 and 283 of the Labor
Code as valid causes for the dismissal of employees.
Again, we must emphasize that the main business of STELLAR is
the supply of manpower to perform janitorial services for its
clients, and the individual private respondents were janitors
engaged to perform activities that were necessary and
desirable to STELLAR's enterprise. In this case, we hold that the
individual private respondents were STELLAR's regular
employees, and there was no valid cause for their dismissal.
Disposition
petition is hereby GRANTED. The assailed
Decision and Resolution are SET ASIDE insofar as they held PAL
liable for separation pay. The July 13, 1994 Decision is however
reinstated insofar as it ORDERED STELLAR liable for such award.

MERCURY DRUG CORPORATION V LIBUNAO


434 SCRA 404
CALLEJO, SR; July 14, 2004
NATURE
Petition for review on certiorari of a CA decision which modified
an RTC decision, and the Resolution of the CA denying the
petitioners motion for reconsideration
FACTS
According to the plaintiff.
> Libunao and his friend bought some items at Mercury. He
paid for his purchase and placed his receipt in his pocket. As
they exited, they were accosted by Sido, the security guard.
Sido was armed with a service gun, and was 20 pounds heavier
than Libunao. He held Libunaos upper right arm and demanded
to see the receipt. Libunao searched but it took time because
Sido was holding his right arm. Sido then said Wala yatang
resibo yan! Libunao finally found it, and asked Sido, Satisfied
ka na? Sido reacted by lunging at him and saying Putang ina
mo! Sido was able to hit lubnao on the face, nose, chin, and
mouth. He then pointed his revolver at Libunao and said
Putang ina mo, pag hindi kayo lumabas ditto papuputukin ko
to sa iyo! Libunao eventually filed a criminal complaint against
Sido. He was traumatized by the event, he had to consult a
psychiatrist, and was found to be suffering from post-traumatic
depression syndrome.
According to the defendants
> Sido, the security guard at Mercury, noticed Libunao exiting
the store with a plastic bag, and that no receipt was stapled to
it. He asked for the receipt, but was given the plastic bag. He
found no receipt, and when Libunao finally found the receipt
and shoved it in his face, he just explained he was doing his
duty. Libunao said Baka hindi mo ako kilala, security guard ka
lang! Ano ba talaga ang problema mo? A violent argument
ensued.
- The court rendered judgment in favor of the plaintiff, that the
defendants Sido, Mercurly Drug Corporation, and Store Manager
Vilma Santos, pay the plaintiff moral and exemplary damages,

Labor Law 1
to discourage disrespect of the public by such acts as were
committed by defendants
ISSUE
WON the remedy of the petitioner is proper (that Mercury Drug
be liable for Sidos actions)
HELD
NO
Ratio The petitioner was not Sidos employer; hence, CC A
2180 should not be applied against petitioner.
Reasoning
- The respondent was burdened to prove that the petitioner was
the employer of Sido but failed to discharge this burden.
- The respondents counsel admitted Sido was not employed by
the petitioner
- Store manager Santos testified that Sido was not an employee
of the petitioner, but of BSSC, Black Shield Agency.
- The petitioner adduced in evidence its contract with the BSSC,
which contained the following provisions: 1. THE AGENCY shall
provide the CLIENT with the necessary number of armed,
uniformed and qualified security guards properly licensed by
the Chief of Philippine Constabulary; who shall provide security
services to the CLIENT at its establishment at
These security guards during the life of the Agreement shall
be assigned in accordance with arrangements to be made
between the CLIENT and the AGENCY.
...
6. The AGENCY assumes full responsibility for any claim or
cause of action which may accrue in favor of any security
guard by reason of employment with the AGENCY, it being
understood that security guards are employees of the
AGENCY and not of the CLIENT.
- Therefore, the respondent had no cause of action against the
petitioner for damages for Sidos illegal and harmful acts. The
respondent should have sued Sido and the BSSC for damages,
conformable to A2180.
- In Soliman, Jr. v. Tuazon the court held that where the security
agency recruits, hires and assigns the works of its watchmen or
security guards to a client, the employer of such guards or
watchmen is such agency, and not the client, since the latter
has no hand in selecting the security guards. Thus, the duty to
observe the diligence of a good father of a family cannot be
demanded from the said client
- The petitioner had assigned Sido to help the management
open and close the door of the drug store; inspect the bags of
customers as they enter the store; and, check the receipts
issued by the cashier to said customers for their purchases.
Such circumstances do not automatically make the security
guard the employee of the petitioner, and, as such, liable for
the guard's tortious acts. The fact that a client company may
give instructions or directions to the security guards assigned
to it, does not, by itself, render the client responsible as an
employer of the security guards concerned and liable for their
wrongful acts or omissions.
Disposition petition is hereby GRANTED. The Decision dated
June 9, 2000 and the Resolution dated August 9, 2000 of the
Court of Appeals in CA-G.R. CV No. 59754 are hereby REVERSED
and SET ASIDE. The complaint filed by the respondent against
petitioner Mercury Drug Corporation in Civil Case No. Q-9214114 is DISMISSED. The counterclaims of the latter are also
DISMISSED. No costs.

MARIVELESSHIPYARD V CA
415 SCRA573
QUISUMBING; November 11, 2003
FACTS
- In October 1993, petitioner Mariveles Shipyard Corporation
engaged the services of Longest Force Investigation and
Security Agency, Inc. to render security services at its premises.

A2010

- 77 -

Disini

Pursuant to their agreement, Longest Force deployed its


security guards, the private respondents herein, at the
petitioners shipyard in Mariveles, Bataan.
- According to petitioner, it found the services being rendered
by the assigned guards unsatisfactory and inadequate, causing
it to terminate its contract with Longest Force on April 1995.
Longest Force, in turn, terminated the employment of the
security guards it had deployed at petitioners shipyard.
- Private respondents filed a case for illegal dismissal and
underpayment of wages, among others. In turn, Longest Force
filed a cross-claim against Mariveles Shipyard, alleging that the
service fee paid by the latter to it was way below the PNPSOSIA
and PADPAO rate.
- The petitioner denied any liability on account of the alleged
illegal dismissal, stressing that no employer-employee
relationship existed between it and the security guards.
Petitioner likewise prayed that Longest Forces cross-claim be
dismissed for lack of merit. Petitioner averred that Longest
Force had benefited from the contract, it was now estopped
from questioning said agreement on the ground that it had
made a bad deal.
- The Labor Arbiter found Mariveles and Longest Force jointly
and severally liable for private respondents money claims and
attorneys fees. Longest Force was likewise ordered to reinstate
private respondents without loss of seniority rights and
privileges with full backwages. The NLRC affirmed the Labor
Arbiters decision.
- The Court of Appeals refused to give due course to Mariveles
Shipyards appeal for failure to comply with procedural
requirements.
ISSUES
1. WON the Court of Appeals dismissal of the petition was in
order despite petitioners subsequent compliance with the
procedural requirements
2. WON petitioner was denied due process of law by the NLRC
3. WON petitioner is jointly and severally liable with Longest
Force for private respondents money claims
HELD
1. NO
- The requirement in the Rules that the certification of nonforum shopping should be executed and signed by the plaintiff
or the principal means that counsel cannot sign said
certification unless clothed with special authority to do so. The
reason for this is that the plaintiff or principal knows better than
anyone else whether a petition has previously been filed
involving the same case or substantially the same issues.
Hence, a certification signed by counsel alone is defective and
constitutes a valid cause for dismissal of the petition. In the
case of the corporations, the physical act of signing may be
performed, on behalf of the corporate entity, only by specifically
authorized individuals for the simple reason that corporations,
as artificial persons, cannot personally do the task themselves.
In this case, not only was the originally appended certification
signed by counsel, but in its motion for reconsideration, still
petitioner utterly failed to show that Ms. Rosanna Ignacio, its
Personnel Manager who signed the verification and certification
of non-forum shopping attached thereto, was duly authorized
for this purpose.
2. NO
- The essence of due process is simply an opportunity to be
heard, or, as applied to administrative proceedings, an
opportunity to explain ones side or an opportunity to seek a
reconsideration of the action or ruling complained of. Not all
cases require a trial-type hearing. The requirement of due
process in labor cases before a Labor Arbiter is satisfied when
the parties are given the opportunity to submit their position
papers to which they are supposed to attach all the supporting
documents or documentary evidence that would prove their
respective claims, in the event the Labor Arbiter determines
that no formal hearing would be conducted or that such hearing

Labor Law 1
was not necessary. In any event, petitioner was given ample
opportunity to present its side in several hearings conducted
before the Labor Arbiter and in the position papers and other
supporting documents that it had submitted. Such opportunity
more than satisfies the requirement of due process in labor
cases.
3. YES
- Petitioners liability is joint and several with that of Longest
Force, pursuant to Articles 106, 107 and 109 of the Labor Code.
In this case, when petitioner contracted for security services
with Longest Force as the security agency that hired private
respondents to work as guards for the shipyard corporation,
petitioner became an indirect employer of private respondents
pursuant to Article 107. Following Article 106, when the agency
as contractor failed to pay the guards, the corporation as
principal becomes jointly and severally liable for the guards
wages.
This is mandated by the Labor Code to ensure
compliance with its provisions, including payment of statutory
minimum wage. The security agency is held liable by virtue of
its status as direct employer, while the corporation is deemed
the indirect employer of the guards for the purpose of paying
their wages in the event of failure of the agency to pay them.
This statutory scheme gives the workers the ample protection
consonant with labor and social justice provisions of the 1987
Constitution.
- Petitioner cannot evade its liability by claiming that it had
religiously paid the compensation of guards as stipulated under
the contract with the security agency. Labor standards are
enacted by the legislature to alleviate the plight of workers
whose wages barely meet the spiraling costs of their basic
needs. Labor laws are considered written in every contract.
Stipulations in violation thereof are considered null. Similarly,
legislated wage increases are deemed amendments to the
contract.
- However, we must emphasize that the joint and several
liability imposed on petitioner is without prejudice to a claim for
reimbursement by petitioner against the security agency for
such amounts as petitioner may have to pay to complainants,
the private respondents herein. The security agency may not
seek exculpation by claiming that the principals payments to it
were inadequate for the guards lawful compensation. As an
employer, the security agency is charged with knowledge of
labor laws; and the adequacy of the compensation that it
demands for contractual services is its principal concern and
not any others.
- On the issue of the propriety of the award of overtime pay
despite the alleged lack of proof thereof, suffice it to state that
such involves a determination and evaluation of facts which
cannot be done in a petition for review.
- Upon review of the award of backwages and attorneys fees,
we discovered certain errors that happened in the addition of
the amount of individual backwages that resulted in the
erroneous total amount of backwages and attorneys fees.
These errors ought to be properly rectified now. Thus, the
correct sum of individual backwages should be P126,648.40
instead of P126,684.40, while the correct sum of total
backwages awarded and attorneys fees should be
P3,926,100.40 and P392,610.04, instead of P3,927,216.40
and P392,721.64, respectively.
Disposition The Court of Appeals Resolution is AFFIRMED with
MODIFICATION.

NEW GOLDEN CITY BUILDERS V CA (GALLO ET AL)


418 SCRA 411
YNARES-SANTIAGO; December 11, 2003
FACTS
- New Golden City Builders and Development Corporation, a
corporation engaged in the construction business, entered into
a construction contract with Prince David Development
Corporation for the construction of a 17-storey office and

A2010

- 78 -

Disini

residential condominium building along Katipunan Road, Loyola


Heights, Quezon City, Metro Manila.
- Petitioner engaged the services of Nilo Layno Builders to do
the specialized concrete works, form works and steel rebar
works, for a total contract price of P5 Million. Nilo Layno
Builders hired private respondents to perform work at the
project. After the completion of the phase for which Nilo Layno
Builders was contracted sometime in 1996, private respondents
filed a complaint case against petitioner and its president,
Manuel Sy, with the Arbitration Branch of the NLRC for unfair
labor practice, non-payment of 13th month pay, non-payment
of 5 days service incentive leave, illegal dismissal and
severance pay in lieu of reinstatement.
- The Labor Arbiter found that Nilo Layno Builders was a laboronly-contractor; thus, private respondents were deemed
employees of New Golden City. Both parties appealed the
decision of the Labor Arbiter to the NLRC. Petitioner maintained
that Nilo Layno Builders was an independent contractor and
that private respondents were not its employees. On the other
hand, private respondents claimed that the Labor Arbiter erred
in finding that they were not illegally dismissed and not entitled
to recover monetary claims like premium pay for rest days,
regular holidays and special holiday. The NLRC affirmed with
modification the Labor Arbiters decision. As modified, the NLRC
held that private respondents were illegally dismissed and
ordered petitioner to reinstate them and to pay their full back
wages. CA affirmed.
ISSUES
1. WON Nilo Layno Builders was an independent contractor
and not a labor-only contractor
2. WON an employer-employee relationship existed between
petitioner and private respondents
HELD
1. YES
Ratio The test to determine the existence of independent
contractorship is whether one claiming to be an independent
contractor has contracted to do the work according to his own
methods and without being subject to the control of the
employer, except only to the results of the work.
Reasoning
- Under Section 8, Rule VIII, Book III, of the Omnibus Rules
Implementing the Labor Code, an independent contractor is one
who undertakes job contracting, i.e., a person who: (a) carries
on an independent business and undertakes the contract work
on his own account under his own responsibility according to his
own manner and method, free from the control and direction of
his employer or principal in all matters connected with the
performance of the work except as to the results thereof; and
(b) has substantial capital or investment in the form of tools,
equipments, machineries, work premises, and other materials
which are necessary in the conduct of the business.
- Nilo Layno Builders hired its own employees, the private
respondents, to do specialized work in the Prince David Project
of the petitioner. The means and methods adopted by the
private respondents were directed by Nilo Layno Builders
except that, from time to time, the engineers of the petitioner
visited the site to check whether the work was in accord with
the plans and specifications of the principal. As admitted by
Nilo G. Layno, he undertook the contract work on his own
account and responsibility, free from interference from any
other persons, except as to the results; that he was the one
paying the salaries of private respondents; and that as
employer of the private respondents, he had the power to
terminate or dismiss them for just and valid cause.
- As a licensed labor contractor, Nilo Layno Builders complied
with the conditions set forth in Section 5, Rule VII-A, Book III,
Rules to Implement the Labor Code, among others, proof of
financial capability and list of equipment, tools, machineries
and implements to be used in the business.
2. YES. [But for a limited purpose only]

Labor Law 1
Ratio In legitimate job contracting, the law creates an
employer-employee relationship for a limited purpose, i.e., to
ensure that the employees are paid their wages. The principal
employer becomes jointly and severally liable with the job
contractor only for the payment of the employees wages
whenever the contractor fails to pay the same. Other than that,
the principal employer is not responsible for any claim made by
the employees.
Reasoning
- The petitioner did not, as it could not, illegally dismissed the
private complainants. Hence, it could not be held liable for back
wages and separation pay. Nevertheless, it is jointly and
severally liable with Nilo Layno Builders for the private
complainants wages, in the same manner and extent that it is
liable to its direct employees. The pertinent provisions of the
Labor Code read:
ART. 106. Contractor or subcontractor. Whenever an
employer enters into a contract with another person for the
performance of the formers work, the employees of the
contractor and of the latters subcontractor, if any, shall be
paid in accordance with the provisions of this Code. XXX In
the event that the contractor or subcontractor fails to pay the
wages of his employees in accordance with this Code, the
employer shall be jointly and severally liable with his
contractor or subcontractor to such employees to the extent
of the work performed under the contract, in the same
manner and extent that he is liable to employees directly
employed by him.
ART. 107.
Indirect employer. The provisions of the
immediately preceding Article shall likewise apply to any
person, partnership, association or corporation which, not
being an employer, contracts with an independent contractor
for the performance of any work, task, job or project.
- This liability covers the payment of service incentive leave and
13th month pay of the private complainants during the time
they were working at petitioners Prince David Project. So long
as the work, task, job or project has been performed for
petitioners benefit or on its behalf, the liability accrues for such
period even if, later on, the employees are eventually
transferred or reassigned elsewhere.
Disposition Petition PARTLY GRANTED. Decision of the CA
MODIFIED. Petitioner ABSOLVED from liability for back wages.
However, he is ORDERED to pay, jointly and severally with Nilo
Layno Builders, private complainants Service Incentive Leave
Pay and 13th Month Pay.

VINOYA V NLRC (REGENT FOOD CORP)


324 SCRA 469
KAPUNAN; February 2, 2000
NATURE
Petition for certiorari under Rule 65 seeking to annul NLRC
decision
FACTS
- Petition seeks to annul and set aside the decision of the NLRC
which reversed the decision of the Labor Arbiter, ordering RFC
to reinstate Alexander Vinoya to his former position and pay
him backwages.
Petitioners Claim
> Vinoya applied and was accepted by RFC as sales
representative. RFC issued him an identification card.
- He reported daily to the RFC office to take the van for the
delivery of products. He was assigned to various supermarkets
and grocery stores where he booked sales orders and collected
payments for RFC. He was required by RFC to put up a monthly
bond of P200 as security deposit to guarantee his work
performance.
- After more than a year, he was transferred by RFC to
Peninsula Manpower Company, Inc., an agency which provides
RFC with additional contractual workers pursuant to a contract

A2010

- 79 -

Disini

for the supply of manpower services. After this, petitioner was


reassigned to RFC as sales rep.
- 5 months later, he was informed by the personnel manager of
RFC that his services were terminated and he was asked to
surrender his ID card. Petitioner was told that his dismissal was
due to the expiration of the Contract of Service between RFC
and PMCI. Petitioner claims that he was dismissed from
employment despite the absence of any notice or investigation.
- He filed a case against RFC before the Labor Arbiter for illegal
dismissal and non-payment of 13th month pay.
Respondents Comments
> RFC maintains that there is no employer-employee
relationship. Petitioner is actually an employee of PMCI, an
independent contractor, which had a Contract of Service with
RFC. RFC presented an Employment Contract signed by
petitioner on 1 July 1991, wherein PMCI appears as his
employer. RFC denies that petitioner was ever employed by it
prior to 1 July 1991. Petitioner was issued an ID card so that its
clients and customers would recognize him as a duly authorized
representative of RFC. With regard to the P200 monthly bond,
RFC asserts that it was required in order to guarantee the
turnover of his collection since he handled funds of RFC. While
RFC admits that it had control and supervision over petitioner, it
argues that such was exercised in coordination with PMCI.
Finally, RFC contends that the termination of its relationship
with petitioner was brought about by the expiration of the
Contract of Service between itself and PMCI.
ISSUES
1. WON PMCI is a labor-only contractor or an independent
contractor
2. WON petitioner was an employee of RFC or PMCI
3. WON petitioner was lawfully dismissed
HELD
1. PMCI is a labor-only contractor.
Ratio In determining the existence of an independent
contractor relationship, several factors might be considered
such as, but not necessarily confined to, whether the contractor
is carrying on an independent business; the nature and extent
of the work; the skill required; the term and duration of the
relationship; the right to assign the performance of specified
pieces of work; the control and supervision of the workers; the
power of the employer with respect to the hiring, firing and
payment of the workers of the contractor; the control of the
premises; the duty to supply premises, tools, appliances,
materials and labor; and the mode, manner and terms of
payment.
Reasoning
- Labor-only contracting, a prohibited act, is an arrangement
where the contractor or subcontractor merely recruits, supplies
or places workers to perform a job, work or service for a
principal. The following elements are present: (a) The contractor
or subcontractor does not have substantial capital or
investment to actually perform the job, work or service under
its own account and responsibility; (b) The employees recruited,
supplied or placed by such contractor or subcontractor are
performing activities which are directly related to the main
business of the principal.
- Permissible job contracting or subcontracting refers to an
arrangement whereby a principal agrees to put out or farm out
with a contractor or subcontractor the performance or
completion of a specific job, work or service within a definite or
predetermined period, regardless of whether such job, work or
service is to be performed or completed within or outside the
premises of the principal. A person is considered engaged in
legitimate job contracting or subcontracting if the following
conditions concur: (a) The contractor or subcontractor carries
on a distinct and independent business and undertakes to
perform the job, work or service on its own account and under
its own responsibility according to its own manner and method,
and free from the control and direction of the principal in all

Labor Law 1
matters connected with the performance of the work except as
to the results thereof; (b) The contractor or subcontractor has
substantial capital or investment; and (c) The agreement
between the principal and contractor or subcontractor assures
the contractual employees entitlement to all labor and
occupational safety and health standards, free exercise of the
right to self-organization, security of tenure, and social and
welfare benefits.
- First of all, PMCI does not have substantial capitalization or
investment in the form of tools, equipment, machineries, work
premises, among others, to qualify as an independent
contractor.
-Second, PMCI did not carry on an independent business nor did
it undertake the performance of its contract according to its
own manner and method, free from the control and supervision
of its principal, RFC. The evidence at hand shows that the
workers assigned by PMCI to RFC were under the control and
supervision of the latter.
-Third, PMCI was not engaged to perform a specific and special
job or service.. As stated in the Contract of Service, the sole
undertaking of PMCI was to provide RFC with a temporary
workforce able to carry out whatever service may be required
by it. Apart from that, no other particular job, work or service
was required from PMCI. Obviously, with such an arrangement,
PMCI merely acted as a recruitment agency for RFC.
- Lastly, in labor-only contracting, the employees recruited,
supplied or placed by the contractor perform activities which
are directly related to the main business of its principal. In this
case, the work of petitioner as sales representative is directly
related to the business of RFC. Being in the business of food
manufacturing and sales, it is necessary for RFC to hire a sales
representative like petitioner to take charge of booking its sales
orders and collecting payments for such. Thus, the work of
petitioner as sales representative in RFC can only be
categorized as clearly related to, and in the pursuit of the
latters business. Logically, when petitioner was assigned by
PMCI to RFC, PMCI acted merely as a labor-only contractor.
2. Petitioner was an employee of RFC
Ratio In determining the existence of employer-employee
relationship the following elements of the "four-fold test" are
generally considered, namely: (1) the selection and
engagement of the employee or the power to hire; (2) the
payment of wages; (3) the power to dismiss; and (4) the power
to control the employee. Of these four, the "control test" is the
most important.
- No particular form of proof is required to prove the existence
of an employer-employee relationship. Any competent and
relevant evidence may show the relationship.
Reasoning
- PMC I as a labor-only contractor, cannot be considered as the
employer of petitioner
- Even granting that PMCI is an independent contractor, still, a
finding of the same will not save the day for RFC. A perusal of
the Contract of Service entered into between RFC and PMCI
reveals that petitioner is actually not included in the
enumeration of the workers to be assigned to RFC. This only
shows that petitioner was never intended to be a part of those
to be contracted out.
-With regard to the first element, ID card is enough proof that
petitioner was previously hired by RFC prior to his transfer as
agency worker to PMCI. ID card issued by RFC to petitioner was
dated more than one year before the Employment Contract was
signed by petitioner in favor of PMCI. While the Employment
Contract indicates the word "renewal," presumably an attempt
to show that petitioner had previously signed a similar contract
with PMCI, no evidence of a prior contract entered into between
petitioner and PMCI was ever presented by RFC. It follows that it
was RFC who actually hired and engaged petitioner to be its
employee
- With respect to the payment of wages, the Court takes judicial
notice of the practice of employers who, in order to evade the
liabilities under the Labor Code, do not issue payslips directly to

A2010

- 80 -

Disini

their employees. Even though the wages were coursed through


PMCI, we note that the funds actually came from the pockets of
RFC. Thus, in the end, RFC is still the one who paid the wages of
petitioner albeit indirectly.
- As to the third element, the power to dismiss, the Contract of
Service gave RFC the right to terminate the workers assigned to
it by PMCI without the latters approval. The dismissal of
petitioner was indeed made under the instruction of RFC to
PMCI.
-The power of control refers to the authority of the employer to
control the employee not only with regard to the result of work
to be done but also to the means and methods by which the
work is to be accomplished. The "control test" calls merely for
the existence of the right to control the manner of doing the
work, and not necessarily to the actual exercise of the right.
The Labor Arbiter found that petitioner was under the direct
control and supervision of the personnel of RFC and not PMCI.
3. YES
Ratio
The requirements for the lawful dismissal of an
employee are two-fold, the substantive and the procedural
aspects. Not only must the dismissal be for a valid or authorized
cause, the rudimentary requirements of due process - notice
and hearing must, likewise, be observed before an employee
may be dismissed.
Reasoning
- Since petitioner, due to his length of service, already attained
the status of a regular employee, he is entitled to the security
of tenure provided under the labor laws. Hence, he may only be
validly terminated from service upon compliance with the legal
requisites for dismissal.
- RFC never pointed to any valid or authorized cause under the
Labor Code which allowed it to terminate the services of
petitioner. Its lone allegation that the dismissal was due to the
expiration or completion of contract is not even one of the
grounds for termination allowed by law.
- Neither did RFC show that petitioner was given ample
opportunity to contest the legality of his dismissal. In fact, no
notice of such impending termination was ever given him.
- An employee who has been illegally dismissed is entitled to
reinstatement to his former position without loss of seniority
rights and to payment of full backwages corresponding to the
period from his illegal dismissal up to actual reinstatement.
Disposition Petition granted.

MANILA WATER COMPANY V PENA


434 SCRA 52
YNARES-SANTIAGO; July 8, 2004
NATURE
Petition for review on certiorari
FACTS
Manila Water is one of the 2 concessionaires contracted by
MWSS to manage water distribution. Pursuant to RA No. 8041,
petitioner undertook to absorb ex-employees of MWSS whose
names were in the list, and those not in the list were
terminated.
Respondents are contractual collectors, not listed, but petitioner
still engaged their services. They signed a 3 month contract.
121 collectors incorporated the Association Collectors Group
Inc. or ACGI. Petitioner continued to transact with ACGI, but
eventually terminated its contract. Respondents filed complaint
for illegal dismissal, saying they were petitioners employees.
Petitioner asserts respondents were employees of ACGI, an
independent contractor.
Arbiter found dismissal illegal. NLRC reversed. Respondents
filed certiorari petition with CA, which reversed NLRC decision
and reinstated with modification the Arbiter decision. Hence
this petition.
ISSUES

Labor Law 1
1. WON theres employer-employee relationship
2. WON respondents were illegally dismissed
HELD
1. YES
- We must resolve WON ACGI is an independent contractor or a
labor-only contractor.
- Labor-only contracting refers to arrangement where contractor
merely recruits and places workers for a principal. Elements
- contractor doesnt have substantial capital
- contractor doesnt control performance of contractual
employee
- Arbiter correctly ruled that ACGI was not an independent
contractor. ACGI doesnt have substantial capital. It has no
office. The work of the respondents was directly related to
business of petitioner.
And ACGI did not carry on an
independent business according to its own manner.
- ACGI was a labor-only contractor, an agent of the petitioner.
- Then the workers are employees of the petitioner.
- Even the four-fold test (selection, payment of wages, dismissal
power, control of conduct) indicate the relationship.
2. YES
- The term fixed in the subsequent contract was used to defeat
the tenurial security.
- Dismissal was illegal.

GRANDSPAN DEVELOPMENT CORP V


BERNARDO
470 SCRA 461
SANDOVAL-GUTIERREZ; September 21, 2005
NATURE
Certiorari under Rule 45
FACTS
- The instant controversy stemmed from a complaint for illegal
dismissal and non-payment of benefits filed with the LA by
Ricardo Bernardo, Antonino Ceidoza and Edgar Del Prado,
against Grandspan and/or its warehouse manager, Manuel G.
Lee
- In their complaint, respondents alleged that sometime in
1990, they were employed as truck scale monitors by
Grandspan with a daily salary of P104.00 each. Eventually,
they were assigned at its Truck Scale Section of the
Warehouse/Materials
Department.
They
were
issued
identification cards signed.
- Oct 28, 1992-Grandspan sent them a notice terminating their
services effective October 29, 1992 for using profane or
offensive language, in violation of Article VI (2) (a) of the
companys Rules and Regulations.
- Grandspan denied the allegations and claimed that
respondents are employees of J. Narag Construction.
- Sometime in the 3rd quarter of 1992, Canad Japan Co., Ltd.
engaged Grandspans services for fabrication works of several
round and rectangular steel tanks needed for the HCMG or Sogo
project due for completion in September, 1992.
As a
consequence, Grandspan subcontracted the services of J. Narag
Construction which, in turn, assigned its 3 helpers (herein
respondents) to work for its project.
- Sometime in October, 1992, Manuel G. Lee, manager of
Grandspans Warehouse Department received a report from
supervisor Robert Ong that respondents vandalized the
companys log book and chairs.
- This prompted petitioner to send J. Narag Construction a
memorandum terminating the services of respondents for
violation of the companys Rules and Regulations.
- June 30, 1994- LA dismissed respondents complaint;
concluded that respondents were validly dismissed from
employment; held too that respondent were project employees
whose services were terminated upon completion of the project
for which they were hired.

A2010

- 81 -

Disini

- NLRC- remanded the case to the LA for appropriate


proceedings to determine whether there is an employeremployee relationship between the parties.
- Both parties filed MRs but were denied by the NLRC in
separate Resolutions
- Meantime, Del Prado died and was substituted by his surviving
parent, Edgardo Del Prado.
- Sept 17, 1999- CA set aside the NLRCs Resolutions and
ordered Grandspan (1) to reinstate respondents Bernardo and
Ceidoza to their former positions and pay, jointly and severally
with J. Narag Construction, their backwages and other benefits,
and (2) to pay respondent Del Prado his separation pay.
- CA found that respondents are employees of petitioner; that
they were non-project workers; and that they were denied
due process, thus:
They(Respondents Bernardo et al) worked in Grandspans
premises using the materials, supplies and equipment of
Grandspan. They were under the supervision of Grandspan
as to the manner and results of their work, and performed
services directly connected to the usual business of
respondent Grandspan for the fabrication of heavy structural
components
- Oct 8, 1999- Grandspan filed MR. Respondents also filed a
motion for reconsideration and/or clarification praying that the
Appellate Courts Decision be modified by awarding respondent
Del Prado his backwages.
- Jan 6, 2000- CA denied Grandspans MR but order Grandspan
and J. Narag Construction to pay respondent Del Prado his
separation pay and backwages.
ISSUE
WON the CA erred in holding that respondents are employees of
Grandspan
HELD
YES
Grandspans Argument:
> it has no employer-employee relationship with respondents
since they are employees of J. Narag Construction, an
independent contractor.
- Miguel vs. JCT Group - The test for determining an employeremployee relationship hinges on resolving who has the power to
select employees, who pays for their wages, who has the power
to dismiss them, and who exercises control in the methods and
the results by which the work is accomplished.
- SC agrees with CA when it found that J. Narag Construction
assigned respondents to perform activities directly related to
the main business of petitioner. . These circumstances
confirm the existence of an employer-employee relationship
between petitioner and respondents.
> They worked in petitioners premises, using its equipment,
materials and supplies. J. Narag Constructions payroll
worksheets covering the period from December 21, 1990 to
July 31, 1991 show that the payment of their salaries was
approved by petitioner.
> The manager and supervisor of petitioners Warehouse
Department supervised the manner and results of their
work.
> It was petitioner who terminated their services after finding
them guilty of using profane or offensive language in violation
of Article VI (2) (a) of the companys Rules and Regulations
- SC also agrees with the CA that J. Narag Construction is a
labor-only contractor.
> A106 LC as amended, provides that there is labor-only
contracting where the person supplying workers to an
employer does not have substantial capital or investment in
the form of tools, equipment, machineries, work premises,
among others, and the workers recruited and placed by such
person are performing activities which are directly related to
the principal business of such employer. x x x.
- J. Narag Construction is indeed a labor-only contractor. These
are the reasons:

Labor Law 1
(1) it is not registered as a building contractor with the SEC;
(2) it has no contract with petitioner; and
(3) there is no proof of its financial capability and has no list
of equipment, tools, machineries and implements used in the
business.
- Kiamco vs. NLRC: The principal test for determining whether
particular employees are properly characterized as project
employees, as distinguished from regular employees, is
whether or not the project employees were assigned to carry
out a specific project or undertaking, the duration and scope of
which were specified at the time the employees were engaged
for that project. As defined, project employees are those
workers hired (1) for a specific project or undertaking, and (2)
the completion or termination of such project or undertaking
has been determined at the time of engagement of the
employee.
- Grandspan could not present employment contracts signed by
respondents showing that their employment was for the
duration of the HCMG or Sogo project and failed to present any
report terminating the services of respondents when its projects
were actually finished pursuant to Sec2.2 (e) of the Labor
Department Order No. 19
SC: The failure of the employer to file termination reports after
every project completion with the nearest public employment
office is an indication that respondents were not project
employees.
TF: respondents are Grandspans regular
employees. As such, they are entitled to security of tenure and
can only be dismissed for a just or authorized cause, as
provided by Article 279 of the Labor Code.
- Bolinao Security and Investigation Service, Inc. vs. Toston- it
is incumbent upon the employer to prove by the quantum of
evidence required by law that the dismissal of an employee is
not illegal, otherwise, the dismissal would be unjustified.
- SC: Termination is ILLEGAL. Grandspan violated respondents
right (both substantive and procedural) to due process as
records show that respondents were not served by notices of
any kind nor were asked to explain the misconduct imputed to
them.
> Loadstar Shipping Co., Inc. vs. Mesano: The law requires
that an employee sought to be dismissed must be served two
written notices before termination of his employment. The
first notice is to apprise the employee of the particular acts or
omissions by reason of which his dismissal has been decided
upon; and the second notice is to inform the employee of the
employers decision to dismiss him. Failure to comply with
the requirement of two notices makes the dismissal illegal.
The procedure is mandatory. Non-observance thereof
renders the dismissal of an employee illegal and void.
- SC: they are entitled to reinstatement without loss of seniority
rights, full backwages, inclusive of allowances, and other
benefits or their monetary equivalent computed from the
time their compensation was withheld from them up to
the time of their actual reinstatement.
Disposition CAs decision AFFIRMED with modification.
Reinstatement in this case is N/A because of antagonism.
Respondents are entitled
to
a
separation
pay
of
P4,992.00 plus their respective full backwages, and other
privileges and benefits, or their monetary equivalent, during the
period of their dismissal up to their supposed actual
reinstatement.

PHILIPPINE AIRLINES INC V NLRC (VILLENA,


NATIONAL ORGANIZATION OF WORKING MEN,
VILLACRUZ)
296 SCRA 214
QUISUMBING; September 25, 1998
NATURE
Special Civil Action in the Supreme Court. Certiorari.
FACTS

A2010

- 82 -

Disini

- Petitioner Philippine Airlines Inc. (PAL) is a domestic


corporation principally engaged in the air transportation
industry for both domestic and foreign markets.
Private
respondent National Organization of the Workingmen (NOWM)
is a labor union, while the other private respondents are
members of respondent union and complainants in
aforementioned cases.
- PAL contracted the services G. C. Services Enterprises, to
undertake specific projects.
Accordingly, G. C. Services
recruited and hired carpenters, painters, and electricians and
assigned them to different PAL shops, namely: Carpentry Shop,
Electrical Shop, Technical Center Shop and Inflight Center Shop,
all under PALs Construction and Corporate Services
Department.
- PAL terminated its contract with G. C. Services. As a result, all
G.C. employees assigned as PAL project workers were notified
by G.C. Services not to report anymore to PAL. Later, PAL
decided to give G.C. Services employees the opportunity to
apply as regular employees, in accordance with its practice of
giving employment priority to qualified persons who had been
connected with PAL. Due to lack of vacant positions and also
due to alleged unsatisfactory work performance records of
some, not all G.C. Services employees were hired. Those who
were not hired instituted the instant complaint for illegal
dismissal. The complainants were represented in their case by
the NOWM.
- Initially, there were 36 complainants in these three
consolidated cases. In the course of the proceedings, PAL
agreed to employ 23 qualified complainants.
Only 12
complainants were left.
- The rest of the complainants alleged that they applied for
employment with G.C. Services; that after they were accepted
they were made to work at PAL Maintenance Department where
each of them worked as carpenters, welders, or electricians;
that they were not considered employees of PAL but that of G.C.
Services; that their work are necessary and directly related to
PALs principal business. In pointing at PAL as their real
employer, they averred that G.C. Services is only an agent of
PAL because it does not have substantial capital in the form of
cash investments, tools, equipment or work premises; that it
merely supplied workers to PAL and these workers were
supervised, directed and controlled by PAL regular employees;
that PAL actually decided when, where and what to work; that
PAL decided how many of them were to be taken in, when they
would start, and when they would not.
Complainants, thus,
argued that G.C. Services being a mere agent, the real
employer was PAL pursuant to Art.106 of the Labor Code which
prohibits the employment of persons through labor only
contracting agencies, like the G.C. Services Enterprise.
- In claiming that they were illegally dismissed, complainants
alleged that they were dismissed from employment without
just cause and without due process and without any prior
notice. They thus prayed for reinstatement with full backwages
from the date of their dismissal on March 31, 1990 up to the
date of their actual reinstatement.
- Respondent PAL denied the existence of employer-employee
relationship between it and the complainants. It averred that
G.C. Services Enterprises, as a duly licensed independent
contractor, contracted on its own account under its own
responsibility; that the contractor has substantial capital or
investment in the form of tools, equipment and other materials
necessary in the conduct of its business; that complainants
were being paid their wages by G.C. Services and not PAL; and
that they were terminated by G.C. Services. PAL further argued
that even granting arguendo that complainants are entitled to
be regularized, it is not obliged to employ all the complainants;
and that there are no more positions or substantially equivalent
positions within its organization for which they maybe (sic)
qualified.
- The Labor Arbiter ruled that G.C. Services Enterprises is a
labor-only contractor and mere agent of PAL (petitioner herein),
thus, the private respondents are deemed employees of

Labor Law 1
petitioner. The Labor Arbiter then declared the termination of
private respondents services illegal, and held petitioner and
G.C. Services Enterprises jointly and severally liable to pay
private respondents their separation pay, backwages as well as
attorneys fees.
- Both parties appealed to the NLRC, which, affirmed the Labor
Arbiters decision with modification as to the computation of the
monetary award.
- Its motion for reconsideration having been denied, petitioner
filed the instant petition.
ISSUES
1. WON the public respondents committed grave abuse of
discretion in declaring the dismissal of private respondents
illegal despite the finding of redundancy
2. WON private respondents are entitled to separation pay as
well as backwages
3. WON petitioner should be held jointly and severally liable
HELD
1. YES
- The petitioner regularized and/or re-employed 23 original
complainants as there were vacant positions to which they
could qualify. However, the remaining 12 complainants (private
respondents herein) could no longer be absorbed into
petitioners regular workforce as there were no longer vacant
positions as evidenced by the Table of Organization of PAL
Construction and Corporate Services Department. Simply put,
the services of private respondents were already in excess of
what is reasonably demanded by the actual manpower
requirement of petitioner. It is settled that where there is need
for reduction of workforce, management has the right to choose
whom to layoff, depending on the work still required to be done
and the qualities of the workers to be retained.
- Under Article 203 (must be 283) of the Labor Code, the
employer may terminate an employee due to redundancy or
retrenchment.
- In Wilshire (sic) File Co., Inc. v. NLRC, 193 SCRA 672 the
Supreme Court aptly ruled:
Redundancy, for purposes of our Labor code, exists where
the services of an employee are in excess of what is
reasonably demanded by the actual requirements of the
enterprise. Succinctly put, a position is redundant where it is
superfluous and the superfluity of a position or positions may
be the outcome of a number of factors, such as over-hiring of
workers xxx. The employer has no legal obligation to keep in
its payroll more employees than are necessary for the
operation of its business. (underscoring supplied)
- Clearly, the Labor Arbiter recognized the existence of
redundancy. Despite said findings the Labor Arbiter ruled as
follows:
xxx In consonance therefore under Art. 280 of the Labor
Code of the Philippines, herein complainants are regular
employees. For being so, they are protected by the Security
of Tenure provision of law (Art. 279, Labor Code) the
complainant dismissal being not in contemplation with Art.
282 of the Labor Code it is therefore illegal. xxx
- The reference to Article 282 is misplaced.
Article 282
enumerates the causes for termination by reason of some
blameworthy act or omission on the part of the employee.
- In the instant case, the cause of termination is redundancy
which is an authorized cause for termination under Article 283.
In any event, it is absurd for the Labor Arbiter to declare a
finding of redundancy, on one hand, and to conclude, on the
other, that the termination of private respondents services is
illegal. There being redundancy, the dismissal of private
respondents is valid
2. NO
- Since private respondents were validly dismissed under Art.
283, they are not entitled to backwages. Apparently, public
respondents awarded backwages to private respondents to
penalize PAL for engaging in a labor-only scheme. However,

A2010

- 83 -

Disini

the law does not give public respondents such authority. The
only effect of labor-only contracting is that the person or
intermediary shall be considered merely as an agent of the
employer who shall be responsible to the workers in the same
manner and extent as if the latter were directly employed by
him (Art. 106, Labor Code).
- Thus, private respondents are entitled to separation pay only.
The award of backwages to them has no basis in law.
3. YES
- Petitioner and G.C. Services Enterprises are jointly and
severally liable to the private respondents for the latters
monetary claims. The reason is that G.C. Services Enterprises,
being a labor only contractor, is merely an agent of the
petitioner (the employer); the resultant liability must be
shouldered by either one or shared by both. Hence, petitioner
cannot avoid liability by invoking its Service Agreement with
G.C. Services Enterprises considering that here the liability is
imposed by law.

SAN MIGUEL V MAERC INTEGRATED SERVICES


405 SCRA 579
BELLOSILLO; July 10, 2003
FACTS
- TWO HUNDRED NINETY-ONE (291) workers filed their
complaints (nine [9] complaints in all) against San Miguel
Corporation (petitioner herein) and Maerc Integrated Services,
Inc. (respondent herein), for illegal dismissal, underpayment of
wages, non-payment of service incentive leave pays and other
labor standards benefits, and for separation pays from 25 June
to 24 October 1991. The complainants alleged that they were
hired by San Miguel Corporation (SMC) through its agent or
intermediary Maerc Integrated Services, Inc. (MAERC) to work in
two (2) designated workplaces in Mandaue City: one, inside the
SMC premises at the Mandaue Container Services, and another,
in the Philphos Warehouse owned by MAERC. They washed and
segregated various kinds of empty bottles used by SMC to sell
and distribute its beer beverages to the consuming public. They
were paid on a per piece or pakiao basis except for a few who
worked as checkers and were paid on daily wage basis.
Complainants alleged that long before SMC contracted the
services of MAERC a majority of them had already been working
for SMC under the guise of being employees of another
contractor, Jopard Services, until the services of the latter were
terminated on 31 January 1988. SMC informed MAERC of the
termination of their service contract by the end of June 1991.
SMC cited its plans to phase out its segregation activities
starting 1 June 1991 due to the installation of labor and costsaving devices. When the service contract was terminated,
complainants claimed that SMC stopped them from performing
their jobs; that this was tantamount to their being illegally
dismissed by SMC who was their real employer as their
activities were directly related, necessary and desirable to the
main business of SMC; and, that MAERC was merely made a
tool or a shield by SMC to avoid its liability under the Labor
Code
- MAERC for its part admitted that it recruited the complainants
and placed them in the bottle segregation project of SMC but
maintained that it was only conveniently used by SMC as an
intermediary in operating the project or work directly related to
the primary business concern of the latter with the end in view
of avoiding its obligations and responsibilities towards the
complaining workers.
-The Labor Arbiter rendered a decision holding that MAERC was
an independent contractor. He dismissed the complaints for
illegal dismissal but ordered MAERC to pay complainants'
separation benefits in the total amount of P2,334,150.00.
MAERC and SMC were also ordered to jointly and severally pay
complainants their wage differentials in the amount of
P845,117.00 and to pay attorney's fees in the amount of
P317,926.70.

Labor Law 1
- The National Labor Relations Commission (NLRC) ruled that
MAERC was a labor-only contractor and that complainants were
employees of SMC. The NLRC also held that whether MAERC
was a job contractor or a labor-only contractor, SMC was still
solidarily liable with MAERC for the latter's unpaid obligations,
citing Art. 109 4 of the Labor Code. Thus, the NLRC modified the
judgment of the Labor Arbiter and held SMC jointly and
severally liable with MAERC for complainants' separation
benefits. In addition, both respondents were ordered to pay
jointly and severally an indemnity fee of P2,000.00 to each
complainant.
- SMC filed petition for certiorari
ISSUE
WON the complainants are employees of petitioner SMC or of
respondent MAERC
HELD
Employees of SMC
- In ascertaining an employer-employee relationship, the
following factors are considered: (a) the selection and
engagement of employee; (b) the payment of wages; (c) the
power of dismissal; and, (d) the power to control an employee's
conduct, the last being the most important. Application of the
aforesaid criteria clearly indicates an employer-employee
relationship between petitioner and the complainants.
- Evidence discloses that petitioner played a large and
indispensable part in the hiring of MAERC's workers. It also
appears that majority of the complainants had already been
working for SMC long before the signing of the service contract
between SMC and MAERC.
- The incorporators of MAERC admitted having supplied and
recruited workers for SMC even before MAERC was created. The
NLRC also found that when MAERC was organized into a
corporation, the complainants who were then already working
for SMC were made to go through the motion of applying for
work with Ms. Olga Ouano, President and General Manager of
MAERC, upon the instruction of SMC through its supervisors to
make it appear that complainants were hired by MAERC. This
was testified to by two (2) of the workers who were segregator
and forklift operator assigned to the Beer Marketing Division at
the SMC compound and who had been working with SMC under
a purported contractor Jopard Services since March 1979 and
March 1981, respectively. Both witnesses also testified that
together with other complainants they continued working for
SMC without break from Jopard Services to MAERC.
- As for the payment of workers' wages, it is conceded that
MAERC was paid in lump sum but records suggest that the
remuneration was not computed merely according to the result
or the volume of work performed. The memoranda of the labor
rates bearing the signature of a Vice-President and General
Manager for the Vismin Beer Operations as well as a director of
SMC appended to the contract of service reveal that SMC
assumed the responsibility of paying for the mandated
overtime, holiday and rest day pays of the MAERC workers. SMC
also paid the employer's share of the SSS and Medicare
contributions, the 13th month pay, incentive leave pay and
maternity benefits. In the lump sum received, MAERC earned a
marginal amount representing the contractor's share. These
lend credence to the complaining workers' assertion that while
MAERC paid the wages of the complainants, it merely acted as
an agent of SMC.
- Petitioner insists that the most significant determinant of an
employer-employee relationship, i.e., the right to control, is
absent. The contract of services between MAERC and SMC
provided that MAERC was an independent contractor and that
the workers hired by it "shall not, in any manner and under any
circumstances, be considered employees of the Company, and
that the Company has no control or supervision whatsoever
over the conduct of the Contractor or any of its workers in
respect to how they accomplish their work or perform the
Contractor's obligations under the Contract."

A2010

- 84 -

Disini

- In deciding the question of control, the language of the


contract is not determinative of the parties' relationship; rather,
it is the totality of the facts and surrounding circumstances of
each case.
- Despite SMCs disclaimer, there are indicia that it actively
supervised the complainants. SMC maintained a constant
presence in the workplace through its own checkers. Its
asseveration that the checkers were there only to check the
end result was belied by the testimony of Carlito R. Singson,
head of the Mandaue Container Service of SMC, that the
checkers were also tasked to report on the identity of the
workers whose performance or quality of work was not
according to the rules and standards set by SMC. According to
Singson, "it (was) necessary to identify the names of those
concerned so that the management [referring to MAERC] could
call the attention to make these people improve the quality of
work."
- Other instances attesting to SMC's supervision of the workers
are found in the minutes of the meeting held by the SMC
officers on 5 December 1988. Among those matters discussed
were the calling of SMC contractors to have workers assigned to
segregation to undergo and pass eye examination to be done
by SMC EENT company doctor and a review of
compensation/incentive system for segregators to improve the
segregation activities.
- But the most telling evidence is a letter by Mr. Antonio Ouano,
Vice-President of MAERC dated 27 May 1991 addressed to
Francisco Eizmendi, SMC President and Chief Executive Officer,
asking the latter to reconsider the phasing out of SMC's
segregation activities in Mandaue City. The letter was not
denied but in fact used by SMC to advance its own arguments.
Briefly, the letter exposed the actual state of affairs under
which MAERC was formed and engaged to handle the
segregation project of SMC. It provided an account of how in
1987 Eizmendi approached the would-be incorporators of
MAERC and offered them the business of servicing the SMC
bottle-washing and segregation department in order to avert an
impending labor strike. After initial reservations, MAERC
incorporators accepted the offer and before long trial
segregation was conducted by SMC at the PHILPHOS
warehouse.
- In legitimate job contracting, the law creates an employeremployee relationship for a limited purpose, i.e., to ensure that
the employees are paid their wages. The principal employer
becomes jointly and severally liable with the job contractor only
for the payment of the employees' wages whenever the
contractor fails to pay the same. Other than that, the principal
employer is not responsible for any claim made by the
employees.
- On the other hand, in labor-only contracting, the statute
creates
an
employer-employee
relationship
for
a
comprehensive purpose: to prevent a circumvention of labor
laws. The contractor is considered merely an agent of the
principal employer and the latter is responsible to the
employees of the labor-only contractor as if such employees
had been directly employed by the principal employer. The
principal employer therefore becomes solidarily liable
with the labor-only contractor for all the rightful claims
of the employees.
- This distinction between job contractor and labor-only
contractor, however, will not discharge SMC from paying the
separation benefits of the workers, inasmuch as MAERC was
shown to be a labor-only contractor; in which case, petitioner's
liability is that of a direct employer and thus solidarily liable
with MAERC.
- SMC also failed to comply with the requirement of written
notice to both the employees concerned and the Department of
Labor and Employment (DOLE) which must be given at least
one (1) month before the intended date of retrenchment. The
fines imposed for violations of the notice requirement have
varied. The measure of this award depends on the facts of each
case and the gravity of the omission committed by the

Labor Law 1
employer. For its failure, petitioner was justly ordered to
indemnify each displaced worker P2,000.00.
Disposition Petition is DENIED.

PHIL. FEDERATION OF CREDIT COOPERATIVES INC


(PFCCI) V NLRC (ABRIL)
300 SCRA 72
ROMERO; December 11, 1998.
FACTS
- Victoria Abril was employed by PFCCI in different capacities
from 1982 to 1988, when she went on leave until she gave
birth. When she went back in 1989, after 8 months, another
employee had been permanently appointed to her former
position of office secretary. She accepted a position of Regional
Field Officer. The contract reads:
"That the employer hires the employee on contractual basis
to the position of Regional Field Officer of Region 4 under
PFCCI/WOCCU/Aid Project No. 8175 and to do the function as
stipulated in the job description assigned to him (her): on
probationary status effective February 17/90 for a period not
to exceed six (6) months from said effectivity, subject to
renewal of this contract should the employee's performance
be satisfactory."
- Said period having elapsed, respondent was allowed to work
until PFCCI presented to her another employment contract for a
period of one year commencing on January 2, 1991 until
December 31, 1991, after which period, her employment was
terminated.
- LA dismissed her complaint for illegal dismissal against PFCCI.
- NLRC set aside LAs decision and ordered her reinstated to her
last position held (RFO) or to an equivalent position, with full
backwages from Jan 1, 1992 until she is reinstated.
ISSUE
WON Abril was a regular employee and thus illegally dismissed
HELD
YES
- It is an elementary rule in the law on labor relations that a
probationary employee who is engaged to work beyond the
probationary period of six months, as provided under Art. 281 of
the Labor Code, as amended, or for any length of time set forth
by the employer, shall be considered a regular employee.
- Article 281 of the Labor Code, as amended, allows the
employer to secure the services of an employee on a
probationary basis which allows him to terminate the latter for
just cause or upon failure to qualify in accordance with
reasonable standards set forth by the employer at the time of
his engagement. A probationary employee is one who is on trial
by an employer during which the employer determines whether
or not he is qualified for permanent employment. A
probationary employment is made to afford the employer an
opportunity to observe the fitness of a probationer while at
work, and to ascertain whether he will become a proper and
efficient employee. Probationary employees, notwithstanding
their limited tenure, are also entitled to security of tenure. Thus,
except for just cause as provided by law, or under the
employment contract, a probationary employee cannot be
terminated.
- PFCCI refutes the findings of the NLRC arguing that, after
respondent had allegedly abandoned her secretarial position for
eight (8) months, she applied for the position of Regional Field
Officer for Region IV, which appointment, as petitioner would
aptly put it, "had been fixed for a specific project or undertaking
the completion or termination of which had been determined at
the time of the engagement of said private respondent and
therefore considered as a casual or contractual employment
under Article 280 of the Labor Code."
- Abril cannot be classified as casual or contractual. (This is why
the Court went into a discussion of the kinds of employment
recognized in this jurisdiction)

A2010

- 85 -

Disini

"Art. 280. Regular and casual employment. The provisions


of written agreement to the contrary notwithstanding and
regardless of the oral agreement of the parties, an
employment shall be deemed to be regular where the
employee has been engaged to perform activities which are
usually necessary or desirable in the usual business or trade
of the employer, except where the employment has been
fixed for a specific project or undertaking the completion or
termination of which has been determined at the time of the
engagement of the employee or where the work or services
to be performed is seasonal in nature and the employment is
for the duration of the season.
- An employment shall be deemed to be casual if it is not
covered by the preceding paragraph: Provided, That, any
employee who has rendered at least one year of service,
whether such service is continuous or broken, shall be
considered a regular employee with respect to the activity in
which he is employed and his employment shall continue while
such activity exists."
- This provision of law comprehends three kinds of employees:
(a) regular employees or those whose work is necessary or
desirable to the usual business of the employer;
(b) project employees or those whose employment has
been fixed for a specific project or undertaking the
completion or termination of which has been determined at
the time of the engagement of the employee or where the
work or services to be performed is seasonal in nature and
the employment is for the duration of the season; and
(c) casual employees or those who are neither regular nor
project employees.
- For contractual employees, stipulations in employment
contracts providing for term employment or fixed period
employment are valid when
(1) the period were agreed upon knowingly and voluntarily by
the parties without force, duress or improper pressure being
brought to bear upon the employee and absent any other
circumstances vitiating his consent, or
(2) where it satisfactorily appears that the employer and
employee dealt with each other on more or less equal terms
with no moral dominance whatever being exercised by the
former over the latter.
- The contract (see facts) contains stipulations so ambiguous as
to preclude a precise application of pertinent labor laws. Since
contract of employment is contract of adhesion, ambiguity is
construed strictly against the party who prepared it. Also, Art.
1702 of CC provides that in case of doubt, all labor contracts
shall be construed in favor of the laborer. The interpretation
which the respondent company seeks to wiggle out is wholly
unacceptable, as it would result in a violation of petitioner's
right to security of tenure guaranteed in Section 3 of Article XIII
of the Constitution and in Articles 279 and 281 of the Labor
Code.
- Regardless of the designation petitioner may have conferred
upon respondent's employment status, it is, however,
uncontroverted that the latter, having completed the
probationary period and allowed to work thereafter, became a
regular employee who may be dismissed only for just or
authorized causes under Articles 282, 283 and 284 of the Labor
Code, as amended. Therefore, the dismissal, premised on the
alleged expiration of the contract, is illegal and entitles
respondent to the reliefs prayed for.
Disposition
The petition is hereby DISMISSED and the
decision of the National Labor Relations Commission dated
November 28. 1994 is AFFIRMED.

PANGILINAN V GENERAL MILLING CORPORATION


434 SCRA 159
CALLEJO, SR; July 12, 2004
NATURE

Labor Law 1
Petition for review on certiorari of a decision of the Court of
Appeals
FACTS
- Respondent General Milling Corporation is a domestic
corporation engaged in the production and sale of livestock and
poultry. It is, likewise, the distributor of dressed chicken to
various restaurants and establishments nationwide.
- Petitioners were employed by the respondent as emergency
workers under separate "temporary/casual contracts of
employment" for a period of five months.
- Upon the expiration of their respective contracts, their
services were terminated.
- They later filed separate complaints for illegal dismissal and
non-payment of holiday pay, 13th month pay, night-shift
differential and service incentive leave pay against the
respondent before the Arbitration Branch of the National Labor
Relations Commission,
- Petitioners alleged that their work as chicken dressers was
necessary and desirable in the usual business of the
respondent, and added that although they worked from 10:00
p.m. to 6:00 a.m., they were not paid night-shift differential.
- They stressed that based on the nature of their work, they
were regular employees of the respondent; hence, could not be
dismissed from their employment unless for just cause and
after due notice.
- Labor Arbiter Voltaire A. Balitaan rendered a decision in favor
of the petitioners declaring that they were regular employees.
- Finding that the termination of their employment was not
based on any of the just causes provided for in the Labor Code,
the Labor Arbiter declared that they were allegedly illegally
dismissed.
- On May 25, 1998, the NLRC rendered a decision reversing that
of the Labor Arbiter
- The NLRC held that the petitioners, who were temporary or
contractual employees of the respondent, were legally
terminated upon the expiration of their respective contracts.
Citing the case of Brent School, Inc. vs. Zamora, the NLRC
explained that while the petitioners' work was necessary and
desirable in the usual business of GMC, they cannot be
considered as regular employees since they agreed to a fixed
term.
- The petitioners' motion for reconsideration of the decision
having been denied by the NLRC, they filed a petition for
certiorari before the Court of Appeals.
- On September 29, 2000, the CA rendered a decision affirming
decision of the NLRC
- The CA ruled that where the duties of the employee consist of
activities usually necessary or desirable in the usual business of
the employer, it does not necessarily follow that the parties are
forbidden from agreeing on a period of time for the
performance of such activities.
- Petitioners MFR was denied, hence, this petition
ISSUE
WON the petitioners were regular employees of the respondent
GMC when their employment was terminated
HELD
NO
- Petitioners were employees with a fixed period, and, as such,
were not regular employees.
- Article 280 of the Labor Code comprehends three kinds of
employees: (a) regular employees or those whose work
is necessary or desirable to the usual business of the
employer; (b) project employees or those whose
employment has been fixed for a specific project or
undertaking the completion or termination of which has
been determined at the time of the engagement of the
employee or where the work or services to be performed
is seasonal in nature and the employment is for the

A2010

- 86 -

Disini

duration of the season; and, (c) casual employees or


those who are neither regular nor project employees.
- A regular employee is one who is engaged to perform
activities which are necessary and desirable in the usual
business or trade of the employer as against those which are
undertaken for a specific project or are seasonal.
There are two separate instances whereby it can be
determined that an employment is regular: (1) if the particular
activity performed by the employee is necessary or desirable in
the usual business or trade of the employer; and, (2) if the
employee has been performing the job for at least a year.
- In the case of St. Theresa's School of Novaliches Foundation
vs. NLRC, we held that Article 280 of the Labor Code does
not proscribe or prohibit an employment contract with a
fixed period. We furthered that it does not necessarily
follow that where the duties of the employee consist of
activities usually necessary or desirable in the usual
business of the employer, the parties are forbidden from
agreeing on a period of time for the performance of such
activities. There is thus nothing essentially contradictory
between a definite period of employment and the nature
of the employee's duties.
- In the case of Brent School Inc. v. Zamora, the SC laid down
the guideline before a contract of employment may be held as
valid, to wit: Stipulations in employment contracts providing for
term employment or fixed period employment are valid when
the period were agreed upon knowingly and voluntarily by the
parties without force, duress or improper pressure, being
brought to bear upon the employee and absent any other
circumstances vitiating his consent, or where it satisfactorily
appears that the employer and employee dealt with each other
on more or less equal terms with no moral dominance whatever
being exercised by the former over the latter.
- An examination of the contracts entered into by the
petitioners showed that their employment was limited to a fixed
period, usually five or six months, and did not go beyond such
period.
- The records reveal that the stipulations in the employment
contracts were knowingly and voluntarily agreed to by the
petitioners without force, duress or improper pressure, or any
circumstances that vitiated their consent. Similarly, nothing
therein shows that these contracts were used as a subterfuge
by the respondent GMC to evade the provisions of Articles 279
and 280 of the Labor Code.
- The petitioners were hired as "emergency workers" and
assigned as chicken dressers, packers and helpers at the Cainta
Processing Plant.
- While the petitioners' employment as chicken dressers is
necessary and desirable in the usual business of the
respondent, they were employed on a mere temporary basis,
since their employment was limited to a fixed period. As such,
they cannot be said to be regular employees, but are merely
"contractual employees."
- Consequently, there was no illegal dismissal when the
petitioners' services were terminated by reason of the
expiration of their contracts.
- Lack of notice of termination is of no consequence, because
when the contract specifies the period of its duration, it
terminates on the expiration of such period. A contract for
employment for a definite period terminates by its own term at
the end of such period.
Disposition Petition is denied.

DE LEON V NLRC (LA TONDENA)


176 SCRA 615
FERNAN; August 21, 1989
NATURE
Petition for certiorari seeking to annul and set aside: (1)
majority decision of the NLRC, which reversed the Order of

Labor Law 1
Labor Arbiter Hernandez; and, (2) the Resolution denying
petitioner's MFR
FACTS
- DE LEON was employed by LA TONDENA (business of
manufacture and distillery of wines and liquors) on Dec 11,
1981, at the Maintenance Section of its Engineering Dept in
Tondo.
- His work consisted mainly of painting company building and
equipment, and other odd jobs relating to maintenance. He was
paid on a daily basis through petty cash vouchers.
- After service of more than 1 year, DE LEON requested that he
be included in the payroll of regular workers. LA TONDENA
responded by dismissing him from work.
- Weeks after this, he was re-hired indirectly through the VitasMagsaysay Village Livelihood Council, a labor agency of
respondent, and was made to perform tasks he used to do.
- Having been refused reinstatement despite repeated
demands, petitioner filed a complaint before the Office of the
Labor Arbiter.
- LA TONDENA claimed he was a casual worker hired only to
paint a certain bldg in the company premises, and such work
terminated upon completion of the painting job.
- Labor Arbiter Hernandez ordered reinstatement and payment
of backwages to petitioner. Complainant's being hired on
casual basis did not dissuade from the cold fact that such jobs
he performed related to maintenance as a maintenance man is
necessary and desirable to the better operation of the business
company.
- On appeal, NLRC reversed such decision because his job
cannot be considered necessary in the usual trade of employer:
"Painting the business or factory building is not a part of the
respondent's manufacturing or distilling process of wines and
liquors.
ISSUE
WON petitioner is a regular employee

HELD
1. YES
Ratio An employment shall be deemed to be casual if it is not
covered by Art.2816 of Labor Code: provided, That any
employee who has rendered at least one year of service,
whether such service is continuous or broken, shall be
considered a regular employee with respect to the activity in
which he is employed and his employment shall continue while
such actually exists.
Reasoning
- During petitioner's period of employment, the records reveal
that the tasks assigned to him included not only painting of
company buildings, equipment and tools but also cleaning and
oiling machines, even operating a drilling machine, and other
odd jobs assigned to him when he had no painting job.
- It is not the will and word of the employer that determines
whether a certain employment is regular or casual, to which the
desperate worker often accedes, but the nature of the activities
performed in relation to the particular business or trade
considering all circumstances, and in some cases the length of
time of its performance and its continued existence.
Disposition Petition is GRANTED.
6

Art. 281. Regular and casual employment. The provisions of a written agreement to
the contrary notwithstanding and regardless of the oral agreements of the parties, an
employment shall be deemed to be regular where the employee has been engaged
to perform activities which are usually necessary or desirable in the usual business or
trade of the employer, except where the employment has been fixed for a specific
project or undertaking the completion or termination of which has been determined
at the time of the engagement of the employee or where the work or services to be
performed is seasonal in nature and the employment is for the duration of the
season.

A2010

- 87 -

Disini

SAN MIGUEL CORPORATION V NLRC (GUZMAN)


297 SCRA 277
QUISUMBING; October 7, 1998
NATURE
Petition for certiorari.
FACTS
- In November 1990, Francisco De Guzman, JR. was hired by
SMC as helper/bricklayer for a specific project, the repair and
upgrading of furnace C at its Manila Glass Plant. His contract of
employment provided that said temporary employment was for
a specific period of approximately 4 months. On April 30, 1991,
De Guzman was able to complete the repair and upgrading of
furnace C. Thus, his services were terminated on that same day
as there was no more work to be done. His employment
contract also ended that day.
- On May 10, 1991, De Guzman was again hired for a specific
job which involved the draining/cooling down of fuenace F and
the emergency repair of furnace E. This project was for a
specific period of approximately 3 months. After the completion
of this task, at the end of July 1991, DE Guzman's services were
terminated.
- On Aug.1, 1991, complainant saw his name in a Memorandum
posted at the Company's Bulletin Board as among those who
were considered dismissed.
- On Aug.12, 1994, or after the lapse of more than 3 years from
the completion of the last undertaking for which De Guzman
was hired, he filed a complaint for illegal dismissal against SMC.
- On June 30, 1995, labor Arbiter Felipe Garduque II rendered
the decision dismissing said complaint for lack of merit,
sustaining SMC's argument that DE Guzman was a project
employee. The position of a helper does not fall within the
classification of regular employees. Hence, complainant never
attained regular employment status. Moreover, his silence for
more than three (3) years without any reasonable explanation
tended to weaken his claim.
Upon appeal, NLRC reversed Labor Arbiter Garduque's
decision. In its ruling, NLRC stated that SMCs scheme of
subsequently re-hiring complainant after only 10 days from the
last day of the expiration of his contract of employment for a
specific period, and giving him again another contract of
employment
for
another
specific
period
cannot
be
countenanced. This is one way of doing violence to the
employee's constitutional right to security of tenure under
which even employees under probationary status are amply
protected.
- SMCs MFR was denied by NLRC. Hence, this petition.
ISSUES
1. WON De Guzman is a regular employee
2. WON De Guzman was illegally dismissed
HELD
1. NO
Art. 280 of the Labor Code defines regular, project and casual
employment as follows:
An employment shall be deemed to be regular where the
employee has been engaged to perform activities which are
usually necessary or desirable in the usual business or trade
of the employer, except where the employment has been
fixed for a specific project or undertaking the completion or
termination of which has been determined at the time of the
engagement of the employee or where the work or services
to be performed is seasonal in nature and the employment is
for the duration of the season.
An employment shall be deemed to be casual if it is not
covered by the preceding paragraph: Provided, That, any
employee who has rendered at least one year of service,
whether such service is continuous or broken, shall be

Labor Law 1
considered a regular employee with respect to the activity in
which he is employed and his employment shall continue
while such actually exists.
- The above mentioned provision reinforces the Constitutional
mandate to protect the interest of labor as it sets the legal
framework for ascertaining one's nature of employment, and
distinguishing different kinds of employees. Its language
manifests the intent to safeguard the tenurial interest of worker
who may be denied the enjoyment of the rights and benefits
due to an employee, regardless of the nature of his
employment, by virtue of lopsided agreements which the
economically powerful employer who can maneuver to keep an
employee on a casual or contractual status for as long as it is
convenient to the employer.
- Thus, under Article 280 of the Labor Code, an employment is
deemed regular when the activities performed by the employee
are usually necessary or desirable in the usual business or trade
of the employer even if the parties enter into an agreement
stating otherwise. But considered not regular under said Article
(1) the so-called "project employment" the termination of which
is more or less determinable at the time of employment, such
as those connected, which by its nature is only for one season
of the year and the employment is limited for the duration of
that season, such as the Christmas holiday season.
Nevertheless, an exception to this exception is made: any
employee who has rendered at least 1 year of service, whether
continuous or intermitent, with respect to the activity he
performed and while such activity actually exists, must be
deemed regular.
- Following Article 280, whether one is employed as a project
employee or not would depend on whether he was hired to
carry out a "specific project or undertaking", the duration and
scope of which were specified at the time his services were
engaged for that particular project. Another factor that may be
undertaken by the employee in relation to the usual trade or
business of the employer, if without specifying the duration and
scope, the work to be undertaken is usually necessary or
desirable in the usual business or trade of the employer, then it
is regular employment and not just "project" must less "casual"
employment.
- Thus, the nature of one's employment does not depend on
the will or word of the employer. Nor on the procedure of hiring
and the manner of designating the employee, but on the nature
of the activities to be performed by the employee, considering
the employer's nature of business and the duration and scope
of the work to be done.
- Project could refer to 2 distinguishable types of activity.
Firstly, a project could refer to a particular job or undertaking
that is within the regular or usual business of the employer
company, but which is distinct at separate, and identifiable as
such, from the other undertakings of the company. Such job or
undertaking begins and ends at determined or determinable
times. . . . Secondly, a project could refer to a particular job or
undertaking that is not within the regular business of the
corporation. Such a job or undertaking must also be identifiably
separate and distinct from the ordinary or regular business
operations of the employer. The job or undertaking also begins
and ends at determined or determinable times . .
- The plant where De Guzman was employed for only 7 months
is engaged in the manufacturer of glass, an integral component
of the packaging and manufacturing business of petitioner. The
process of manufacturing glass requires a furnace, which has a
limited operating life. SMC resorted to hiring project or fixed
term employees in having said furnaces repaired since said
activity is not regularly performed. Said furnaces are to be
repaired or overhauled only in case of need and after being
used continuously for a varying period of 5-10 years. In 1990,
one of the furnaces of petitioner required repair and upgrading.
This was an undertaking distinct and separate from SMC's
business of manufacturing glass. For this purpose, SMC must
hire workers to undertake the said repair and upgrading. De
Guzman was, thus, hired by SMC on November 28, 1990 on a

A2010

- 88 -

Disini

"temporary status for a specific job" for a determined period of


approximately four months
- Upon completion of the undertaking, or on April 30, 1991, DE
Guzman's services were terminated. A few days, thereafter, two
of SMC's furnaces required "draining/coolong down" and
"emergency repair". De Guzman was again hired on May 10,
1991 to help in the new undertaking, which would take
approximately 3 months to accomplish. Upon completion of the
second undertaking, private respondent's services were
likewise terminated. He was not hired a third time, and his two
engagements taken together did not total one full year in order
to qualify him as an exception to the exception falling under the
cited proviso in the second paragraph of Art. 280 of the Labor
Code.
2. NO
- De Guzman was hired for a specific project that was not within
the regular business of the corporation. For SMC is not engaged
in the business of repairing furnaces. Although the activity was
necessary to enable petitioner to continue manufacturing glass,
the necessity therefor arose only when a particular furnace
reached the end of its life or operating cycle. Or, as on the
second undertaking, when a particular furnace required an
emergency repair. In other words, the undertakings where he
was hired primarily as helper/bricklayer have specified goals
and purpose which are fulfilled once the designated work was
completed. Moreover, such undertakings were also identifiably
separate and distinct from the usual, ordinary or regular
business operations of petitioner, which is glass manufacturing.
These undertakings, the duration and scope of which had been
determined and made known to private respondent at the time
of his employment clearly indicated the nature of his
employment as a project employee. Thus, his services were
terminated legally after the completion of the project.
- If NLRCs decision is upheld, it would amount to negating the
distinction made in Article 280 of the Labor Code. It would shunt
aside the rule that since a project employee's work defends on
the availability of a project, necessarily, the duration of his
employment is coterminous with the project to which he is
assigned. It would become a burden for an employer to retain
an employee and pay him his corresponding wages it there was
no project for him to work on.
- While the Constitution is committed to the policy of social
justice and the protection of the working class, it should not be
supposed that every dispute will be automatically decided in
favor of labor. Management has also rights, which, as such, are
entitled to respect and enforcement in the interest of fair play.
Although the SC has inclined more often than not toward the
worker and has upheld has cause in his conflicts with the
employer, such favoritism has no blinded the Court to the rule
that justice is in avery case for the deserving, to be dispensed
in the light of the established facts and the applicable law and
doctrine.
Disposition
Petition is hereby GRANTED. The decision of
respondent NLRC is hereby REVERSED, and the judgment of the
Labor Arbiter REINSTATED.

TABAS V NLRC (CALIFORNIA MANUFACTURING)


169 SCRA 497
SARMIENTO; January 26, 1989
NATURE
PETITION to review the decision and resolution of the National
Labor Relations Commission.
FACTS
- Petitioners were employees of Livi Manpower Services, Inc.
(Livi). Livi subsequently assigned them to work as promotional
merchandisers for California Manufacturing Co. (California)
pursuant to a manpower supply agreement. The agreement
provided the following, among others: (1) that California had no
control/supervision over the petitioners with respect to how

Labor Law 1
they accomplish their work; (2) that Livi is an independent
contractor and that the relationship between Livi and California
should not be construed to be of principal-agent or employeremployee; (3) that California is free and harmless (!?!) from any
liability arising from such laws or from any accident that may
befall the workers and employees of Livi while in the
performance of their duties for California; (4) that the
assignment of workers to California shall be on a seasonal and
contractual basis; (5) that most of living allowance and the 10
legal holidays will be charged directly to California at cost; and
(6) that the payroll for the preceding week shall be delivered by
LIvi at Californias premises.
- Petitioners were then made to sign employment contracts with
durations of six months, upon the expiration of which
they signed new agreements with the same period, and
so on. Unlike regular California employees, who received not
less than P2,823.00 a month in addition to a host of fringe
benefits and bonuses, they received P38.56 plus P15.00 in
allowance daily.
- Petitioners filed complaints, demanding to have similar
benefits as regular employees; but pending their claims,
California notified them that they would not be rehired.
California then amended their complaint charging California
with illegal dismissal. Thereafter, Livi reabsorbed them into its
labor pool on a wait-in or standby status.
- Respondents claim: they are not the petitioners employer
(Livi is, therefore, no employer-employee relationship between
them) and that the "retrenchment" had been forced by
business losses as well as expiration of contracts
("unfavorable political and economic atmosphere coupled by
the February Revolution.")
- LA: no employer-employee relationship in the light of the
manpower supply contract; California not liable for the money
claims demanded. Livi also absolved from any obligation
because retrenchment was allegedly beyond its control, but
were to pay separation pay and attorneys fees.
- NLRC: affirm labor arbiters deci
ISSUES
1. WON the petitioners are employees of California
Manufacturing Company
2. WON the petitioners were illegally dismissed
HELD
1. YES
Ratio The existence of an employer-employees relation is a
question of law and being such, it cannot be made the subject
of agreement. The determination of whether or not there is an
employer-employee relation depends upon four standards: (1)
the manner of selection and engagement of the putative
employee; (2) the mode of payment of wages; (3) the presence
or absence of a power of dismissal; and (4) the presence or
absence of a power to control the putative employee's conduct.
Of the four, the right-of-control test has been held to be the
decisive factor.
Reasoning
- IN RELATION TO THE MANPOWER SUPPLY AGREEMENT: The fact that the
manpower supply agreement between Livi and California had
specifically designated the former as the petitioners' employer
and had absolved the latter from any liability as an employer,
will not erase either party's obligations as an employer, if an
employer-employee relation otherwise exists between the
workers and either firm. At any rate, since the agreement was
between Livi and California, they alone are bound by it, and the
petitioners cannot be made to suffer from its adverse
consequences.

A2010

- 89 -

Disini

- Art. 106 of the Labor Code7 still imposes responsibility on


both firms: Notwithstanding the absence of a direct employeremployee relationship between the employer in whose favor
work had been contracted out by a "labor-only" contractor, and
the employees, the former has the responsibility, together with
the "labor-only" contractor, for any valid labor claims by
operation of law. The reason, so we held, is that the "labor-only"
contractor is considered "merely an agent of the employer,"
and liability must be shouldered by either one or shared by
both.
- Livi, as a placement agency, had simply supplied California
with the manpower necessary to carry out Californias
merchandizing activities, using the latters premises and
equipment.
- ON PETITIONERS BEING DIRECT EMPLOYEES OF LIVI: Not conclusive
will not absolve California from liability imposed by law and
relations of parties are not determined by their declarations
- ON TEMPORARY OR SEASONAL BASIS HIRING: temporary or casual
employee, under Article 218 of the Labor Code, becomes
regular after service of one year, unless he has been contracted
for a specific project. Merchandising is not a specific project, it
is an activity related to the day-to-day operations of California.
*The Court need not therefore consider whether it is Livi or
California which exercises control over the petitioner vis-a-vis
the four barometers reffered to earlier, since by fiction of law,
either or both shoulder responsibility.
-REASONING FOR DISMISSING THE TERMS AND CONDITIONS OF THE MANPOWER
SUPPLY AGREEMENT: not illegal, under the Labor Code, genuine
job contracts are permissible, provided they are genuine
job contracts. But when such arrangements are resorted to "in
anticipation of, and for the very purpose of making possible, the
secondment"of the employees from the true employer, the
Court will be justified in expressing its concern. For then that
would compromise the rights of the workers, especially their
right to security of tenure.
2. YES
Ratio Retrenchment of workers, unless clearly warranted, has
serious consequences not only on the State's initiatives to
maintain a stable employment record for the country, but more
so, on the workingman himself, amid an environment that is
desperately scarce in jobs.
Reasoning The 6-month contracts of the petitioners were
renewed, and accordingly, under Article 281 (Labor Code), they
had become regular employees of California and had acquired a
secure tenure. Hence, they cannot be separated without due
process of law.
- ON VALIDITY OF RETRENCHMENT: California has not shown enough
evidence that it had in fact suffered serious business reverses
as a result alone of the prevailing political and economic
climate; attribution to February Revolution as cause of alleged
losses gratuitous and without basis in fact.
Disposition
petition is GRANTED. Judgment is hereby
RENDERED: (1) SETTING ASIDE the decision, dated March 20,
7

ART. 106. Contractor or subcontractor.-Whenever an employee enters into a


contract with another person for the performance of the former's work, the
employees of the contractor and of the letter's subcontractor, if any, shall be paid
in accordance with the provisions of this Code.
In the event that the contractor or subcontractor fails to pay wages of his
employees in accordance with this Code, the employer shall be jointly and
severally liable with his contractor or subcontractor to such employees to the
extent of the work performed under the contract, in the same manner and extent
that he is liable to employees directly employed by him.
The Secretary of Labor may, by appropriate regulations, restrict or prohibit the
contracting out of labor to protect the rights of workers established under this
Code. In so prohibiting or restricting, he may make appropriate distinctions
between labor-only contracting and job contracting as well as differentiations
within these types of contracting and determine who among the parties involved
shall be considered the employer for purposes of this Code, to prevent any
violation or circumvention of any provisions of this Code.
There is "labor-only" contracting where the person supplying workers to an
employer does not have substantial capital or investment in the form of tools,
equipment, machineries, work premises, among others, and the workers recruited
and placed by such person are performing activities which are directly related to
the principal business of such employer. In such cases, the person or intermediary
shall be considered merely as an agent of the employer who shall be responsible to
the workers in the same manner and extent as if the latter were directly employed
by him.

Labor Law 1
1987, and the resolution, dated August 19, 1987; (2) ORDERING
the respondent, the California Manufacturing Company, to
REINSTATE the petitioners with fall status and rights of regular
employees; and (3) ORDERING the respondent, the California
Manufacturing Company, and the respondents, Livi Manpower
Service, Inc. and/or Lily-Victoria A. Azarcon, to PAY, jointly and
severally, unto the petitioners: (a) backwages and differential
pays effective as and from the time they had acquired a regular
status under the second paragraph, of Section 281, of the Labor
Code, but not to exceed three (3) years, and (b) all such other
and further benefits as may be provided by existing collective
bargaining agreement(s) or other relations, or by law, beginning
such time; and (4) ORDERING the private respondents to PAY
unto the petitioners attorney's fees equivalent to ten (10%)
percent of all money claims hereby awarded, in addition to
those money claims.

PHILIPS SEMICONDUCTORS V FADRIQUELA


427 SCRA 408
CALLEJO, SR; April 14, 2004
FACTS
- On May 8, 1992, respondent Eloisa Fadriquela executed a
Contract of Employment with petitioner Philips Semiconductors
as a production operator, initially for 3 months. Because her
performance constantly met petitioners ratings requirements,
her contract was renewed several times, extending to 12
months. However, over the last few months, respondent
incurred several absences for which she offered no valid
justification despite a prompting to do so by the line supervisor.
As a consequence, her performance rating dropped, and
respondents contract was no longer renewed.
- Respondent filed a complaint with the NLRC for illegal
dismissal, claiming she had not been duly notified; she
furthered that having rendered over 6 months of service, she
was already a regular employee and could not be terminated
without just cause.
- Petitioner contended that respondent had not been dismissed;
rather, her contract merely expired and was not renewed.
- The Labor Arbiter dismissed the complaint for lack of merit but
awarded her severance of 1 months pay. He stated that
petitioner and its unions CBA required one to render 17 months
of service to be considered regular. He also added that
respondent could not complain of being deprived of notice and
hearing as the line supervisor had asked her to explain her
absences. An appeal with the NLRC yielded the same results. It
was pointed out that as a contractual employee respondent was
bound by the stipulations of her contract of employment, which
in this case was a satisfactory performance rating.
- Dissatisfied, respondent filed a petition for certiorari before
the CA, which reversed the decisions of the NLRC and the Labor
Arbiter. The appellate court argued that the NLRC and the Labor
Arbiter employed inappropriate bases for their decisions, since
the CBA did not apply to contractual employees like Fadriquela.
The CA cited Art. 280 of the Labor Code which states that
regardless of any written or oral agreements between employer
and employee, an employment shall be deemed to be regular
where the employee has been engaged to perform activities
which are usually necessary or desirable in the usual business
or trade of the employer. Petitioners contention that
employment was obtained as the need arose was illogical, as
this would mean the employee would never attain regular
status. The CA further held that a less punitive penalty would
suffice for absenteeism. Finally, it held that the dialogue
between the respondent and line supervisor was insufficient as
to amount to notice, and thus the former was deprived of due
process.
- Petitioner filed a motion for reconsideration in which petitioner
claimed that its hiring policy was neither new nor prohibited
and that it was a valid exercise of its management prerogative
since demand for its semiconductors is cyclical in nature. It

A2010

- 90 -

Disini

added that it had the prerogative to set reasonable standards of


employment qualification as provided by law. The motion was
denied, hence this petition for review.
ISSUES
1. WON the NLRC and Labor Arbiter erred in not finding
respondent to be a regular employee
2. WON the CBA applies to respondent
3. WON respondent was deprived of due process
4. WON dismissal was a just penalty
HELD
1. YES
- According to Article 280 of the Labor Code, there are 2 kinds
of regular employees: (1) those engaged to perform activities
which are necessary or desirable in the usual business or trade
of the employer; and (2) those casual employees who have
rendered at least one year of service, whether continuous or
broken, with respect to the activities in which they are
employed. The respondent obviously falls under the first type of
regular employee. She had been working continuously for the
petitioner for over a year, evidencing the necessity and
indispensability of her services to the petitioners business. By
operation of law, respondent had attained regular status and
was thus entitled to security of tenure as provided in Art. 279 of
the code. The said article requires a just cause before
termination, and entitles the employee to reinstatement and
other privileges in absence of one.
- Petitioners hiring policy for contract employees is contrary to
the spirit of Articles 279 and 280 of the code; it is but an excuse
to prevent regularization and circumvent the law on security of
tenure. This is echoed in Sec. 3 Art XVI of the Constitution which
deems security of tenure a State policy to guarantee social
justice. The fact is that the operation of every business depends
on supply and demandthe cyclical nature of ones trade
cannot be invoked as a reason to place an employees status on
shaky ground.
- This is not to say that term employment is illegal outright. In
Romares v NLRC it was said that term employment does not
circumvent the law when the fixed period was knowingly and
voluntarily agreed upon by both parties and that such
agreement was made with no party holding moral dominance
over the other. However, none of these requisites are present in
the instant case.
2. NO
- Petitioners reliance on the CBA is misplaced. The CBA
constitutes the law between the employer and regular
employees, but cannot be binding on contractual employees
who are not represented by the bargaining union. The CBA
provision requiring 17 months for regularization runs contrary to
what is clearly stipulated in law, which provides that
regularization requires only 1 year.
3. YES
- Respondent was dismissed without the requisite notice and
formal investigation. Dismissals must not be arbitrary and
capricious; a mere dialogue between the respondent and line
supervisor cannot possibly suffice as a substitute for actual
notice and hearing.
4. NO
- Dismissal is too harsh a penalty for mere absences, especially
since the repeated renewal of respondents contract proves her
efficiency as a worker. The SC mandates that where a penalty
less punitive would suffice, whatever missteps may be
committed by labor ought not to be visited with a consequence
so severe.
Disposition IN LIGHT OF ALL THE FOREGOING, the assailed
decision of the appellate court is AFFIRMED. The petition at bar
is DENIED.

MAGSALIN V NATIONAL ORGANIZATION OF


WORKING MEN (NOWM)

Labor Law 1
403 SCRA 199
VITUG; May 9, 2003
FACTS
- Coca-Cola Bottlers Phils., Inc., herein petitioner, engaged the
services of respondent workers as sales route helpers for a
limited period of five months. After five months, respondent
workers were employed by petitioner company on a day-to-day
basis to substitute for regular sales route helpers whenever the
latter would be unavailable or when there would be an
unexpected shortage of manpower in any of its work places or
an unusually high volume of work. The practice was for the
workers to wait every morning outside the gates of the sales
office of petitioner company. If thus hired, the workers would
then be paid their wages at the end of the day.
- Ultimately, respondent workers asked petitioner company to
extend to them regular appointments. Petitioner company
refused.
- November 7, 1997 - twenty-three (23) of the temporary
workers (herein respondents) filed with the National Labor
Relations Commission (NLRC) a complaint for the
regularization of their employment with petitioner
company. The complaint was amended a number of times to
include other complainants that ultimately totaled fifty-eight
(58) workers. Claiming that petitioner company meanwhile
terminated their services, respondent workers filed a notice of
strike and a complaint for illegal dismissal and unfair labor
practice with the NLRC.
- 01 April 1998 - voluntary arbitration
- 18 May 1998 - the voluntary arbitrator rendered a decision
dismissing the complaint on the thesis that respondents (then
complainants) were not regular employees of petitioner
company.
- 11 August 2000, the Court of Appeals reversed and set
aside the ruling of the voluntary arbitrator ; (Petitioners were
declared regular employees of Coca Cola Bottlers; dismissal
illegal; ordered to reinstate the workers)
ISSUES
1. WON the nature of work of respondents in the company is of
such nature as to be deemed necessary and desirable in the
usual business or trade of petitioner that could qualify them to
be regular employees
2.
WON the quitclaims executed by the 36 individual
respondents were valid
HELD
1. YES
Ratio
In determining whether an employment should be
considered regular or non-regular, the applicable test is the
reasonable connection between the particular activity
performed by the employee in relation to the usual business or
trade of the employer.
Reasoning
a. Intentionalist approach - Even while the language of law (Art
280)8 might have been more definitive, the clarity of its spirit
and intent, i.e., to ensure a regular workers security of
tenure, however, can hardly be doubted.
b. Although the work to be performed is only for a specific
8

Art. 280.
Regular and Casual Employment. The provisions of
written agreement to the contrary notwithstanding and regardless of the oral
agreement of the parties, an employment shall be deemed to be regular
where the employee has been engaged to perform activities which are usually
necessary or desirable in the usual business or trade of the employer, except
where the employment has been fixed for a specific project or undertaking the
completion or termination of which has been determined at the time of the
engagement of the employee or where the work or services to be performed
is seasonal in nature and the employment is for the duration of the season.
An employment shall be deemed to be casual if it is not covered by the
preceding paragraph: Provided, That, any employee who has rendered at least
one year of service, whether such service is continuous or broken, shall be
considered a regular employee with respect to the activity in which he is
employed and his employment shall continue while such activity exists.

A2010

- 91 -

Disini

project or seasonal, where a person thus engaged has been


performing the job for at least one year, even if the
performance is not continuous or is merely intermittent, the law
deems the repeated and continuing need for its performance as
being sufficient to indicate the necessity or desirability of that
activity to the business or trade of the employer.
The
employment of such person is also then deemed to be regular
with respect to such activity and while such activity exists.
c. The postproduction activities done by sales route helpers
are important. The nature of the work performed must be
viewed from a perspective of the business or trade in its
entirety and not on a confined scope.
d.
The repeated rehiring of respondent workers and the
continuing need for their services clearly attest to the necessity
or desirability of their services in the regular conduct of the
business or trade of petitioner company.
e. A contract of employment is impressed with public interest.
The provisions of applicable statutes are deemed written into
the contract, and the parties are not at liberty to insulate
themselves and their relationships from the impact of labor
laws and regulations by simply contracting with each other.
2. YES
Ratio While quitclaims executed by employees are commonly
frowned upon as being contrary to public policy and are
ineffective to bar claims for the full measure of their legal
rights, there are, however, legitimate waivers that represent a
voluntary and reasonable settlement of laborers claims which
should be so respected by the Court as the law between the
parties. Where the person making the waiver has done so
voluntarily, with a full understanding thereof, and the
consideration for the quitclaim is credible and reasonable, the
transaction must be recognized as being a valid and binding
undertaking. Dire necessity is not an acceptable ground for
annulling the release, when it is not shown that the employee
has been forced to execute it.
Disposition Questioned decision of the Court of Appeals, is
AFFIRMED with MODIFICATION in that the Release, Waiver and
Quitclaim executed by the thirty-six (36) individual
respondents are hereby declared VALID and LEGAL.

HACIENDA FATIMA V NATIONAL FEDERATION OF


SUGARCANE WORKERS-FOOD AND
GENERAL TRADE
396 SCRA 518
PANGANIBAN; January 28, 2003
NATURE
Before the Court is a Petition for Review under Rule 45 of the
Rules of Court, seeking to set aside CA Decision denying
petition for certiorari the Decision of NLRC. NLRC set aside and
vacated the Labor Arbiters finding that there was no illegal
dismissal.
FACTS
- According to the Labor arbiter, the respondents refused to
work and/or were choosy in the kind of jobs they wanted to
perform. NLRC found that the record is replete with the workers
persistence and determination of going back to work.
- When the union was certified as the collective bargaining
representative in the certification elections, Hacienda Fatima
under the pretext that the result was on appeal, refused to sit
down with the union for the purpose of entering into a CBA.
Moreover, the workers were not given work for more than one
month. In protest, the Union staged a strike which was however
settled upon the signing of a Memorandum of Agreement.
- When Company again reneged on its commitment, Union filed
the complaint. For all their persistence, the risk they had to
undergo in conducting a strike, complainants now find
themselves being accused of refusing to work and being
choosy in the kind of work they have to perform.

Labor Law 1
- The CA affirmed that while the work of respondents was
seasonal in nature, they were considered to be merely on leave
during the off-season and were therefore still employed by
petitioners. Moreover, the workers enjoyed security of tenure.
Any infringement upon this right was deemed by the CA to be
tantamount to illegal dismissal. Hence this Petition.
ISSUES
1. WON CA erred in holding that respondents, admittedly
seasonal workers, were regular employees, contrary to the clear
provisions of Article 2809 of the Labor Code, which categorically
state that seasonal employees are not covered by the definition
of regular employees under paragraph 1, nor covered under
paragraph 2 which refers exclusively to casual employees who
have served for at least one year
2. WON CA committed grave abuse of discretion in upholding
the NLRCs conclusion that private respondents were illegally
dismissed, that petitioner[s were] guilty of unfair labor practice,
and that the union be awarded moral and exemplary damages.
HELD
1. NO, the CA did not err when it held that respondents were
regular employees.
- The fact that respondents do not work continuously for one
whole year but only for the duration of the season does not
detract from considering them in regular employment since in a
litany of cases this Court has already settled that seasonal
workers who are called to work from time to time and are
temporarily laid off during off-season are not separated from
service in said period, but merely considered on leave until reemployed.
- For respondents to be excluded from those classified as
regular employees, it is not enough that they perform work or
services that are seasonal in nature. They must have also been
employed only for the duration of one season. The evidence
proves the existence of the first, but not of the second,
condition. The fact that respondents -- with the exception of
Luisa Rombo, Ramona Rombo, Bobong Abriga and Boboy Silva
-- repeatedly worked as sugarcane workers for petitioners for
several years is not denied by the latter. Evidently, petitioners
employed respondents for more than one season. Therefore,
the general rule of regular employment is applicable.
- The test of WON an employee is a regular employee has
been laid down in De Leon v. NLRC, in which this Court held:
- The primary standard of determining regular employment is
the reasonable connection between the particular activity
performed by the employee in relation to the usual trade or
business of the employer. The test is whether the former is
usually necessary or desirable in the usual trade or business of
the employer.
The connection can be determined by
considering the nature of the work performed and its relation to
the scheme of the particular business or trade in its entirety.
Also if the employee has been performing the job for at least a
year, even if the performance is not continuous and merely
intermittent, the law deems repeated and continuing need for
its performance as sufficient evidence of the necessity if not
indispensability of that activity to the business. Hence, the
employment is considered regular, but only with respect to such
activity and while such activity exists. (Abasolo v. National
Labor Relations Commission)
9

Art. 280. Regular and Casual Employment. - The provisions of written agreement to
the contrary notwithstanding and regardless of the oral agreement of the parties, an
employment shall be deemed to be regular where the employee has been engaged
to perform activities which are usually necessary or desirable in the usual business or
trade of the employer, except where the employment has been fixed for a specific
project or undertaking the completion or termination of which has been determined
at the time of the engagement of the employee or where the work or services to be
performed is seasonal in nature and the employment is for the duration of the
season.
An employment shall be deemed to be casual if it is not covered by the preceding
paragraph: Provided, That, any employee who has rendered at least one year of
service, whether such service is continuous or broken, shall be considered a regular
employee with respect to the activity in which he is employed and his employment
shall continue while such activity exist.

A2010

- 92 -

Disini

- The sudden changes in work assignments reeked of bad faith.


These changes
were implemented
immediately
after
respondents had organized themselves into a union and started
demanding collective bargaining.
Those who were union
members were effectively deprived of their jobs. Petitioners
move actually amounted to unjustified dismissal of
respondents, in violation of the Labor Code.
2. NO
- Factual findings of labor officials, who are deemed to have
acquired expertise in matters within their respective
jurisdictions, are generally accorded not only respect but even
finality. Their findings are binding on the Supreme Court.
Verily, their conclusions are accorded great weight upon appeal,
especially
when
supported
by
substantial
evidence.
Consequently, the Court is not duty-bound to delve into the
accuracy of their factual findings, in the absence of a clear
showing that these were arbitrary and bereft of any rational
basis.
- The NLRC found herein petitioners guilty of unfair labor
practice. It ruled that from respondents refusal to bargain, to
their acts of economic inducements resulting in the promotion
of those who withdrew from the union, the use of armed guards
to prevent the organizers to come in, and the dismissal of union
officials and members, one cannot but conclude that
respondents did not want a union in their haciendaa clear
interference in the right of the workers to self-organization.
Disposition
Petition is hereby DENIED and the assailed
Decision AFFIRMED. Costs against petitioners.

MILLARES V NLRC (TRANS-GLOBAL MARITIME


AGENCY, ESSO INTERNATIONAL SHIPPING
COMPANY)
485 SCRA 307
KAPUNAN; July 29, 2002
FACTS
- Petitioner Douglas Millares was employed by private
respondent ESSO International Shipping Company LTD. (Esso
Int.) through its local manning agency, private respondent
Trans-Global Maritime Agency, Inc. (Trans-Global) as a
machinist. In 1975, he was promoted as Chief Engineer. He
was then receiving a monthly salary of US $1,939.00. On June
13, 1989, Millares applied for a leave of absence for the period
July 9 to August 7, 1989. Trans-global approved it. He then
informed Esso Int. of his intention to avail of the optional
retirement plan under the Consecutive Enlistment Incentive
Plan (CEIP) considering that he had already rendered more than
twenty (20) years of continuous service.
But the denied
petitioner Millares request for optional retirement on the
following grounds, to wit: (1) he was employed on a contractual
basis; (2) his contract of enlistment (COE) did not provide for
retirement before the age of sixty (60) years; and (3) he did not
comply with the requirement for claiming benefits under the
CEIP, i.e., to submit a written advice to the company of his
intention to terminate his employment within thirty (30) days
from his last disembarkation date. He then requested for an
extension of his leave of absence from August 9 to 24, 1989.
But the company told him that they have promoted a First
Assistant Engineer to his position as a result of his previous
leave of absence which expired last August 8, 1989. The
adjustment in said rank was required in order to meet
manpower schedules as a result of his inability.
Esso
International told Millares that in view of his absence without
leave, which is equivalent to abandonment of his position, he
had been dropped from the roster of crew members effective
September 1, 1989.
- On the other hand, petitioner Lagda was employed by private
respondent Esso International as wiper/oiler in June 1969. He
was promoted as Chief Engineer in 1980, a position he
continued to occupy until his last COE expired on April 10, 1989.
He also filed for a leave of absence and informed the company

Labor Law 1
of his intention to avail the early retirement. His request was
denied on the same grounds and he too was dropped from
work.
- On October 5, 1989, petitioners Millares and Lagda filed a
complaint-affidavit for illegal dismissal and non-payment of
employee
benefits
against
private
respondents
Esso
International and Trans-Global, before the POEA.
POEA
dismissed it for lack of merit. NLRC affirmed.
ISSUE
WON the petitioners are contractual employees whose
employment are terminated everytime their contracts expire
HELD
YES
- it is clear that seafarers are considered contractual
employees. They can not be considered as regular employees
under Article 280 of the Labor Code. Their employment is
governed by the contracts they sign everytime they are rehired
and their employment is terminated when the contract expires.
Their employment is contractually fixed for a certain period of
time. They fall under the exception of Article 280 whose
employment has been fixed for a specific project or undertaking
the completion or termination of which has been determined at
the time of engagement of the employee or where the work or
services to be performed is seasonal in nature and the
employment is for the duration of the season. We need not
depart from the rulings of the Court in the two aforementioned
cases which indeed constitute stare decisis with respect to the
employment status of seafarers. Petitioners make much of the
fact that they have been continually re-hired or their contracts
renewed before the contracts expired (which has admittedly
been going on for twenty (20) years). By such circumstance
they claim to have acquired regular status with all the rights
and benefits appurtenant to it.
- Such contention is untenable. Undeniably, this circumstance
of continuous re-hiring was dictated by practical considerations
that experienced crew members are more preferred.
Petitioners were only given priority or preference because of
their experience and qualifications but this does not detract the
fact that herein petitioners are contractual employees. They
can not be considered regular employees.
Disposition
IN VIEW OF THE FOREGOING, THE COURT
Resolved to Partially GRANT Private Respondents Second
Motion for Reconsideration and Intervenor FAMES Motion for
Reconsideration in Intervention. The Decision of the National
Labor Relations Commission dated June 1, 1993 is hereby
REINSTATED with MODIFICATION. The Private Respondents,
Trans-Global Maritime Agency, Inc. and Esso International
Shipping Co.,Ltd. are hereby jointly and severally ORDERED to
pay petitioners One Hundred Percent (100%) of their total
credited contributions as provided under the Consecutive
Enlistment Incentive Plan(CEIP).

PETROLEUM SHIPPING LIMITED V NLRC (TANCHICO)


491 SCRA 35
CARPIO; June 16, 2006
FACTS
- On 6 March 1978, Esso International Shipping (Bahamas) Co.,
Ltd., ("Esso") through Trans-Global Maritime Agency, Inc.
("Trans-Global") hired Florello W. Tanchico ("Tanchico") as First
Assistant Engineer. In 1981, Tanchico became Chief Engineer.
- On 13 October 1992, Tanchico returned to the Philippines for a
two-month vacation after completing his eight-month
deployment.
- On 8 December 1992, Tanchico underwent the required
standard medical examination prior to boarding the vessel. The
medical examination revealed that Tanchico was suffering from
"Ischemic Heart Disease, Hypertensive Cardio-Muscular Disease
and Diabetes Mellitus." Tanchico took medications for two

A2010

- 93 -

Disini

months and a subsequent stress test showed a negative result.


However, Esso no longer deployed Tanchico. Instead, Esso
offered to pay him benefits under the Career Employment
Incentive Plan. Tanchico accepted the offer.
- On 26 April 1993, Tanchico filed a complaint against Esso,
Trans-Global and Malayan Insurance Co., Inc. ("Malayan")
before the Philippine Overseas Employment Administration
(POEA) for illegal dismissal with claims for backwages,
separation pay, disability and medical benefits and 13th month
pay.
ISSUES
1. WON Tanchico is a regular employee of petitioners
2. WON Tanchico is entitled to 13th month pay, disability
benefits and attorneys fees
HELD
1. NO
- The Court squarely passed upon the issue in Millares v. NLRC17
where one of the issues raised was whether seafarers are
regular or contractual employees whose employment are
terminated everytime their contracts of employment expire.
The Court explained:
[I]t is clear that seafarers are considered contractual
employees. They can not be considered as regular employees
under Article 280 of the Labor Code. Their employment is
governed by the contracts they sign everytime they are
rehired and their employment is terminated when the
contract expires. Their employment is contractually fixed for
a certain period of time. They fall under the exception of
Article 280 whose employment has been fixed for a specific
project or undertaking the completion or termination of which
has been determined at the time of engagement of the
employee or where the work or services to be performed is
seasonal in nature and the employment is for the duration of
the season. We need not depart from the rulings of the Court
in the two aforementioned cases which indeed constitute
stare decisis with respect to the employment status of
seafarers.
- The circumstance of continuous re-hiring was dictated by
practical considerations that experienced crew members are
more preferred. Petitioners were only given priority or
preference because of their experience and qualifications but
this does not detract the fact that herein petitioners are
contractual employees. They can not be considered regular
employees.
2. On 13th Month Pay
- The Court of Appeals premised its grant of 13th month pay on
its ruling that Tanchico was a regular employee. The Court of
Appeals also ruled that petitioners are not exempt from the
coverage of PD 851 which requires all employers to pay their
employees a 13th month pay.
- We do not agree with the Court of Appeals. Again, Tanchico
was a contractual, not a regular, employee. Further, PD 851
does not apply to seafarers.
- Tanchicos employment is governed by his Contract of
Enlistment. The Contract has been approved by the POEA in
accordance with Title I, Book One of the Labor Code and the
POEA Rules Governing Employment. Hence, in the absence of
any provision in his Contract governing the payment of 13th
month pay, Tanchico is not entitled to the benefit.
On Disability Benefits
- Since Tanchico received compensation during his vacation, the
Contract did not terminate on the day he returned to Manila.
The Contract remained in force during Tanchicos vacation
period.
- However, the Court of Appeals erred when it ruled that
Tanchico is entitled to disability benefits of 18 days for every
year of service. The Court of Appeals ruled that Tanchicos
employment was continuous and that his tenure with
petitioners was for 14 years. Again, the Court of Appeals
assumed that Tanchico was a regular employee. The Court of

Labor Law 1
Appeals failed to consider that Tanchicos employment
terminated with the end of each contract.
- Indications that Tanchico was suffering from ischemia were
detected on 8 December 1992 during Tanchicos vacation
period. Thus, petitioners paid him disability benefits for 18 days
in accordance with the Contract. Tanchico cannot claim that he
only acquired the illness during his last deployment since the
Medical Report26 he submitted to the NLRC showed that he has
been hypertensive since 1983 and diabetic since 1987. In the
absence of concrete proof that Tanchico acquired his disability
during his last deployment and not during his vacation, he is
only entitled to disability benefits for 18 days.

SKIPPERS UNITED PACIFIC INC V NLRC (CA &


ROSAROSO)
494 SCRA 66
AUSTRIA-MARTINEZ; July 12, 2006
NATURE
Appeal from a decision of the CA
FACTS
- Private respondent Gervasio Rosaroso was employed as a
Third Engineer with Nicolakis Shipping, S.A., a foreign firm
through its recruitment and manning agency, petitioner
Skippers. The employment contract was for the period of one
year beginning July 10, 1997 with a salary of $800 per month
and other benefits. Rosaroso boarded M/V Naval Gent on July
15, 1997. He was however ordered to disembark in Bulgaria on
August 7, 1997 and repatriated to the Philippines.
- Soon after arrival in Manila, respondent filed a complaint for
illegal dismissal and monetary claims. The Labor Arbiter found
the respondent was in fact illegally dismissed and issued an
order directing petitioner, Skippers, to pay Rosaroso separation
pay of $2,4000 or the equivalent of P100,000, representing
three months pay and unpaid salary for seven days of $186.69
or the equivalent of P7,840.98. Atorneys fees of P5,000 was
also awarded. The NLRC and the CA affirmed en toto the ruling
of the Arbiter.
- Hence this appeal to the SC.
ISSUE
WON private respondent Rosaroso was illegal dismissed
HELD
YES
- The employer of Rosaroso did not provide the quantum of
evidence needed to prove that dismissal was in fact for cause.
The evidence presented was just a telefax coming from the
alleged Chief Engineer of the vessel which the Arbiter up to the
CA considered as mere hearsay. While the Master of the vessel
was grated under Paragraph D of Section 17 of the Philippine
Overseas
employment
Administration
(POEA) Standard
Employment condition governing the employment of Filipino
Seafarers on Board Ocean Going Vessels the power to dismiss
for just cause without furnishing the seafarer with a notice of
dismissal if doing so will prejudice the safety of the crew and
the vessel, the SC noted that the complete report on the
circumstances of the dismissal was not forwarded to the
manning agency as called for under the same provision.
Minor issues
- The award of backwages and separation pay in lieu of
reinstatement as provided for in Article 279 of the Labor Code is
not applicable in this case. The Seafarer is a contractual
employee whose rights and obligations are governed by the
POEA Employment Contract and by RA 8042 (1995). The
Employment contract does not provide for the award of
separation or termination pay. However, under Section 10 of RA
8042 the award of money claims in cases of illegal dismissal is
allowed. Under this provision, an illegal dismissed seafarer is

A2010

- 94 -

Disini

entitled to indemnity equivalent to his salary for the unexpired


term of his employment contract or three months for every year
of the unexpired term, whichever is less.
- The award by the Arbiter of the peso equivalent of the dollar
awards cannot be enforced as the same is contrary to law. The
peso equivalent must be computed at the exchange rate
computed at the time of payment as provided for by RA 8183.
Disposition The questioned decision is affirmed with the
modification that the dollar award should be payable in its peso
equivalent computed at the prevailing rate of exchange at the
time of payment.

PENTAGON INTERNATIONAL SHIPPING INC V


ADELANTAR
435 SCRA 342
YNARES-SANTIAGO; July 27, 2001
NATURE
Petition for review on certiorari of CA decision which modified
an NLRC decision
FACTS
- August 16, 1997 > William B. Adelantar was hired by Dubai
Ports Authority of Jebel Ali under an employment contract (first
contract) which provided for an unlimited period of employment
with a monthly salary of Dhs 5,500.
- September 3, 1997 > Adelantar and Pentagon International
Shipping, Inc, for and in behalf of Dubai Ports Authority of Jebel
Ali, entered into a Philippine Overseas Employment
Administration (POEA) standard employment contract (second
contract), this time providing for a 12-month period with basic
monthly salary of US$380.00 and fixed overtime pay of
US$152.00.
- April 5, 1998 > Adelantars basic salary was increased to Dhs
5,890 and overtime pay was increased to Dhs 2,356
- June 11, 1998 > Dubai Ports barred Adelantar from entering
the port due to a previous dispute with his superior. On the
same date, he was given a letter as he was terminated for
assaulting his superior officer, although he was promised
employment in another company.
- Adelantar filed a complaint for illegal dismissal with money
claim against Pentagon with the NLRC
- LABOR ARBITER: found the dismissal of Adelantar was illegal
and ordered Pentagon to pay Adelantar the amount of Dhs
24,738.00 representing the latters three (3) months basic
salary inclusive of overtime pay
- NLRC: affirmed the Labor Arbiters decision and held that in
Section 10 of RA8042 (Migrant Workers and Overseas Filipinos
Act of 1995) an illegally dismissed contract worker is entitled to
the salaries corresponding to the unexpired portion of his
contract, or for three (3) months for every year of the unexpired
term, whichever is less. They awarded backwages to Adelantar
equivalent to three (3) months of his basic salary, but exclusive
of overtime pay
- CA: September 26, 2002, CA modified the amounts awarded
by the Labor Arbiter and the NLRC and instead awarded full
backwages computed from the time of the dismissal up to the
finality of the decision because Section 10 of R.A. No 8042 is
not applicable because said provision only contemplates a fixed
period of employment and that A279 LC should apply
considering that Adelantars first contract provided for an
unlimited period of employment.
ISSUE
WON A279 LC should apply given the first contract provided for
an unlimited period of employment
HELD
NO. Sec 10 RA 8042 should apply because the second contract
(with POEA), which provided for a fixed period of 1 year as

Labor Law 1
employment, is applicable at bar. Also, landmark case of
Millares v NLRC applies
Ratio It is clear that seafarers are considered contractual
employees. They can not be considered as regular employees
under A280 LC. Their employment is governed by the contracts
they sign every time they are rehired and their employment is
terminated when the contract expires. Their employment is
contractually fixed for a certain period of time. They fall under
the exception of A280 LC whose employment has been fixed for
a specific project or undertaking the completion or termination
of which has been determined at the time of engagement of the
employee or where the work or services to be performed is
seasonal in nature and the employment is for the duration of
the season. (Millares v NLRC)
Reasoning
- Coyoca v NLRC: Filipino seamen are governed by the Rules
and Regulations of the POEA. The Standard Employment
Contract governing the Employment of All Filipino Seamen on
Board Ocean-Going Vessels of the POEA, particularly in Part I,
Sec. C specifically provides that the contract of seamen shall be
for a fixed period. In no case should the contract of seamen be
longer than 12 months and any extension of the Contract period
shall be subject to the mutual consent of the parties.
- It should be stressed that whatever status of employment or
increased benefits that the complainant may have gained while
under the employ of Dubai Ports Authority, the undisputed fact
remains that prior to his deployment, he agreed to be hired
under a 12-month POEA contract, the duration of which is the
basis for the determination of the extent of the respondents
liability.
- Moreover, it is an accepted maritime industry practice that
employment of seafarers is for a fixed period only. Constrained
by the nature of their employment which is quite peculiar and
unique in itself, it is for the mutual interest of both the seafarer
and the employer why the employment status must be
contractual only or for a certain period of time. Seafarers spend
most of their time at sea and understandably, they can not stay
for a long and an indefinite period of time at sea. Limited
access to shore society during the employment will have an
adverse impact on the seafarer. The national, cultural and
lingual diversity among the crew during the COE is a reality that
necessitates the limitation of its period.
Disposition Petition is partly GRANTED and CA decision is
REVERSED and SET ASIDE. Petitioner Pentagon International
Shipping, Inc. is ORDERED to pay private respondent William B.
Adelantar the amount equivalent to the unexpired portion of the
September 3, 1997 POEA Standard Contract of Employment
plus ten percent (10%) of the award as attorneys fees.

LOPEZ V MWSS
[PAGE 60]
AUDION ELECTRIC CO INC V NLRC (MADOLID)
308 SCRA 340
GONZAGA-REYES; June 17, 1999
NATURE
Petition for certiorari, seeking annulment of resolution of the
NLRC (of which the presiding officer was our very own Dean
Carale )
FACTS
- Madolid was employed by Audion Electric Co. on June 30, 1976
as fabricator and continuously rendered service in different
offices and projects as helper technician, stockman, and
timekeeper. He rendered 13 years of service with a clean
record. On August 3, Madolid received a letter informing him
that he will be considered terminated after the turnover of
materials, including companys tools and equipments not later
than August 15, 1989.

A2010

- 95 -

Disini

- Madolid claims that he was dismissed without justifiable cause


and due process and that his dismissal was done in bad faith
which renders the dismissal illegal. For this reason, he claims
that he is entitled to reinstatement with full backwages, and
moral and exemplary damages. He also includes payment of his
overtime pay, project allowance, minimum wage increase
adjustment, proportionate 13th month pay and attorney's fees.
Audion rebuts his allegations by saying that the employment
contract of Madolid was one that was co-terminus with the
project, thus he should not be considered as a regular
employee. Also, the company contends that it had paid all the
alleged unpaid wages.
- The Labor arbiter decided the case in favor of Madolid,
ordering Audion to pay him backwages, OT pay, project
allowances, min. wage increase adjustment, 13th month pay,
and awarding him moral and exemplary damages and
attorneys fees. Appeal to NLRC was dismissed.
ISSUES
1. WON Madolid was a regular employee, thus entitling him to
backwages, etc.
2. WON Audion was denied due process with the award of all
the claims of Madolid
HELD
1. YES
Ratio
Where the employment of project employees is
extended long after the supposed project has been finished, the
employees are removed from the scope of project employees
and considered regular employees.
Reasoning
- (citing NLRCs decision): Audions assigning Madolid to its
various projects did not make him a project worker. As found by
the Labor Arbiter, it appears that complainant was employed
by respondent xxx as fabricator and or projects as helper
electrician, stockman and timekeeper.' Simply put, complainant
was a regular non-project worker.
- Madolids employment status was established by the
Certification of Employment dated April 10, 1989 issued by
Audion which certified that private respondent is a bonafide
employee from June 30, 1976 up to the time of issuance on
April 10, 1989. This showed that his exposure to their field of
operation
was
as
fabricator,
helper/electrician,
stockman/timekeeper. This proves that he was regularly and
continuously employed by Audion in various job assignments
from 1976 to 1989, for a total of 13 years. The alleged gap in
employment service does not defeat his regular status as he
was rehired for many more projects without interruption and
performed functions which are vital, necessary and
indispensable to the usual business of petitioner.
- Audion could have presented substantial evidence to support
its claim that Madolid was a project worker, like the
employment contract (which stated the employees nature of
employment) or reports of termination (which were required by
DOLE upon termination of the project, and failure to submit this
is an indication of regular status of an employee as held in
cases), but it did not.
2. NO
Ratio Due process is not denied when one is afforded the
opportunity to be heard and present his case, but the same
decided not to take the opportunity.
Reasoning
- Madolid clearly specified in his affidavit the specific dates in
which he was not paid overtime pay, project allowances, 13th
month pay, and wage adjustments. The claim of Audion that it
paid him such must be proved by evidence, which it did not do
(despite of having the burden to prove the claim).
- In fact, records show that the company did not appear in
hearings, which the court took to be a waiver of its right to be
heard.
- However, award to moral and exemplary damages and
attorneys fees are deleted for being devoid of moral basis.

Labor Law 1
Disposition
Petition denied, resolutions affirmed with
modifications (deletion of award of damages and attorneys
fees)

BETA ELECTRIC CORP V NLRC (BETA ELECTRIC


EMPLOYEES ASSOCIATION, PETILLA)
182 SCRA 384
SARMIENTO; February 15, 1990
NATURE
Petition to review the decision of the National Labor Relations
Commission affirming the judgment of the labor arbiter
reinstating the private respondent with backwages.
FACTS
- The petitioner hired the private respondent as clerk typist for
one month, which appointment was extended five times in five
months (one month /contract).Her appointments were covered
by corresponding written contracts. On June 22, 1987, her
services were terminated without notice or investigation. On the
same day, she went to the labor arbiter on a complaint for
illegal dismissal. Both the labor arbiter and the respondent
National Labor Relations Commission ruled for her.Petioner
claims the private respondents appointment was temporary
and hence she may be terminated at will.
ISSUES
WON the fact that private respondents employment has been a
contract-to-contract basis alters the character of her
employment as a regular employee
HELD
NO
Ratio. The fact that her employment has been a contract-tocontract basis can not alter the character of employment,
because contracts can not override the mandate of law..
Reasoning
- private employee was employed from December 15, 1986
until June 22, 1987 when she was ordered laid off. Her tenure
having exceeded six months, she attained regular employment.
- petitioner can not rightfully say that since the private
respondent's employment hinged from contract to contract, it
was ergo, "temporary", depending on the term of each
agreement. Under the Labor Code, an employment may only be
said to be "temporary" "where [it] has been fixed for a specific
undertaking the completion of or termination of which has been
determined at the time of the engagement of the employee or
where the work or services to be performed is seasonal in
nature and the employment is for the duration of the season."
Quite to the contrary, the private respondent's work, that of
"typist-clerk" is far from being "specific" or "seasonal", but
rather, one, according to the Code, "where the employee has
been engaged to perform activities which are usually necessary
or desirable in the usual business." And under the Code, where
one performs such activities, he is a regular employee, "[t]he
provisions
of
written
agreement
to
the
contrary
notwithstanding.
Disposition Petition DISMISSED. Private respondent is ordered
REINSTATED with backwages equivalent to three years with no
qualification or deductions.

UNIVERSAL ROBINA CORPORATION V CATAPANG


473 SCRA 189
CALLEJO, SR; October 14, 2005
FACTS
- Petitioner Universal Robina Corporation is a corporation duly
organized and existing under the Philippine laws, while
petitioner Randy Gregorio is the manager of the petitioner
companys duck farm in Calauan, Laguna.

A2010

- 96 -

Disini

- The individual respondents were hired by the petitioner


company on various dates from 1991 to 1993 to work at its
duck farm in Barangay Sto. Tomas, Calauan, Laguna. The
respondents were hired under an employment contract which
provided for a five-month period. After the expiration of the said
employment contracts, the petitioner company would renew
them and re-employ the respondents. This practice continued
until sometime in 1996, when the petitioners informed the
respondents that they were no longer renewing their
employment contracts.
- In October 1996, the respondents filed separate complaints for
illegal dismissal, reinstatement, backwages, damages and
attorneys fees against the petitioners. The complaints were
later consolidated. On March 30, 1999, after due proceedings,
the Labor Arbiter rendered a decision in favor of the
respondents, which NLRC and the CA affirmed.
- On appeal, the petitioners submit that the respondents are not
regular employees. They aver that it is of no moment that the
respondents have rendered service for more than a year since
they were covered by the five-month individual contracts to
which they duly acquiesced. The petitioners contend that they
were free to terminate the services of the respondents at the
expiration of
their individual contracts.
The petitioners
maintain that, in doing so, they merely implemented the terms
of the contracts.
- The petitioners assert that the respondents contracts of
employment were not intended to circumvent security of
tenure. They point out that the respondents knowingly and
voluntarily agreed to sign the contracts without the petitioners
having exercised any undue advantage over them. Moreover,
there is no evidence showing that the petitioners exerted moral
dominance on the respondents.[\
ISSUE
WON the respondent employees of the corporation are regular
employees and therefore their termination for causes outside of
the Labor Code is patently illegal
HELD
YES
Ratio An employee shall be deemed to be of regular status
when he has been performing a job for at least one year even if
the performance is not continuous and merely intermittent.
Reasoning
- In any case, we find that the CA, the NLRC and the Labor
Arbiter correctly categorized the respondents as regular
employees of the petitioner company. In Abasolo v. National
Labor Relations Commission, the Court reiterated the test in
determining whether one is a regular employee:
- The primary standard, therefore, of determining
regular employment is the reasonable connection
between the particular activity performed by the
employee in relation to the usual trade or business of
the employer. The test is whether the former is usually
necessary or desirable in the usual business or trade of
the employer. The connection can be determined by
considering the nature of work performed and its
relation to the scheme of the particular business or
trade in its entirety. Also, if the employee has been
performing the job for at least a year, even if the
performance is not continuous and merely intermittent,
the law deems repeated and continuing need for its
performance as sufficient evidence of the necessity if
not indispensability of that activity to the business.
Hence, the employment is considered regular, but only
with respect to such activity and while such activity
exists.
- It is obvious that the said five-month contract of employment
was used by petitioners as a convenient subterfuge to prevent
private respondents from becoming regular employees. Such
contractual arrangement should be struck down or disregarded
as contrary to public policy or morals. To uphold the same

Labor Law 1
would, in effect, permit petitioners to avoid hiring permanent or
regular employees by simply hiring them on a temporary or
casual basis, thereby violating the employees security of
tenure in their jobs. Petitioners act of repeatedly and
continuously hiring private respondents in a span of 3 to 5
years to do the same kind of work negates their contention that
private respondents were hired for a specific project or
undertaking only.
- Further, factual findings of labor officials who are deemed to
have acquired expertise in matters within their respective
jurisdiction are generally accorded not only respect but even
finality, and bind us when supported by substantial evidence.
Disposition petition is DENIED DUE COURSE. The Decision of
the Court of Appeals is AFFIRMED.

MARAGUINOT V NLRC (DEL ROSARIO, VIVA FILMS)


284 SCRA 539
DAVIDE; January 22, 1998
NATURE
Special civil action for certiorari seeking to annul the decision of
NLRC and its Resolution
FACTS
- Petitioner Alejandro Maraguinot, Jr. maintains that he was
employed by private respondents as part of the filming crew.
About 4 months later, he was designated Asst. Electrician. He
was then promoted to the rank of Electrician.
- Petitioner Paulino Enero claims that private respondents
employed him as a member of the shooting crew.
- Petitioners tasks consisted of loading, unloading and
arranging movie equipment in the shooting area as instructed
by the cameraman, returning the equipment to Viva Films
warehouse, assisting in the fixing of the lighting system, and
performing other tasks that the cameraman and/or director may
assign.
- Petitioners requested that private respondents adjust their
salary in accordance with the minimum wage law. Petitioners
were informed that Mr. Vic del Rosario would agree to increase
their salary only if they signed a blank employment contract.
As petitioners refused to sign, private respondents forced Enero
to go on leave then refused to take him back when he reported
for work. Meanwhile, Maraguinot was dropped from the
company payroll but was returned and again asked to sign a
blank employment contract, and when he still refused, private
respondents terminated his services. Petitioners thus sued for
illegal dismissal before the Labor Arbiter.
- Private respondents claim that Viva Films is primarily
engaged in the distribution and exhibition of movies, but not in
the business of making movies; in the same vein, private
respondent Vic del Rosario is merely an executive producer,
i.e., the financier who invests a certain sum of money for the
production of movies distributed and exhibited by VIVA; that
they contract persons called producers -- also referred to as
associate producers-- to produce or make movies for
private respondents; and that petitioners are project employees
of the associate producers who, in turn, act as independent
contractors.
As such, there is no employer-employee
relationship between petitioners and private respondents; that
it was the associate producer of a film who hired
Maraguinot.and he was released upon payment of his last
salary, as his services were no longer needed; that Enero was
hired for a movie, went on vacation and by the time he reported
back to work the move had been completed.
- The Labor Arbiter found that:
-- complainants are the employees of the respondents. The
producer cannot be considered as an independent contractor
but should be considered only as a labor-only contractor and as
such, acts as a mere agent of the real employer, the herein
respondents. Also, it is an admitted fact that the complainants
received their salaries from the respondents. It is very clear

A2010

- 97 -

Disini

also that complainants are doing activities which are necessary


and essential to the business of the respondents, that of moviemaking. Complainant Maraguinot worked as an electrician while
complainant Enero worked as a crew [member]. Hence, the
complainants were illegally dismissed.
- Private respondents appealed to the NLRC. In its decision, it
said that:
1. Complainants were hired for specific movie projects and their
employment was co-terminus with each movie project the
completion/termination of which are pre-determined, such fact
being made known to complainants at the time of their
engagement.
2. Each shooting unit works on one movie project at a time. And
the work of the shooting units, which work independently from
each other, are not continuous in nature but depends on the
availability of movie projects.
3. Further shown by respondents is the irregular work schedule
of complainants on a daily basis. Maraguinot was supposed to
report on 05 August 1991 but reported only on 30 August 1991,
or a gap of 25 days. Complainant Enero worked on 10
September 1991 and his next scheduled working day was 28
September 1991, a gap of 18 days.
4. The extremely irregular working days and hours of
complainants work explain the lump sum payment for
complainants services for each movie project. Hence,
complainants were paid a standard weekly salary regardless of
the number of working days and hours they logged in.
Otherwise, if the principle of no work no pay was strictly
applied, complainants earnings for certain weeks would be very
negligible.
5. Respondents also alleged that complainants were not
prohibited from working with other movie companies.
The NLRC, in reversing the Labor Arbiter, then concluded that
these
circumstances,
taken
together,
indicated
that
complainants (herein petitioners) were project employees.
Petitioners Claim To support their claim that they were
regular (and not project) employees of private respondents,
petitioners cited their performance of activities that were
necessary or desirable in the usual trade or business of private
respondents and added that their work was continuous, i.e.,
after one project was completed they were assigned to another
project.
Respondents Private respondents reiterate their version of the
facts and stress that their evidence supports the view that
petitioners are project employees; point to petitioners irregular
work load and work schedule; emphasize the NLRCs finding
that petitioners never controverted the allegation that they
were not prohibited from working with other movie companies;
and ask that the facts be viewed in the context of the peculiar
characteristics of the movie industry.
The Office of the Solicitor General (OSG) is convinced that
this petition is improper since petitioners raise questions of fact;
and submits that petitioners reliance on Article 280 of the
Labor Code to support their contention that they should be
deemed regular employees is misplaced, as said section
merely distinguishes between two types of employees, i.e.,
regular employees and casual employees, for purposes of
determining the right of an employee to certain benefits. The
OSG likewise rejects petitioners contention that since they
were hired not for one project, but for a series of projects, they
should be deemed regular employees. In closing, the OSG
disagrees with petitioners claim that the NLRCs classification
of the movie producers as independent contractors had no basis
in fact and in law, since, on the contrary, the NLRC took pains
in explaining its basis for its decision.
ISSUES
1. WON this is a proper action
2. WON an employer-employee relationship existed between
the petitioners and private respondents or any one of them
3. WON petitioners were illegally dismissed

Labor Law 1
HELD
1. YES
Ratio We rule that a special civil action for certiorari under Rule
65 of the Rules of Court is the proper remedy for one who
complains that the NLRC acted in total disregard of evidence
material to or decisive of the controversy. In the instant case,
petitioners allege that the NLRCs conclusions have no basis in
fact and in law, hence the petition may not be dismissed on
procedural or jurisdictional grounds.
2. YES
Ratio The relationship between VIVA and its producers or
associate producers seems to be that of agency, as the latter
make movies on behalf of VIVA, whose business is to make
movies.
As such, the employment relationship between
petitioners and producers is actually one between petitioners
and VIVA, with the latter being the direct employer.
The employer-employee relationship between petitioners and
VIVA can further be established by the control test. While four
elements are usually considered in determining the existence of
an employment relationship, namely: (a) the selection and
engagement of the employee; (b) the payment of wages; (c) the
power of dismissal; and (d) the employers power to control the
employees conduct, the most important element is the
employers control of the employees conduct, not only as to
the result of the work to be done but also as to the means and
methods to accomplish the same. These four elements are
present here.
Reasoning (On job contracting)
It is settled that the contracting out of labor is allowed only in
case of job contracting.10
- Assuming that the associate producers are job contractors,
they must then be engaged in the business of making motion
pictures. As such, and to be a job contractor under the
preceding description, associate producers must have tools,
equipment, machinery, work premises, and other materials
necessary to make motion pictures.The associate producer did
not have substantial capital nor investment in the form of tools,
equipment and other materials necessary for making a movie. If
private respondents insist that their associate producers are
labor contractors, then these producers can only be labor-only
contractors.11
- As labor-only contracting is prohibited, the law considers the
person or entity engaged in the same a mere agent or
intermediary of the direct employer. But even by the preceding
standards, the associate producers of VIVA cannot be
considered labor-only contractors as they did not supply, recruit
nor hire the workers.
Reasoning (On control test)
- VIVAs control is evident in its mandate that the end result
must be a quality film acceptable to the company. The
means and methods to accomplish the result are likewise
controlled by VIVA, viz., the movie project must be finished
within schedule without exceeding the budget, and additional
expenses must be justified; certain scenes are subject to
10

Section 8, Rule VIII, Book III of the Omnibus Rules Implementing the Labor Code
describes permissible job contracting in this wise:
Sec. 8. Job contracting. -- There is job contracting permissible under the Code if the
following conditions are met:
(1) The contractor carries on an independent business and undertakes the contract
work on his own account under his own responsibility according to his own manner
and method, free from the control and direction of his employer or principal in all
matters connected with the performance of the work except as to the results thereof;
and
(2) The contractor has substantial capital or investment in the form of tools,
equipment, machineries, work premises, and other materials which are necessary in
the conduct of his business.

11

Art. 106. Contractor or subcontractor.-- x x x There is labor-only


contracting where the person supplying workers to an employer does not
have substantial capital or investment in the form of tools, equipment,
machineries, work premises, among others, and the workers recruited and
placed by such persons are performing activities which are directly related to
the principal business of such employer. In such cases, the person or
intermediary shall be considered merely as an agent of the employer who
shall be responsible to the workers in the same manner and extent as if the
latter were directly employed by him.

A2010

- 98 -

Disini

change to suit the taste of the company; and the Supervising


Producer, the eyes and ears of VIVA and del Rosario,
intervenes in the movie-making process by assisting the
associate producer in solving problems encountered in making
the film.
- Aside from control, the element of selection and engagement
is likewise present in the instant case and exercised by VIVA. A
sample appointment slip was offered by private respondents to
prove that members of the shooting crew except the driver are
project employees of the Independent Producers. Notably,
nowhere in the appointment slip does it appear that it was the
producer or associate producer who hired the crew members;
moreover, it is VIVAs corporate name which appears on the
heading of the appointment slip. What likewise tells against
VIVA is that it paid petitioners salaries as evidenced by
vouchers, containing VIVAs letterhead, for that purpose.
3. YES
Ratio A project employee or a member of a work pool may
acquire the status of a regular employee when the following
concur: 1) There is a continuous rehiring of project employees
even after cessation of a project; and 2) The tasks performed by
the alleged project employee are vital, necessary and
indispensable to the usual business or trade of the employer.
However, the length of time during which the employee was
continuously re-hired is not controlling, but merely serves as a
badge of regular employment.
- In the instant case, the evidence on record shows that
petitioner Enero was employed for a total of two (2) years and
engaged in at least eighteen (18) projects, while petitioner
Maraguinot was employed for some three (3) years and worked
on at least twenty-three (23) projects. Moreover, as petitioners
tasks involved, among other chores, the loading, unloading and
arranging of movie equipment in the shooting area as
instructed by the cameramen, returning the equipment to the
Viva Films warehouse, and assisting in the fixing of the
lighting system, it may not be gainsaid that these tasks were
vital, necessary and indispensable to the usual business or
trade of the employer. As regards the underscored phrase, it
has been held that this is ascertained by considering the nature
of the work performed and its relation to the scheme of the
particular business or trade in its entirety.
Reasoning
- It may not be ignored, however, that private respondents
expressly admitted that petitioners were part of a work pool;
and, while petitioners were initially hired possibly as project
employees, they had attained the status of regular employees
in view of VIVAs conduct.
- At this time, we wish to allay any fears that this decision
unduly burdens an employer by imposing a duty to re-hire a
project employee even after completion of the project for which
he was hired. The import of this decision is not to impose a
positive and sweeping obligation upon the employer to re-hire
project employees. What this decision merely accomplishes is a
judicial recognition of the employment status of a project or
work pool employee in accordance with what is fait accompli,
i.e., the continuous re-hiring by the employer of project or work
pool employees who perform tasks necessary or desirable to
the employers usual business or trade. Let it not be said that
this decision coddles labor, for as Lao12 has ruled, project or
work pool employees who have gained the status of regular
employees are subject to the no work-no pay principle.
- The Courts ruling here is meant precisely to give life to the
constitutional policy of strengthening the labor sector, but, we
12

A work pool may exist although the workers in the pool do not receive
salaries and are free to seek other employment during temporary breaks in
the business, provided that the worker shall be available when called to report
for a project. Although primarily applicable to regular seasonal workers, this
set-up can likewise be applied to project workers insofar as the effect of
temporary cessation of work is concerned. This is beneficial to both the
employer and employee for it prevents the unjust situation of coddling labor
at the expense of capital and at the same time enables the workers to attain
the status of regular employees.

Labor Law 1
stress, not at the expense of management.
Lest it be
misunderstood, this ruling does not mean that simply because
an employee is a project or work pool employee even outside
the construction industry, he is deemed, ipso jure, a regular
employee. All that we hold today is that once a project or work
pool employee has been: (1) continuously, as opposed to
intermittently, re-hired by the same employer for the same
tasks or nature of tasks; and (2) these tasks are vital, necessary
and indispensable to the usual business or trade of the
employer, then the employee must be deemed a regular
employee, pursuant to Article 280 of the Labor Code and
jurisprudence.
Disposition instant petition is GRANTED.

ABESCO CONSTRUCTION AND DEVELOPMENT


CORPORATION V RAMIREZ
487 SCRA 9
CORONA; April 10, 2006
NATURE
Appeal by certiorari
FACTS
- Petitioner company was engaged in a construction business
where respondents were hired on different dates from 1976 to
1992 either as laborers, road roller operators, painters or
drivers.
- In 1997, respondents filed 2 separate complaints for illegal
dismissal against the company and its General Manager before
the Labor Arbiter (LA). Petitioners allegedly dismissed them
without a valid reason and without due process of law. The
complaints also included claims for non-payment of the 13th
month pay, five days service incentive leave pay, premium pay
for holidays and rest days, and moral and exemplary damages.
The LA later ordered the consolidation of the two complaints.
- Petitioners denied liability and countered that respondents
were project employees since their services were necessary
only when the company had projects to be completed.
Petitioners argued that, being project employees, respondents
employment was coterminous with the proj. to which they were
assigned. They werent regular employees who enjoyed security
of tenure and entitlement to separation pay upon termination
from work.
- After trial, the LA declared respondents as regular employees
because they belonged to a work pool from which the
company drew workers for assignment to different projects, at
its discretion. He ruled that respondents were hired and re-hired
over a period of 18 years, hence, they were deemed to be
regular employees. He likewise found that their employment
was terminated without just cause. Thus, in its judgment, the LA
declared petitioner company guilty of illegal dismissal and
ordered it to reinstate the respondents to their former positions
with backwages and other benefits and that if reinstatement
was not feasible, that separation pay be awarded.
- Petitioners appealed to the NLRC which affirmed the LAs
decision. They later filed a petition for review in the CA arguing
that they were not liable for illegal dismissal since respondents
services were merely put on hold until the resumption of their
business operations. They also averred that they had paid
respondents their full wages and benefits as provided by law,
hence, the latter had no more right to further benefits.
- The CA, taking note of the fact that petitioners previously used
the defense that the respondents were project employees who
were not entitled to security of tenure and now say that the
respondents were not dismissed but their employment merely
suspended, dismissed the appeal and dismissed the MFR as
well.
ISSUES
1. WON the respondents were regular employees
2. WON respondents were illegally dismissed

A2010

- 99 -

Disini

HELD
1. YES
Ratio In determining the nature of ones employment, length of
service is not a controlling factor
Reasoning
- Jurisprudence: The SC ruled that respondents were regular
employees but not for the reasons given by the LA (which both
the NLRC and the CA affirmed). Citing Palomar, et al. v. NLRC,
the SC held that contrary to the disquisitions of the LA,
employees (like respondents) who work under different project
employment contracts for several years do not automatically
become regular employees; they can remain as project
employees regardless of the number of years they work. Length
of time is not a controlling factor in determining the nature of
ones employment.
- Moreover, employees who are members of a work pool from
which a company (like petitioner corp.) draws workers for
deployment to its different projects do not become regular
employees by reason of that fact alone. The Court has
enunciated in the cases of Raycor Aircontrol Systems, Inc. v.
NLRC, and ALU-TUCP v. NLRC, that members of a work pool
can either be project employees or regular employees
- The principal test for determining whether employees are
project employees or regular employees is whether they
are assigned to carry out a specific project or undertaking, the
duration and scope of which are specified at the time they are
engaged for that project. Such duration, as well as the
particular work/service to be performed, is defined in an
employment agreement and is made clear to the employees at
the time of hiring.
- Petitioners did not have that kind of agreement with
respondents. Neither did they inform the respondents of the
nature of their work at the time of hiring. Hence, for failure of
petitioners to substantiate their claim that respondents were
project employees, we are constrained to declare them as
regular employees
- Furthermore, petitioners cannot belatedly argue that
respondents continue to be their employees (so as to escape
liability for illegal dismissal). Before the LA, petitioners
staunchly postured that respondents were only project
employees whose employment tenure was coterminous with
the projects they were assigned to. However, before the CA,
they took a different stance by insisting that respondents
continued to be their employees. Petitioners inconsistent and
conflicting positions on their true relation with respondents
make it all the more evident that the latter were indeed their
regular employees.
2. YES
Ratio The law requires that the employer furnish the employee
2 written notices: (1) a notice informing them of the particular
acts for which they are being dismissed and (2) a notice
advising them of the decision to terminate the employment,
before termination can be validly effected.
Reasoning
- In resolving the issue of illegal dismissal, the SC simply stated
that petitioners failed to adhere to the two-notice rule, and
said that respondents were never given such notices.
Disposition Petition denied

ALU-TUCP V NLRC (NATIONAL STEEL CORP)


234 SCRA 678
FELICIANO; August 2, 1994
NATURE
Petition for Certiorari to review the resolutions of the NLRC
FACTS
- Petitioners claim that they have been employed by respondent
National Steel Corporation (NSC) in connection with its Five Year

Labor Law 1
Expansion Program (FAYEP I and II) for varying lengths of time
when they were separated from NSCs service.
- Petitioners filed separate complaints for unfair labor practice,
regularization and monetary benefits. The Labor Arbiter
declared petitioners regular project employees who shall
continue their employment as such for as long as such (project)
activity exists, but entitled to the salary of a regular employee
pursuant to the provisions in the collective bargaining
agreement. It also ordered payment of salary differentials.
- Both parties appealed. Petitioners argued they were regular,
not project employees. NSC claimed petitioners are project
employees as they were employed to undertake a specific
project.
- The NLRC modified the Labor Arbiters decision, affirming the
holding that they were project employees since they were hired
to perform work in a specific undertaking. It, however, set aside
the award to petitioners of the same benefits enjoyed by
regular employees for lack of legal and factual basis.
- Petitioners appealed to the SC, arguing that they are regular
employees of NSC because: (i) their jobs are necessary,
desirable and work-related to private respondents main
business, steel-making; and (ii) they have rendered service for
six (6) or more years to NSC.
ISSUE
WON petitioners are properly characterized as project
employees rather than regular employees of NSC
SCs NOTE: The issue relates to an important consequence: the
services of project employees are co-terminous with the project
and may be terminated upon the end or completion of the
project for which they were hired. Regular employees, in
contrast, are legally entitled to remain in the service of their
employer until that service is terminated by one or another of
the recognized modes of termination of service under the Labor
Code.
HELD
YES
petitioners are project employees.
- The law governing the matter is Article 280 of the Labor Code:
ART. 280. Regular and casual employment. - The
provisions of written agreement to the contrary
notwithstanding and regardless of the oral agreement of
the parties, an employment shall be deemed to be
regular where the employee has been engaged to
perform activities which are usually necessary or
desirable in the usual business or trade of the employer,
except where the employment has been fixed for a
specific project or undertaking the completion or
termination of which has been determined at the time of
the engagement of the employee or where the work or
service to be performed is seasonal in nature and the
employment is for the duration of the season.
An employment shall be deemed to be casual if it is not
covered by the preceding paragraph: Provided, That any
employee who has rendered at least one year of service,
whether such service is continuous or broken, shall be
considered a regular employee with respect to the
activity in which he is employed and his employment
shall continue while such activity exists.
- As evident in Article 280 of the Labor Code, the principal test
for determining whether particular employees are properly
characterized as project employees as distinguished from
regular employees is whether or not the project employees
were assigned to carry out a specific project or undertaking,
the duration (and scope) of which were specified at the time the
employees were engaged for that project.
- In business and industry, project could refer to one or the
other of at least two distinguishable types of activities. Firstly, a
project could refer to a particular job or undertaking that is
within the regular or usual business of the employer company,
but which is distinct and separate, and identifiable as such,

A2010

- 100 -

Disini

from the other undertakings of the company. Such job or


undertaking begins and ends at determined or determinable
times. Secondly, the term project could also refer to a
particular job or undertaking that is not within the regular
business of the corporation. Such job or undertaking must also
be identifiably separate and distinct from the ordinary or
regular business operations of the employer. The job or
undertaking also begins and ends at determined or
determinable times.
- Whichever type of project employment is found in a particular
case, a common basic requisite is that the designation of
named employees as project employees and their assignment
to a specific project, are effected and implemented in good
faith, and not merely as a means of evading otherwise
applicable requirements of labor laws.
- The particular component projects embraced in the FAYEP, to
which petitioners were assigned, were distinguishable from the
regular or ordinary business of NSC, which is the production or
making and marketing of steel products. During the time
petitioners rendered services to NSC, their work was limited to
one or another of the specific component projects which made
up the FAYEP I and II. It is not shown that petitioners were hired
for or assigned to other purposes.
Re Length of Service
- SC affirmed the Labor Arbiter and NLRCs basic finding that
the length of service of a p[roject employee is not the
controlling test of employment tenure but whether or not the
employment has been fixed for a specific project or undertaking
the completion or termination of which has been determined at
the time of the engagement of the employee.
- The simple fact that the employment of petitioners as project
employees had gone beyond one year does not detract from, or
legally dissolve, their status as project employees.
Disposition Petition for Certiorari is dismissed. Resolutions of
NLRC affirmed.

KIAMCO V NLRC (PNOC)


309 SCRA 424
BELLOSILLO; June 29, 1999
FACTS
- Private respondent PHILIPPINE NATIONAL OIL COMPANY
(PNOC) through its Energy Research and Development Division,
hired petitioner Cisell Kiamco as a project employee in its
Geothermal Agro-Industrial Plant Project in Valencia, Negros
Oriental. The Contract of Employment1 stipulated among others
that Kiamco was being hired by the company as a technician for
a period of 5 months from July 1 1992 to Nov 30 1992, or up to
the completion of the project, whichever would come first.
- After the termination of the contract, a 2nd one was entered
into by the parties containing basically the same terms and
conditions. The period of employment was from Dec 1 1992 to
April 30 1993.
- Kiamco was again re-hired for 6 months (May 1 1993 to Nov
30 1993)
- On Oct 20 1993 Kiamco received a Memorandum from the
administration department demanding an explanation from him
on certain infractions he allegedly committed: 1. Misconduct 2.
Absence without official leave (AWOL) 3. Non-compliance of
administrative
reporting
procedure
on
accidents
4.
Unauthorized use of company vehicles
- Kiamco tried to explain his side but private respondents found
his explanation unsatisfactory. On Oct 28 1993 Kiamco received
a Memorandum placing him under preventive suspension from
Nov 1 1993 to Nov 30 1993 pending further investigation. No
investigation however was ever conducted. Private respondents
contended that an investigation was not necessary since
Kiamco had ceased to be an employee ipso facto upon the
expiration of his employment contract on Nov 30 1993.
- On Dec 1 1993 Kiamco reported back to work but was
prevented by security guards from entering the company
premises. On May 27 1994 private respondent reported to the

Labor Law 1
Department of Labor and Employment that petitioner Kiamco
was terminated on Nov 1 1993 due to the expiration of his
employment contract and the abolition of his position.
- On April 25 1994 Kiamco filed before the NLRC Sub-Regional
Arbitration Branch No. VII a Complaint for illegal suspension and
dismissal against the PNOC. He prayed that he be reinstated to
his former position and paid back wages. Labor Arbiter
dismissed the complaint for lack of merit. According to the
Labor Arbiter, the three (3) employment contracts were freely
and
voluntarily
signed
by
Kiamco
and
the
PNOC
representatives. The contracts plainly stated that Kiamco was
being hired for a specific project and for a fixed term. Therefore
Kiamco could not question his dismissal since it was in
accordance with his employment contract.
- Kiamco appealed the decision of the Labor Arbiter to public
respondent NLRC which on Sept 27 1996 reversed the Labor
Arbiter and declared Kiamco as a regular employee of the
respondents and to have been illegally dismissed by the latter.
Ordering respondents to REINSTATE the complainant to his
former position without loss of seniority rights and privileges
with back wages from the date of his dismissal up to actual
reinstatement less any income he may have earned during the
pendency of the case.
- Private respondents filed a MFR of the decision of the NLRC
contending that it erred in holding that Kiamco was a regular
employee and that the findings of the Labor Arbiter that Kiamco
was a project employee should be affirmed.
- NLRC modified its Sept 27 1996 Decision declaring that the
complainant-appellant is declared a project employee at
respondents Geothermal Plant and to continue with said
employment until the full completion of the project but in the
absence of proof to that effect, complainant is hereby awarded
back wages for a period of 6 months or in the amount of
P23,100.00. The order declaring the complainant-appellant as a
regular employee of respondent PNOC, and for said company to
reinstate the complainant with full back wages is hereby
deleted.
- In his petition for certiorari, Kiamco charges the NLRC with
grave abuse of discretion amounting to lack or excess of
jurisdiction in issuing the questioned Resolution and prays that
it be nullified and he reinstated to his former position. He also
seeks payment of back wages, damages and attorneys fees.
ISSUES
1. WON petitioner is a regular employee or a project employee
2. WON petitioner is entitled to reinstatement without loss of
seniority rights and privileges and to the payment of full back
wages
3.
WON petitioner is entitled to moral and exemplary
damages.
HELD
1. Kiamco was correctly labeled by the NLRC as a project
employee.
-Article 280 of the Labor Code
Regular and casual employment. - The provisions of written
agreement to the contrary notwithstanding and regardless of
the oral agreement of the parties, an employment shall be
deemed to be regular where the employee has been engaged
to perform activities which are usually necessary or desirable
in the usual business or trade of the employer, except where
the employment has been fixed - for a specific project or
undertaking the completion or termination of which has been
determined at the time of the engagement of the employee
or where the work or service to be performed is seasonal in
nature and the employment is for the duration of the season.
- An employee shall be deemed to be casual if it is not covered
by the preceding paragraph: Provided, that any employee who
has rendered at least one year of service, whether such service
is continuous or broken, shall be considered a regular employee
with respect to the activity in which he is employed and his
employment shall continue while such activity exists.

A2010

- 101 -

Disini

- In Violeta v. NLRC [10 October 1997, 280 SCRA 520.] it was


held The principal test for determining whether particular employees
are properly characterized as "project employees," as
distinguished from "regular employees," is whether or not the
"project employees" were assigned to carry out a "specific
project or undertaking," the duration (and scope) of which were
specified at the time the employees were engaged for that
project. As defined, project employees are those workers hired
(1) for a specific project or undertaking, and (2) the completion
or termination of such project or undertaking has been
determined at the time of engagement of the employee.
- Under Policy Instruction No. 20 of the Secretary of Labor,
project employees are those employed in connection with a
particular project. Non-project or regular employees are those
employed without reference to any particular project.
- The three Contracts of Employment entered into by Kiamco
clearly established that he was a project employee because (a)
he was specifically assigned to work for a particular project,
which was the Geothermal Agro-Industrial Demonstration Plant
Project of private respondents, and (b) the termination and the
completion of the project or undertaking was determined and
stipulated in the contract at the time of his employment.
2. YES
- In Santos v. NLRC (154 SCRA 166) it was held The normal consequences of a finding that an employee has
been illegally dismissed are, that the employee becomes
entitled to reinstatement to his former position without loss of
seniority rights and the payment of back wages.
- Reinstatement restores the employee who was unjustly
dismissed to the position from which he was removed, that is,
to his status quo ante dismissal; while the grant of back wages
allows the same employee to recover from the employer that
which he had lost by way of wages as a result of his dismissal.
- The argument of private respondents that reinstatement and
payment of back wages could not be made since Kiamco was
not a regular employee is apparently misplaced. As quoted
above, the normal consequences of an illegal dismissal are the
reinstatement of the aggrieved employee and the grant of back
wages. These rights of an employee do not depend on the
status of his employment prior to his dismissal but rather to the
legality and validity of his termination. The fact that an
employee is not a regular employee does not mean that he can
be dismissed any time, even illegally, by his employer.
3. NO
- Moral damages are recoverable only where the dismissal of
the employee was attended with bad faith or fraud or
constituted an act oppressive to labor or was done in a manner
contrary to morals, good custom or public policy. Exemplary
damages, on the other hand, may be awarded only if the
dismissal was effected in a wanton, oppressive or malevolent
manner. The evidence on record does not show any fraud,
malice or bad faith on the part of private respondents that
would justify payment to petitioner of moral and exemplary
damages.

PHIL. JAI-ALAI & AMUSEMENT CORP V CLAVE


126 SCRA 299
MELENCIO-HERRERA; December 21, 1983
NATURE
Petition for Certiorari with Preliminary Injunction
FACTS
- Petitioner is a corporation operating a jai-alai fronton for sport
and amusement.
- It has its own maintenance group for the upkeep of its
premises. For the renovation of its main building, which work is
not included in maintenance, it hired private respondents,
Cadatal, Jr., a plumber, and Delgra, a mason, together with 30
other workers on February 2, 1976 for a period of one month,

Labor Law 1
open to extension should the need for the arise in the course of
the renovation.
- Renovation was completed by October 1976. Management
then decided to construct an annex to the building and private
respondents worked on the fire escape.
- November 27, 1976 Notice of termination given to the
respondents effective November 29 but they still continued to
work nonetheless. They worked until December 11 and were
fully paid for the work they rendered up to that date.
- December 13, 1976 - Petitioner filed with the former
Department of Labor a report of termination of the services of
private respondents and 30 others, listing them as casual
emergency workers.
Private workers alleged illegal
termination.
Assistant Minister Leogardo ordered the
reinstatement of the workers with full backwages before
petitioner could file a reply to the letter-complaint of the
respondents.
- Leogardo said that the respondents were already regular
employees according to Art. 170 (now Art. 281) of the CC and
that termination was unjust.
- An appeal was filed which Clave, in his capacity as Presidential
Executive Assistant, dismissed it.
ISSUE
WON private respondents are regular employees entitled to
security of tenure
HELD
NO
Ratio Casual employees are engaged for a specific project or
undertaking and fall within the exception provided for in Article
281 of the Labor Code, supra . Not being regular employees, it
cannot be justifiably said that petitioner had dismissed them
without just cause. They are not entitled to reinstatement with
full backwages.
Reasoning
- A281 defines regular and casual employees. In the case at
hand, the casual or limited character of private respondents'
employment, therefore, is evident.
- Private respondents were hired for a specific project - to
renovate the main budding, where major repairs such as
painting the main building, repair of the roof, cleaning of
clogged water pipes and drains, and other necessary repairs
were required.
- It was made known, and so understood at the start of the
hiring, that their services would last until the completion of the
renovation. They rendered service from February 2 to
December 11, 1976, almost 11 months, but less than a year.
- There could be no other reason, however, than that the
termination of private respondents was because their services
were no longer needed and they had nothing more to do since
the project for which they were hired had been completed.
Disposition Order of public respondent Vicente Leogardo, Jr.,
dated December 24, 1976, and the Orders of the other public
respondents dated July 13, 1977, January 25, 1979, March 19,
1979, and June 5, 1980, are hereby reversed and set aside. The
Complaint for illegal dismissal against petitioner in Case No.
R04-12-11832-76 LS (Regional Office No. IV, Department of
Labor) is dismissed, and the Temporary Restraining Order
heretofore issued is hereby made permanent.

SANDOVAL V NLRC
136 SCRA 675
AQUINO; May 31, 1985
NATURE
Appeal by certiorari
FACTS
- 5 workers were assigned to the construction of the LCT
Catamaran. After three months of work, the project was

A2010

- 102 -

Disini

completed and the five workers were served a termination


notice. The termination was reported to the Ministry of Labor.
The workers filed a complaint for illegal dismissal.
- The Labor Arbiter ordered the reinstatement of the workers
with backwages. The NLRC affirmed.
- 55 workers were assigned to work in the construction of a
tanker. When the tanker was finished, the personel manager of
Sandoval Shipyards terminated the services of the welders,
helpers, and construction workers. The termination was duly
reported to the Ministry of Labor. 17 workers filed a complaint
for illegal dismissal.
- The Director of the Ministrys Capital Region ordered the
reinstatement of the complainants. The Deputy Minister of
Labor affirmed. Hence this petition.
ISSUE
WON private respondents were project employees whose work
was coterminous with the project for which they were hired
HELD
YES
Ratio The public respondents in the instant two cases acted
with grave abuse of discretion amounting to lack of jurisdiction
in disregarding the precedents cited by the petitioners.
Reasoning
- Project Employees, as distinguished from regular or nonproject employees, are mentioned in Article 281 of the Labor
Code, as those where the employment has been fixed for a
specific project or undertaking the completion or termination of
which has been determined at the time of the engagement of
the employee.
- The petitioner cited three of its own cases wherein the NLRC,
Deputy Minister of Labor, and the Director of the National
Capital Region held that the layoff of its project employees was
lawful.
- In the case of In Re: Sandoval Shipyards, Inc. Application for
Clearance to Terminate Employees, it was held that:
It is significant to note that the corporation does not construct
vessels for sale or otherwise which will demand continuous
productions of ships and will need permanent or regular
workers
The completion of their work or project automatically
terminates their employment
- The other two cases cited affirmed that the workers of the
petitioner were project employees whose employment was
terminated upon the completion of the project.
- Respondent Deputy Minister himself affirmed such finding. He
ruled that the complainants are project workers whose
employments are coterminous with the completion of the
project, regardless of the number of projects in which they have
worked, as provided under Policy Instructions No. 20 of the
Ministry of Labor and Employment and as their employment is
one for a definite period, they are not entitled to separation
pay.
Disposition REVERSED.

IMBUIDO V NLRC (LIBRANDO)


329 SCRA 357
BUENA; March 31, 2000
NATURE
Petition for review on certiorari of the decision of the NLRC
FACTS
- Petitioner was employed as a data encoder by private
respondent International Information Services, Inc., a domestic
corporation engaged in the business of data encoding and
keypunching, from August 26, 1988 until October 18, 1991
when her services were terminated due to "low volume of
work".

Labor Law 1
- Petitioner filed a complaint for illegal dismissal with prayer for
service incentive leave pay and 13th month differential with
NLRC alleging that her employment was terminated not due to
the low volume of work but because she "signed a petition for
certification election among the rank and file employees of
respondents,"
thus
charging private respondent with
committing unfair labor practices.
- Private respondent maintained that it had valid reasons to
terminate petitioner's employment and disclaimed any
knowledge of the existence or formation of a union among its
rank-and-file employees at the time petitioner's services were
terminated. Private respondent stressed that its business ". . .
relies heavily on companies availing of its services. Its retention
by client companies with particular emphasis on data encoding
is on a project to project basis," usually lasting for a period of
"two (2) to five (5) months." Private respondent further argued
that petitioner's employment was for a "specific project with a
specified period of engagement." According to private
respondent, ". . . the certainty of the expiration of complainant's
engagement has been determined at the time of its
engagement (until 27 November 1991) or when the project is
earlier completed or when the client withdraws," as provided in
the contract. "The happening of the second event [completion
of the project] has materialized, thus, her contract of
employment is deemed terminated.
ISSUE
WON petitioner is a "project employee" and not a "regular
employee" who has security of tenure
HELD
- We agree with the findings of the NLRC that petitioner is a
project employee. The principal test for determining whether an
employee is a project employee or a regular employee is
whether the project employee was assigned to carry out a
specific project or undertaking, the duration and scope of which
were specified at the time the employee was engaged for that
project. A project employee is one whose employment has been
fixed for a specific project or undertaking, the completion or
termination of which has been determined at the time of the
engagement of the employee or where the work or service to
be performed is seasonal in nature and the employment is for
the duration of the season. In the instant case, petitioner was
engaged to perform activities which were usually necessary or
desirable in the usual business or trade of the employer, as
admittedly, petitioner worked as a data encoder for private
respondent, a corporation engaged in the business of data
encoding and keypunching, and her employment was fixed for a
specific project or undertaking the completion or termination of
which had been determined at the time of her engagement, as
may be observed from the series of employment contracts 32
between petitioner and private respondent, all of which
contained a designation of the specific job contract and a
specific period of employment.
- However, even as when petitioner is a project employee,
according to jurisprudence "[a] project employee or a member
of a work pool may acquire the status of aregular employee
when the following concur:
1) There is a continuous rehiring of project employees even
after the cessation of a project; and
2) The tasks performed by the alleged "project employee" are
vital, necessary and indispensable to the usual business or
trade of the employer.
- The evidence on record reveals that petitioner was employed
by private respondent as a data encoder, performing activities
which are usually necessary or desirable in the usual business
or trade of her employer, continuously for a period of more than
three (3) years, from August 26, 1988 to October 18, 1991 and
contracted for a total of thirteen (13) successive projects. We
have previously ruled that "[h]owever, the length of time during
which the employee was continuouslyre-hired is not controlling,

A2010

- 103 -

Disini

but merely serves as a badge of regular employment." Based


on the foregoing, we conclude that petitioner has attained the
status of a regular employee of private respondent.
- Being a regular employee, petitioner is entitled to security of
tenure and could only be dismissed for a just or authorized
cause, as provided in Article 279 of the
Labor Code, as amended:
Art. 279. Security of Tenure In cases of regular
employment, the employer shall not terminate the services of
an employee except for a just cause or when
authorized by this Title. An employee who is unjustly
dismissed from work shall be entitled to reinstatement
without loss of seniority rights and other privileges
and to his full backwages, inclusive of allowances, and to his
other benefits or their monetary equivalent computed from
the time his compensation was withheld
from him up to the time of his actual reinstatement.
- The alleged causes of petitioner's dismissal (low volume of
work and belatedly, completion of project) are not valid causes
for dismissal under Articles 282 and 283 of the Labor Code.
Thus, petitioner is entitled to reinstatement without loss of
seniority rights and other privileges, and to her full backwages,
inclusive of allowances, and to her other benefits or their
monetary equivalent computed from the time her compensation
was withheld from her up to the time of her actual
reinstatement. However, complying with the principles of
"suspension of work" and "no work, no pay" between the end of
one project and the start of a new one, in computing
petitioner's backwages, the amounts corresponding to what
could have been earned during the periods from the date
petitioner was dismissed until her reinstatement when private
respondent was not undertaking any project, should be
deducted.
- With regard to petitioner's claim for service incentive leave
pay, we agree with the labor arbiter that petitioner is entitled to
service incentive leave pay, as provided
in Article 95 of the Labor Code, which reads:
Art. 95: Right to service incentive leave
(a) Every employee who has rendered at least one year of
service shall be entitled to a yearly service incentive leave of
five days with pay.
- Having already worked for more than three (3) years at the
time of her unwarranted dismissal, petitioner is undoubtedly
entitled to service incentive leave benefits, computed from
1989 until the date of her actual reinstatement.
Disposition Petition granted.

DE OCAMPO V NLRC
186 SCRA 360
DAVIDE JR; May 7, 2002
NATURE
The petition seeks a reversal of the decision of the respondent
NLRC (ordering respondent to reinstate, without back
wages, the individual complainants who were regular
employees except those who were officers of the union among
them or paid separation pay at their option, equivalent to one
month's pay or one-half month's pay for every year of service,
whichever is greater. )
FACTS
- On September 30, 1980, the services of 65 employees of
private respondent Makati Development Corporation were
terminated on the ground of the expiration of their contracts.
The said employees filed a complaint for illegal dismissal
against the MDC on October 1, 1980; On October 8, 1980, as a
result of the aforementioned termination, the Philippine
Transport and General Workers Association, of which the
complainants were members, filed a notice of strike on the
grounds of union-busting, subcontracting of projects which
could have been assigned to the dismissed employees, and

Labor Law 1
unfair labor practice; that on October 14, 1980, the PTGWA
declared a strike and established picket lines in the perimeter of
the MDC premises- On November 4, 1980, the MDC filed with
the Bureau of Labor Relations a motion to declare the strike
illegal and restrain the workers from continuing the strike; that
on that same day and several days thereafter the MDC filed
applications for clearance to terminate the employment of 90 of
the striking workers, whom it had meanwhile preventively
suspended; that of the said workers, 74 were project employees
under contract with the MDC with fixed terms of employment;
and that on August 31, 1982, Labor Arbiter Apolinar L. Sevilla
rendered a decision 1 denying the applications for clearance
filed by the MDC and directing it to reinstate the individual
complainants with two months back wages each.
- This is the decision modified by the NLRC 2 which is now
faulted by the petitioners for grave abuse of discretion. The
contention is that the public respondent acted arbitrarily and
erroneously in ruling that: a) the motion for reconsideration was
filed out of time; b) the strike was illegal; and c) the separation
of the project employees was justified
ISSUES

1.

WON the strike held by the workers was legal


2. WON the contract workers are considered regular employees
3. WON the project workers are entitled to separation pay
HELD
1. YES
- under the law then in force, to wit, PD No. 823 as amended by
PD No. 849, the strike was indeed illegal. In the first place, it
was based not on the ground of unresolved economic issues,
which was the only ground allowed at that time, when the
policy was indeed to limit and discourage strikes. Secondly, the
strike was declared only after 6 days from the notice of strike
and before the lapse of the 30-day period prescribed in the said
law for a cooling-off of the differences between the workers and
management and a possible avoidance of the intended strike.
That law clearly provided Sec. 1. It is the policy of the state to
encourage free trade unionism and free collective bargaining
within the framework of compulsory and voluntary arbitration.
Therefore all forms of strikes, picketing and lockout are hereby
strictly prohibited in vital industries such as in public utilities,
including transportation and communication, companies
engaged in the manufacturer processing as well as in the
distribution of fuel gas, gasoline and fuel or lubricating oil, in
companies engaged in the production or processing of essential
commodities or products for export, and in companies engaged
in banking of any kind, as well as in hospitals and in schools and
colleges. However, any legitimate labor union may strike and
any employer may lockout in establishments not covered by
General Order No. 5 only on grounds of unresolved economic
issues in collective bargaining, in which case the union or the
employer shall file a notice with the Bureau of Labor Relations
at least 30 days before the intended strike or lockout.
(Emphasis supplied)
The Court ruled that the leaders of the illegal strike were
correctly punished with dismissal, but their followers (other
than the contract workers) were properly ordered reinstated,
considering their lesser degree of responsibility. The penalty
imposed upon the leaders was only proper because it was they
who instigated the strike even if they knew, or should have
known, that it was illegal. It was also fair to rule that the
reinstated strikers were not entitled to backpay as they
certainly should not be compensated for services not rendered
during the illegal strike. In our view, this is a reasonable
compromise between the demands of the workers and the
rights of the employer.
2. The Court stress the rule in Cartagenas v. Romago Electric
Co.,
that contract workers are not considered regular
employees, their services being needed only when there are
projects to be undertaken. 'The rationale of this rule is that if a
project has already been completed, it would be unjust to

A2010

- 104 -

Disini

require the employer to maintain them in the payroll while they


are doing absolutely nothing except waiting until another
project is begun, if at all. In effect, these stand-by workers
would be enjoying the status of privileged retainers, collecting
payment for work not done, to be disbursed by the employer
from profits not earned. This is not fair by any standard and can
only lead to a coddling of labor at the expense of management.
However, this rule is not applicable in the case at bar. The
record shows that although the contracts of the project workers
had indeed expired, the project itself was still on-going and so
continued to require the workers' services for its completion.
There is no showing that such services were unsatisfactory to
justify their termination. It is obvious that the real reason for the
termination of their services-which, to repeat, were still neededwas the complaint the project workers had filed and their
participation in the strike against the private respondent. These
were the acts that rendered them persona non grata to the
management. Their services were discontinued by the MDC not
because of the expiration of their contracts, which had not
prevented their retention or rehiring before as long as the
project they were working on had not yet been completed. The
real purpose of the MDC was to retaliate against the workers, to
punish them for their defiance by replacing them with more
tractable employees.
3. Noteworthy in this connection is Policy Instruction No. 20 of
the Department of Labor, providing that "project employees are
not entitled to separation pay if they are terminated as a result
of the completion of the project or any phase thereof in which
they are employed, regardless of the projects in which they had
been employed by a particular construction company." This rule
would entitle project employees to separation pay if the
projects they are working on have not yet been completed
when their services are terminated. And this should be true
even if their contracts have expired, on the theory that such
contracts would have been renewed anyway because their
services were still needed.
- Applying this rule, The Court held that the project workers who
were separated even before the completion of the project at the
New Alabang Village and not really for the reason that their
contracts had expired, are entitled to separation pay.
Considering the workers to have been separated without valid
cause, the Court shall compute their separation pay at the rate
of one month for every year of service of each dismissed
employee, up to the time of the completion of the project. This
is the most equitable way to treat their claim in light of their
cavalier dismissal by the private respondent despite their long
period of satisfactory service with it.
Disposition The appealed decision of the NLRC is affirmed but
with the modification that the contract workers are hereby
declared to have been illegally separated before the expiration
of the project they were working on and so are entitled to
separation pay equivalent to one month salary for every year of
service.

A.M. ORETA & CO INC V NLRC (GRULLA)


176 SCRA 218
MEDIALDEA; August 10, 1989
NATURE
Petition for certiorari
FACTS
- Private respondent Grulla was engaged by Engineering
Construction and Industrial Development Company (ENDECO)
through A.M. Oreta and Co., Inc. as a carpenter in its project in
Jeddah, Saudi Arabia.
- The contract of employment, which was entered into on June
11, 1980 was for a period of 12 months. Respondent Grulla left
the Philippines for Jeddah, Saudi Arabia on August 5, 1980.
- On August 15, 1980, Grulla met an accident which fractured
his lumbar vertebrae while working at the jobsite. He was

Labor Law 1
rushed to the New Jeddah Clinic and was confined there for 12
days.
- On August 27, 1980, Grulla was discharged from the hospital
and was told that he could resume his normal duties after
undergoing physical therapy for two weeks.
- On September 18, 1980, respondent Grulla reported back to
his Project Manager and presented to the latter a medical
certificate declaring the former already physically fit for work.
Since then, he stated working again until he received a notice of
termination of his employment on October 9, 1980.
- Grulla filed a complaint for illegal dismissal, recovery of
medical benefits, unpaid wages for the unexpired ten (10)
months of his contract and the sum of P1,000.00 as
reimbursement of medical expenses against A.M. Oreta and
Company, Inc. and ENDECO with the POEA.
- The petitioner A.M. Oreta and Company, Inc. and ENDECO filed
their answer and alleged that the contract of employment
entered into between petitioners and Grulla provides, as one of
the grounds for termination of employment, violation of the
rules and regulations promulgated by the contractor; and that
Grulla was dismissed because he has not performed his duties
satisfactorily within the probationary period of three months.
- POEA held that complainant's dismissal was illegal and
warrants the award of his wages for the unexpired portion of
the contract.
- Petitioner appealed from the adverse decision to the
respondent Commission.
- Respondent Commission dismissed the appeal for lack of merit
and affirmed in toto the decision of the POEA..
ISSUES
1. WON the employment of respondent Grulla was illegally
terminated by the petitioner
2. WON Grulla is entitled to salaries corresponding to the
unexpired portion of his employment contract.
HELD
1. YES
- Article 280 (formerly Article 281) of the Labor Code, as
amended, provides:
"Article 280.
Regular and Casual Employment. - The
provisions of written agreement to the contrary
notwithstanding and regardless of the oral agreements of
the parties, an employment shall be deemed to be regular
where the employee has been engaged to perform
activities which are usually necessary or desirable in the
usual business or trade of the employer, except where the
employment has been fixed for a specific project or
undertaking the completion or termination of which has
been determined at the time of the engagement of the
employment or where the work or service to be performed
is seasonal in nature and the employment is far the
duration of the season.
"An employment shall be deemed to be casual if it is not
covered by the preceding paragraph: Provided, that any
employee who has rendered at least one year of service,
whether such service is continuous or broken, shall be
considered a regular employee with respect to the activity
in which he is employed and his employment shall continue
while such actually exists."
- Policy Instructions No. 12 of the then Minister of Labor (now
Secretary of Labor and Employment) which provides:
"PD 850 has defined the concept of regular and casual
employment. What determines regularity or casualness is
not the employment contract, written or otherwise, but the
nature of the job. If the job is usually necessary or
desirable to the main business of the employer, then
employment is regular. . . ."
- A perusal of the employment contract reveals that although
the period of employment of respondent Grulla is 12 months,
the contract period is renewable subject to future agreement of
the parties. It is clear from the employment contract that the

A2010

- 105 -

Disini

respondent Grulla was hired by the company as a regular


employee and not just a mere probationary employee.
- On the matter of probationary employment, the law in point is
Article 281 (formerly Article 252) of the Labor Code which
provides in part:
"Art. 281. Probationary Employment. - . . . . The services of
an employee who has been engaged on a probationary
basis may be terminated for a just cause or when he fails
to qualify as a regular employee in accordance with
reasonable standards made known by the employer to the
employee at the time of his engagement. An employee
who is allowed to work after a probationary period shall be
considered regular employee."
- The law is clear to the effect that in all cases involving
employees engaged on probationary' basis, the employer shall
make known to the employee at the time he is hired, the
standards by which he will qualify as a regular employee.
Nowhere in the employment contract executed between
petitioner company and respondent Grulla is there a stipulation
that the latter shall undergo a probationary period for three
months before he can quality as a regular employee. There is
also no evidence on record showing that the Grulla had been
apprised of his probationary status and the requirements which
he should comply in order to be a regular employee. In the
absence of these requisites, there is justification in concluding
that respondent Grulla was a regular employee at the time he
was dismissed by petitioner.
As such, he is entitled to security of tenure during his period of
employment and his services cannot be terminated except for
just and authorized causes enumerated under the Labor Code
and under the emloyment contract.
Granting, in gratia argumenti, that respondent is a probationary
employee, he cannot, likewise, be removed except for cause
during the period of probation. Although a probationary or
temporary employee has limited tenure, he still enjoys security
of tenure. During his tenure of employment or before his
contract expires, he cannot be removed except for cause as
provided for by law.
- The alleged ground of unsatisfactory performance relied upon
by petitioner for dismissing respondent Grulla is not one of the
just causes for dismissal provided in the Labor Code. Neither is
it included among the grounds for termination of employment
under Article VII of the contract of employment executed by
petitioner company and respondent Grulla.
- Grulla was not, in any manner, notified of the charges against
him before he was outrightly dismissed. Neither was any
hearing or investigation conducted by the company to give the
respondent a chance to be heard concerning the alleged
unsatisfactory performance of his work.
2. YES
- The dismissal of Grulla violated the security of tenure under
the contract of employment which specifically provides that the
contract term shall be for a period of twelve (12) calendar
months. Consequently, the respondent Grulla should be paid his
salary for the unexpired portion of his contract of employment
which is ten (10) months.
Disposition Petition was dismissed

PURE FOODS CORPORATION V NLRC (CORDOVA,


CRUSIS, ET AL)
174 SCRA 415
DAVIDE, JR; December 12, 1997
NATURE
Petition for certiorari
FACTS
- Private respondents (numbering 906) were hired by Pure
Foods Corporation (PFC) to work for a fixed period of five
months at its tuna cannery plant in Tambler, General Santos
City. After the expiration of their respective contracts of

Labor Law 1
employment in June and July 1991, their services were
terminated. They forthwith executed a "Release and Quitclaim"
stating that they had no claim whatsoever against PFC.
- 29 July 1991: private respondents filed before the NLRC SubRegional Arbitration Branch a complaint for illegal dismissal
against PFC and its plant manager, Marciano Aganon. Labor
Arbiter Arturo P. Aponesto dismissed the complaint on the
ground that the private respondents were mere contractual
workers, and not regular employees; hence, they could not avail
of the law on security of tenure. The termination of their
services by reason of the expiration of their contracts of
employment was, therefore, justified.
- On appeal, NLRC affirmed the Labor Arbiter's decision. But on
MR, NLRC held that the private respondent and their cocomplainants were regular employees. It declared that the
contract of employment for five months was a "clandestine
scheme employed by PFC to stifle private respondents' right to
security of tenure" and should therefore be struck down and
disregarded for being contrary to law, public policy, and morals.
Hence, their dismissal on account of the expiration of their
respective contracts was illegal.
- Accordingly, the NLRC ordered PFC to reinstate the private
respondents to their former position without loss of seniority
rights and other privileges, with full back wages; and in case
their reinstatement would no longer be feasible, PFC should pay
them separation pay equivalent to one-month pay or one-halfmonth pay for every year of service, whichever is higher, with
back wages and 10% of the monetary award as attorney's fees.
- PFC's motion for reconsideration was denied. Hence, this
petition.
Purefoods Corp's Contention: that the private respondents
are now estopped from questioning their separation from
petitioner's employ in view of their express conformity with the
five-month duration of their employment contracts; that the
"Release and Quitclaim" private respondents had executed has
unconditionally released PFC from any and all other claims
which might have arisen from their past employment with PFC.
OSG's Comment: the private respondents were regular
employees, since they performed activities necessary and
desirable in the business or trade of PFC. The period of
employment stipulated in the contracts of employment was null
and void for being contrary to law and public policy, as its
purpose was to circumvent the law on security of tenure. The
expiration of the contract did not, therefore, justify the
termination of their employment. Also, private respondents'
quitclaim was ineffective to bar the enforcement for the full
measure of their legal rights.
Private Respondent's Argument: contracts with a specific
period of employment may be given legal effect provided,
however, that they are not intended to circumvent the
constitutional guarantee on security of tenure; the practice of
PFC in hiring workers to work for a fixed duration of five months
only to replace them with other workers of the same
employment duration was apparently to prevent the
regularization of these so-called "casuals," which is a clear
circumvention of the law on security of tenure.
ISSUES
1. WON employees hired for a definite period and whose
services are necessary and desirable in the usual business or
trade of the employer are regular employees
2. WON the private respondents' five-month contracts of
employment are valid
3. WON the execution by the private respondents of a "Release
and Quitclaim" precluded them from questioning the
termination of their services
HELD
1. YES
- Art. 280 of the Labor Code defines regular and casual
employment. Under this provision, there are two kinds of
regular employees: (1) those who are engaged to perform

A2010

- 106 -

Disini

activities which are necessary or desirable in the usual business


or trade of the employer; and (2) those casual employees who
have rendered at least one year of service, whether continuous
or broken, with respect to the activity in which they are
employed.
- In the instant case, the private respondents' activities
consisted in the receiving, skinning, loining, packing, and
casing-up of tuna fish which were then exported by PFC.
Indisputably, they were performing activities which were
necessary and desirable in petitioner's business or trade.
- not hired for a specific project or undertaking. The term
"specific project or undertaking" under Article 280 of the
Labor Code contemplates an activity which is not commonly or
habitually performed or such type of work which is not done on
a daily basis but only for a specific duration of time or until
completion; the services employed are then necessary and
desirable in the employer's usual business only for the period of
time it takes to complete the project.
2. NO
- None of the criteria, under which term employment cannot be
said to be in circumvention of the law on security of tenure, had
been met in the present case.
-Brent School, Inc. v. Zamora ruling on the legality of fixed-term
employment has held that the decisive determinant in term
employment should not be the activities that the employee is
called upon to perform but the day certain agreed upon by the
parties for the commencement and termination of their
employment relationship. But, where from the circumstances it
is apparent that the periods have been imposed to preclude
acquisition of tenurial security by the employee, they should be
struck down or disregarded as contrary to public policy and
morals.
-Criteria under which term employment cannot be said
to be in circumvention of the law on security of tenure:
a) The fixed period of employment was knowingly and
voluntarily agreed upon by the parties without any force,
duress, or improper pressure being brought to bear upon the
employee and absent any other circumstances vitiating his
consent; or
b) It satisfactorily appears that the employer and the employee
dealt with each other on more or less equal terms with no moral
dominance exercised by the former over the latter.
- It was shown that it was really the practice of the company to
hire workers on a uniformly fixed contract basis and replace
them upon the expiration of their contracts with other workers
on the same employment duration. This scheme of PFC was
apparently designed to prevent the private respondents and the
other "casual" employees from attaining the status of a regular
employee. It was a clear circumvention of the employees' right
to security of tenure and to other benefits like minimum wage,
cost-of-living allowance, sick leave, holiday pay, and 13th
month pay.
3. NO
- The execution by the private respondents of a "Release and
Quitclaim" did not preclude them from questioning the
termination of their services. Generally, quitclaims by laborers
are frowned upon as contrary to public policy and are held to be
ineffective to bar recovery for the full measure of the workers'
rights. The reason for the rule is that the employer and the
employee do not stand on the same footing.
Disposition Petition dismissed. NLRC decision affirmed, subject
to modification on the computation of the separation pay and
back wages. Since reinstatement is no longer possible because
PFC's tuna cannery plant had, admittedly, been close in
November 1994, the proper award is separation pay equivalent
to one month pay or one-half month pay for every year of
service, whichever is higher, to be computed from the
commencement of their employment up to the closure of the
tuna cannery plant. The amount of back wages must be
computed from the time the private respondents were

Labor Law 1
dismissed until the time petitioner's cannery plant ceased
operation.

LABAYOG V MY SAN BISCUITS INC


494 SCRA 486
CORONA; July 11, 2006
NATURE
Petition for review on certiorari is the resolution of the Court of
Appeals.
FACTS
- 1992: petitioners entered into contracts of employment with
respondent company as mixers, packers and machine operators
for a fixed term. On the expiration of their contracts, their
services were terminated.
- April 15, 1993: petitioners filed complaints for illegal dismissal,
underpayment of wages, non-payment of overtime, night
differential and 13th month pay, damages and attorneys fees.
The labor arbiter ruled their dismissal to be illegal on the
ground that they had become regular employees who
performed duties necessary and desirable in respondent
companys business..
- On appeal to the National Labor Relations Commission (NLRC),
the decision of the labor arbiter was set aside.
- CA set aside the NLRC decision and reinstated the decision of
the labor arbiter. However, on respondents motion for
reconsideration, the CA reversed itself. The CA reasoned that,
while petitioners performed tasks which were necessary and
desirable in the usual business of respondent company, their
employment contracts providing for a fixed term remained
valid. No force, duress, intimidation or moral dominance was
exerted on them. Respondents dealt with petitioners in good
faith and within the valid parameters of management
prerogatives.
ISSUE
WON the contract with a fixed period is valid (therefore
determining WON the workers were dismissed illegally).
HELD
YES, contract is valid.
Ratio Contracts of employment for a fixed period are not
unlawful. What is objectionable is the practice of some
scrupulous employers who try to circumvent the law protecting
workers from the capricious termination of employment.
Reasoning
- Petitioners were not regular employees. While their
employment as mixers, packers and machine operators was
necessary and desirable in the usual business of respondent
company, they were employed temporarily only, during periods
when there was heightened demand for production. There could
have been no illegal dismissal when their services were
terminated on expiration of their contracts.
ART. 280. (Labor Code) Regular and Casual Employment.
The provisions of written agreement to the contrary
notwithstanding and regardless of the oral agreement of the
parties, an employment shall be deemed to be regular where
the employee has been engaged to perform activities which
are usually necessary or desirable in the usual business of the
employer, except where the employment has been fixed for a
specific project or undertaking the completion or termination
of which has been determined at the time of the engagement
of the employee or where the work or services to be
performed is seasonal in nature and the employment is for
the duration of the season.
Where the duties of the employee consist of activities
which are necessary or desirable in the usual business of the
employer, the parties are not prohibited from agreeing on the
duration of employment.

A2010

- 107 -

Disini

- Article 280 does not proscribe or prohibit an employment


contract with a fixed period[11] provided it is not intended to
circumvent the security of tenure.
Two criteria validate a contract of employment with a
fixed period:
(1) The fixed period of employment was knowingly and
voluntarily agreed upon by the parties without any
force, duress or improper pressure being brought to
bear on the employee and without any circumstances
vitiating consent;
(2) It satisfactorily appears that the employer and
employee dealt with each other on more or less equal
terms with no moral dominance whatever being
exercised by the former on the latter.
Disposition Resolution of CA is affirmed.

CHUA V CA (SOCIAL SECURITY COMMISSION, SSS,


PAGUIO ET AL)
440 SCRA 121
TINGA; October 6, 2004
NATURE
This is a petition for review of the Decision of the Court of
Appeals in CA-G.R. CV No. 38269 dated 06 March 1996, and its
Resolution dated 30 July 1996 denying petitioners Motion for
Reconsideration, affirming the Order of the Social Security
Commission (SSC) dated 1 February 1995 which held that
private respondents were regular employees of the petitioner
and ordered petitioner to pay the Social Security System (SSS)
for its unpaid contributions, as well as penalty for the delayed
remittance thereof.
FACTS
- On 20 August 1985, private respondents Andres Paguio, Pablo
Canale, Ruel Pangan, Aurelio Paguio, Rolando Trinidad, Romeo
Tapang and Carlos Maliwat (hereinafter referred to as
respondents) filed a Petition with the SSC for SSS coverage and
contributions against petitioner Reynaldo Chua, owner of Prime
Mover Construction Development, claiming that they were all
regular employees of the petitioner in his construction business.
Private respondents alleged that petitioner dismissed all of
them without justifiable grounds and without notice to them and
to the then Ministry of Labor and Employment. They further
alleged that petitioner did not report them to the SSS for
compulsory coverage in flagrant violation of the Social Security
Act.
- On the other hand, the petitioner claimed that private
respondents were project employees, whose periods of
employment were terminated upon completion of the project.
Thus, he claimed, no employer-employee relation existed
between the parties. There being no employer-employee
relationship, private respondents are not entitled to coverage
under the Social Security Act. Moreover, petitioner invokes the
defense of good faith, or his honest belief that project
employees are not regular employees under Article 280 of the
Labor Code. The SSC and CA ruled in favor of the respondents.
ISSUE
WON private respondents were regular employees of the
petitioner
HELD
YES
Ratio
Elements of the control test: (a) selection and
engagement of the employee; (b) payment of wages; (c) the
power of dismissal; and (d) the power of control with regard to
the means and methods by which the work is to be
accomplished, with the power of control being the most
determinative factor.
- Even though the employer does not admit, the existence of an
employer-employee relationship between the parties can easily

Labor Law 1
be determined by the application of the "control test, the
elements of which are: (a) selection and engagement of the
employee; (b) payment of wages; (c) the power of dismissal;
and (d) the power of control with regard to the means and
methods by which the work is to be accomplished, with the
power of control being the most determinative factor.
- There is no dispute that private respondents were employees
of petitioner. Petitioner himself admitted that they worked in his
construction projects, although the period of their employment
was allegedly co-terminus with their phase of work. It is clear
that private respondents are employees of petitioner, the latter
having control over the results of the work done, as well as the
means and methods by which the same were accomplished.
Suffice it to say that regardless of the nature of their
employment, whether it is regular or project, private
respondents are subject of the compulsory coverage under the
SSS Law, their employment not falling under the exceptions
provided by the law. This rule is in accord with the Courts ruling
in Luzon Stevedoring Corp. v. SSS to the effect that all
employees, regardless of tenure, would qualify for compulsory
membership in the SSS, except those classes of employees
contemplated in Section 8(j) of the Social Security Act.
- In Violeta v. NLRC, this Court ruled that to be exempted from
the presumption of regularity of employment, the agreement
between a project employee and his employer must strictly
conform to the requirements and conditions under Article 280
of the Labor Code. It is not enough that an employee is hired for
a specific project or phase of work. There must also be a
determination of, or a clear agreement on, the completion or
termination of the project at the time the employee was
engaged if the objectives of Article 280 are to be achieved. This
second requirement was not met in this case.
- This Court has held that an employment ceases to be coterminus with specific projects when the employee is
continuously rehired due to the demands of the employers
business and re-engaged for many more projects without
interruption. The Court likewise takes note of the fact that, as
cited by the SSC, even the National Labor Relations Commission
in a labor case involving the same parties, found that private
respondents were regular employees of the petitioner.
- Another cogent factor militates against the allegations of the
petitioner. In the proceedings before the SSC and the Court of
Appeals, petitioner was unable to show that private
respondents were appraised of the project nature of their
employment, the specific projects themselves or any phase
thereof undertaken by petitioner and for which private
respondents were hired. He failed to show any document such
as private respondents employment contracts and employment
records that would indicate the dates of hiring and termination
in relation to the particular construction project or phases in
which they were employed. Moreover, it is peculiar that
petitioner did not show proof that he submitted reports of
termination after the completion of his construction projects,
considering that he alleges that private respondents were hired
and rehired for various projects or phases of work therein
- To be exempted from the presumption of regularity of
employment, the agreement between a project employee and
his employer must strictly conform to the requirements and
conditions under Article 280 of the Labor Code. It is not enough
that an employee is hired for a specific project or phase of work.
There must also be a determination of, or a clear agreement on,
the completion or termination of the project at the time the
employee was engaged if the objectives of Article 280 are to be
achieved.

C.E. CONSTRUCTION CORP V CIOCO


437 SCRA 648
PUNO; September 8, 2004
NATURE

A2010

- 108 -

Disini

Two (2) petitions for review of the Decision of the CA which


reversed the NLRC, as well as its Resolution which denied the
parties separate motions for reconsideration.
FACTS
- Cioco, et al. (WORKERS) were hired by C.E. Construction Corp.
(COMPANY), a domestic corporation engaged in the
construction business. They were hired as carpenters and
laborers in various construction projects from 1990 to 1999, the
latest of which was the GTI Tower in Makati. Prior to the start of
every project, the WORKERS signed individual employment
contracts.
- Sometime in May and June 1999, the WORKERS, along with
sixty-six (66) others, were terminated by the COMPANY on the
ground of completion of the phases of the GTI Tower project for
which they had been hired. Alleging that they were regular
employees, the WORKERS filed complaints for illegal dismissal
with the NLRC. Claims for underpaid wages and unpaid
overtime pay, premium for holiday and rest days, service
incentive leave pay, night shift differential, and 13th month pay
were likewise demanded.
ISSUES
1. WON the WORKERS were regular employees of the COMPANY
2. WON the WORKERS were illegally dismissed
HELD
1. NO
Ratio The Labor Arbiter, the NLRC, and the CA, unanimously
found that the WORKERS were project employees of the
COMPANY. This finding is binding on this Court. We again hold
that the fact that the WORKERS have been employed with the
COMPANY for several years on various projects, the longest
being nine (9) years, did not automatically make them regular
employees considering that the definition of regular
employment in Article 280 of the Labor Code, makes specific
exception with respect to project employment. The re-hiring of
petitioners on a project-to-project basis did not confer upon
them regular employment status.
2. NO
Ratio The labor arbiter categorically found that the appropriate
notices to the WORKERS and the corresponding reports were
submitted by the COMPANY to the DOLE. The NLRC affirmed
this finding of fact on appeal. The rule is that factual findings of
administrative agencies, if supported by substantial evidence,
are entitled to great weight. More importantly, no prior notice of
termination is required if the termination is brought about by
completion of the contract or phase thereof for which the
worker has been engaged.
Disposition
The decision of the CA is MODIFIED. The
termination of the WORKERS is declared valid and legal. The
award of backwages is set aside.

MARAGUINOT V NLRC
[PAGE 82]
AGUILAR V NLRC (ACEDILLO)
269 SCRA 596
ROMERO; March 13, 1997
NATURE
Petition for certiorari
FACTS
- Romeo Acedillo worked for the petitioner as a helperelectrician. He was dismissed from allegedly due to lack of
available projects and excess in the number of workers needed.
- He filed a case for illegal dismissal before the NLRC upon
learning that the petitioner as hiring new workers and his
request to be reinstated was denied. In reply, petitioner

Labor Law 1
maintained that its need for workers varied, depending on
contracts procured in the course of its business of contracting
refrigeration and other related works. According to them,
Acedillo was a contractual employee.
- NLRC ruled in favor of Acedilo. It held that Acedillo was a
regular employee, as seen from the nature of his job and the
length of time he has served. The petitioner was also held liable
for the monetary benefits being claimed by Acedillo since
employees, whether regular or not, are entitled to such.
ISSUE
WON Acedillo is a regular employee
HELD
YES
- definition of project employee: a project employee is one
whose "employment has been fixed for a specific project or
undertaking, the completion or termination of which has been
determined at the time of the engagement of the employee or
where the work or services to be performed is seasonal in
nature and the employment is for the duration of the season
- petitioner did not specify the duration and scope of the
undertaking at the time Acedillo's services were contracted.
Neither is there any proof that the duration of his assignment
was made clear to him other than the self-serving assertion of
petitioner that the same can be inferred from the tasks he was
made to perform
- Acedillos work clearly was an activity "necessary or desirable
in the usual business or trade" of petitioner, since refrigeration
requires considerable electrical work. This necessity is further
bolstered by the fact that petitioner would hire him anew after
the completion of each project, a practice which persisted
throughout the duration of his tenure
- Petitioners assertion that it held 2 sets of workers. This
practice renders untenable petitioner's position that Acedillo is
not a regular employee. Citing Philippine National Construction
Corp v NLRC: "Members of a work pool from which a
construction company draws its project employees, if
considered employees of the construction company while in the
work pool, are non-project employees or employees for an
indefinite period. If they are employed in a particular project,
the completion of the project or any phase thereof will not
mean severance of (the) employer-employee relationship."
Disposition Petition dismissed

ABESCO CONSTRUCTION AND DEVELOPMENT CO V


RAMIREZ
[PAGE 83]
PALOMARES V NLRC (NATIONAL STEEL
CORPORATION)
277 SCRA 439
ROMERO; August 15, 1997
FACTS
- Palomares and Mutia was hired by respondent National Steel
Corporation (NSC) by virtue of contracts of employment for its
Five Year Expansion Program or FYEP, Phase I and II-4, for
varying lengths of time. Palomares and Mutia asked for
regularization, wage differential, CBA coverage and other
benefits. Palomares, Mutia and four other complainants were
adjudged as regular employees of NSC. The NLRC reversed the
findings of the Labor Arbiter. Respondent Commission held that
petitioners were project employees and that their assumption of
regular jobs were mainly due to peakloads or the absence of
regular employees during the latter's temporary leave.
- Petitioners argue that as regards functions and duration of
work, contracted employees should, by operation of law, be
considered regular employees. Respondent NSC, on the other
hand, maintains that petitioners are mere project employees,

A2010

- 109 -

Disini

engaged to work on the latter's Five-Year Expansion Projects


(FYEP), Phases I and II-A, hence, dismissible upon the expiration
of every particular project.
- Petitioners were employed for a specific project or projects
undertaken by respondent corporation such as the Five Year
Expansion Program include the setting up of a Cold Rolling Mill
Expansion Project, establishing a Billet Steel-Making Plant,
installation of a Five Stand TDM and Cold Mill Peripherals
Project.
- Petitioners were hired to work on projects for FYEP I and II-A as
shown in the records. On account of the expiration of their
contracts of employment and/or project completion, petitioners
were terminated from their employment. They were, however,
rehired for other component projects of the FYEP because they
were qualified. Thus, the Court is convinced that petitioners
were engaged only to augment the workforce of NSC for its
aforesaid expansion program.
ISSUE
WON petitioners should be considered regular employees of
respondent corporation
HELD
NO
- They should not be. It should be noted that there were
intervals in petitioners' respective employment contracts with
NSC, thus bolstering the latter's position that, indeed,
petitioners are project employees. Since its work depends on
availability of such contracts or projects, necessarily the
employment of its work force is not permanent but coterminous with the projects to which they are assigned
and from whose payrolls they are paid. It would be
extremely burdensome for their employer to retain them as
permanent employees and pay them wages even if there are no
projects to work on. The fact that petitioners worked for NSC
under different project employment contracts for several years
cannot be made a basis to consider them as regular employees,
for they remain project employees regardless of the number of
projects in which they have worked.
Even if petitioners were repeatedly and successively re-hired on
the basis of a contact of employment for more than one year,
they cannot be considered regularized. Length of service is
not the controlling determinant of the employment
tenure of a project employee. As stated earlier, it is
based on whether or not the employment has been fixed
for a specific project or undertaking, the completion of
which has been determined at the time of the
engagement of the employee. Furthermore, the second
paragraph of Article 280, providing that an employee who has
rendered service for at least one (1) year, shall be considered a
regular employee, pertains to casual employees and not to
project employees such as petitioners.
-The principal test for determining whether an employee
is a project employee and not a regular employee is
whether he was assigned to carry out a specific project
or undertaking, the duration and scope of which were
specified at the time he was engaged for that project.
Disposition instant petition is DISMISSED. The decision and
resolution of the National Labor Relations Commission dated
November 23, 1994 and March 23, 1995, respectively, are
AFFIRMED.

FILIPINAS PRE-FABRICATED BUILDING SYSTEMS


INC V PUENTE
453 SCRA 820
PANGANIBAN; March 18, 2005
NATURE
Petition for Review
FACTS

Labor Law 1
- Respondent Puentes contention:
> That he began working with Petitioner Filsystems, Inc., a
corporation engaged in construction business, on June 12, 1989;
that he was initially hired by [petitioner] company as an
installer; that he was later promoted to mobile crane operator
and was stationed at the company premises in Quezon City;
that his work was not dependent on the completion or
termination of any project; that since his work was not
dependent on any project, his employment with the Filsystems
was continuous and without interruption for the past 10 years;
that on Oct. 1, 1999, he was dismissed from his employment
allegedly because he was a project employee.
He filed
complaint for illegal dismissal against the petitioner.
- Petitioner-companys claims
> That complainant was hired as a project employee in the
companys various projects; that his employment contracts
showed that he was a project worker with specific project
assignments; that after completion of each project assignment,
his employment was likewise terminated and the same was
correspondingly reported to the DOLE.
Labor Arbiter dismissed complaint. NLRC affirmed. CA reversed
LA and NLRC rulings holding that respondent was a regular
employee of petitioners.
ISSUES
1. WON respondent Roger Puente is a project employee
2. WON he is entitled to reinstatement with full back wages
HELD
1. YES
Ratio Provisions in the Labor Code and DOLE Order No.
19(1993) make it clear that a project employee is one whose
employment has been fixed for a specific project or
undertaking the completion or termination of which has been
determined at the time of the engagement of the employee or
where the work or services to be performed is seasonal in
nature and the employment is for the duration of the season. It
is a settled rule that the length of service of a project
employee is not the controlling test of employment tenure but
whether or not the employment has been fixed for a specific
project or undertaking the completion or termination of which
has been determined at the time of the engagement of the
employee.
Reasoning
1) The contracts of employment of Puente show that he was
hired for specific projects. His employment was coterminous
with the completion of the projects for which he had been hired.
Those contracts expressly provided that his tenure of
employment depended on the duration of any phase of the
project or on the completion of the construction projects. Also,
petitioners regularly submitted to DOLE reports of the
termination of services of project workers. Such compliance
with the requirement confirms that respondent was a project
employee
2) Evidently, although the employment contract did not state a
particular date, it did specify that the termination of the parties
employment relationship was to be on a day certain -- the day
when the phase of work termed Lifting & Hauling of Materials
for the World Finance Plaza project would be completed.
Thus, respondent cannot be considered to have been a regular
employee. He was a project employee.
3) That he was employed with Filsystems for 10 yrs. in various
projects did not ipso facto make him a regular employee,
considering that the definition of regular employment in Art.280
of the Labor Code makes a specific exception with respect to
project employment. The mere rehiring of respondent on a
project-to-project basis did not confer upon him regular
employment status. The practice was dictated by the practical
consideration that experienced construction workers are more
preferred. It did not change his status as a project employee.
2. YES

A2010

- 110 -

Disini

Ratio In termination cases, the burden of proving that an


employee has been lawfully dismissed lies with the employer.
Employers who hire project employees are mandated to state
and, once its veracity is challenged, to prove the actual basis
for the latters dismissal.
Reasoning
- There is no allegation or proof, however, that the World
Finance Plaza project -- or the phase of work therein to which
respondent had been assigned -- was already completed by
Oct.1, 99, the date when he was dismissed. The inescapable
presumption is that his services were terminated for no valid
cause prior to the expiration of the period of his employment;
hence, the termination was illegal. Reinstatement with full back
wages, inclusive of allowances and other benefits or their
monetary equivalents -- computed from the date of his
dismissal until his reinstatement -- is thus in order. If
reinstatement no longer possible due to the completion of the
project during the pendency of this case, he must be entitled to
salary and benefits of the unexpired portion of his employment.
Disposition Petition is PARTLY GRANTED.

You might also like